Download as pdf or txt
Download as pdf or txt
You are on page 1of 262

‫‪RN.

ZAHRA ACTIVITIES‬‬
‫تم بحمد هللا و توفيقه تحديث ملفات األنشطة الثالث الموجودة في خطة المذاكرة على القنوات التالية‪:‬‬

‫‪ -1‬قناة تجميعات سعيد‬


‫‪https://t.me/SAEED5ALAKLABI‬‬

‫‪ -2‬قناة التمريض واالمتياز‬


‫‪https://t.me/Nursing_training‬‬

‫‪ -3‬قناة االنشطة ( ‪)Zahra activities‬‬


‫‪https://t.me/Zahraactivities‬‬

‫وهي عبارة عن أسئلة نشاط من ملف غيث و ملف )‪(EMS‬‬


‫لألسئلة المهمة و تم ترتيبها حسب المواضيع في هذا الملف و حذف المكرر منها و إضافة المزيد من‬
‫األسئلة من ملف غيث و ملف )‪ (EMS‬كذلك تم إضافة جميع األسئلة من ملف‬
‫‪Cleft lip and Palate, Period Isolation‬‬
‫تنبيه ‪..‬‬

‫جميع الملفات المعتمدة موجودة على قنواتنا ‪.‬‬

‫ال نحلل و ال نبيح من يستغل هذا العمل و يقوم بسرقته و يتاجر فيه لمصالح مادية‬

‫وغيرها ‪..‬‬

‫و في الختام ‪..‬‬

‫هذا العمل خالصا لوجه هللا ال نريد منكم جزاء وال شكورا‪ ..‬نسأل هللا لنا ولكم التوفيق و‬

‫النجاح دائما و أبدا ‪.‬‬

‫‪1‬‬
1- A 16 year-old girl developed an infection over the surface of the heart
after having her nose pierced to place jewelry On admission, she had
developed pyrexia of 38C, a heart murmur and petechiae over the whole
body. She was admitted to the hospital and treated with intravenous
antibiotics. The nurse explains to the patient that she is at high-risk for
re-infection and provides discharge teaching on preventive measures.
Which of the following would most likely require prophylactic treatment?
A. Pelvic examination ‫سؤال أم زمام‬
B. Dental. Care ✓
C. Bronchoscopy
D. Urinary catheterization

2- A 20 year-old woman is hospitalized with a strong and uncontrollable


cough and has difficulty breathing while coughing. A doctor writes an
order for the patient to be transported from the medical surgical
department to the radiology department for an Xray examination. The
nurse prepares to transfer the patient and considers standard precautions
and additional transmission precautions. Who of the following would be
required to wear a mask?
A. Radiology staff
B. Nurse and patient ✓
C. Patient
D. Nurse

3- A nurse is caring for an elderly bed ridden patient who is dependent on


staff most activities of daily living. There is an order for standard
precautions in the patient's chart. The patient's care plan includes
feeding, washing hair with dry shampoo, brushing teeth, and nail care.
Which of the activities would require precaution?
A. Brushing teeth ✓
B. Feeding
C. Hair washing
D. Nail car

4- Diaphragmatic hernia diagnostic test?


A. Radiology ✓
B. Ultrasound
C. MRI D. CT scan
2
5- A 10-year-old girl presents to the Emergency Room (ER) with pain. On
assessment, you noticed that when you palpate the left lower quadrant of
the child’s abdomen, the child feels pain in the right quadrant. Which of
the following is the name of this signs?
A. Rebound tenderness
B. McBurney sign
C. Roving’s sign ✓
D. Obdurat signs

6- A 10-year-old girl presents to the Emergency Room (ER) with pain. On


assessment, you noticed that when you palpate the right lower quadrant
of the child’s abdomen, the child feels pain in the right quadrant. Which
of the following is the name of this sign?
A. Rebound tenderness ✓
B. McBurney sign
C. Roving’s sign
D. Obdurate sign

7- A nurse called the doctor while he is at home when she asked him for
an order for patient's medication as a nurse how many hours must the
doctor took to write the order in the hospital documentation?
A. 6 hours
B. 24 hour ✓
C. 48 hours
D. 12 hours

8- A patient treated an insect bite with traditional alternative therapy of


row garlic juice caused strong skin reaction. The patient agreed not use it
again after the nurse explained its harmful effects. Which of the following
is the best expression of the nurse's role?
A. patient's advocate
B. Resource person
C. Care planning
D. Caregiver ✓

9- Leaving the tourniquet for long time can cause?


A. Low blood flow ✓
B. Limb embolism
3
10- Leaving the tourniquet for long time can cause?
A. Hemoconcentration ✓
B. Low blood flow
C. Limb embolism

11- Following a lumbar puncture, a patient has several complaints.


Which of the following complaints indicate that patient is experiencing a
complaints?
A. I have a headache that gets worse when I sit up ✓
B. I am having pain in my lower back when I move my legs
C. My throat hurts when I swallow
D. I feel sick to my stomach and I'm going throw up

12- The nurse is performing a physical examination to the client with


hearing difficulty. The nurse activated the tuning fork and placed it on
top of the client’s head. What test did the nurse perform?
A. Whisper test
B. Rinne test
C. Audiometer
D. Weber test ✓

13- 62 year old man present to the clinic with complains of hearing loss in
the left ear the hearing loss has been progressive over the past year. A
semi-transparent, a pale grey color and clear of cerumen. A weber test is
performed and the sound lateralized to the right ear. Which type of
hearing loss is suspected?
A. Left-sided conductive
B. Right-sided conductive
C. Left-sided sensorineural ✓
D. Right-sided sensorineural

14- What is the name of procedure? ( see


image )
A. Myringoplasty
B. Tympanoplasty ✓
C. Tympanotomy

4
15- Otitis media surgery is?
A. Myringotomy ✓
B. Tympanoplasty
C. Sitting

16- During skin assessment of a patient the nurse observed a skin


described as round solid mass beneath the skin (see image). Which of the
following terms is best to describe the lesion?
A. Cyst ✓
B. Papule
C. Pastul
D. Blister

17- A nurse measures blood pressure of a patient admitted to the male


Medical Ward for 3 days. Which of the following is the best method to
ensure that the nurse's reading is reliable and accurate?
A. Check patient's blood pressure every shift ✓
B. Measure patient's temperature
C. Document result in chart
D. Measure patient's heart rate

18- A 65 year-old woman who was diagnosed with hyperthyroidism at the


age of 45 brought to the clinic by ambulance. On arrival, she had a
severely decreased level of consciousness. Her breathing is shallow and
irregular. Th skin is cool, dry and pale. There is generalized non- pitting
edema of all extremities and face. Blood pressure 70/40 mmhg Heart rate
60/min Respiratory rate 12/ min Body temperature 35.5 C tympanic
What is the most appropriate method to rewarm this patient?
A. Warmed intravenous isotonic fluids
B. Place the patient is a warm bath
C. Apply heat packs to the head and neck
D. Cover the patient with hospital blankets ✓

5
19- Adult patient complains of diarrhea, vomiting, abdomen cramp and
pain within the past 2 weeks. The patient reported that the pain increases
when he eats and relieves when he passes stool. Which of the following
may be the cause?
A. Appendicitis
B. Crohns disease ✓
C. Ectopic pregnant
D. Cholecystitis

20- A 45-year-old of man was admitted to the Surgical Ward after


removed of pancreatic cyst. The registered nurse checked the post-
operative orders Ondansetron (Zofran) PRN was prescribed for the
patient. Which of the following complaints from the patient will require
the registered nurse to serve the Ondansetron after 12 hours of pos -
operative period?
A. Wound pain
B. Nausea and vomiting ✓
C. Congestion of flame
D. difficulty in passing urine.

21- A patient comes to the Emergency Department after falling out The
nurse notices that the patient's left leg is externally rotate shorter than
the right leg The patient cannot move the left complains of severe pain.
What is the most likely cause of these symptoms?
A. A Dislocated patella
B. Fractured left hip ✓
C. Fractured left tibia
D. Dislocated left fibula

22- A patient is admitted to the emergency department after sustaining


abdominal injuries and a broken femur from a motor vehicle accident.
The patient is pale, diaphoretic, and is not talking coherently Vital signs
upon admission are temperature 98.0 F (36.3 C), heart rate 130
beats/minute, respiratory rate 34 breaths /minute, blood pressure
50/40mmHg The healthcare provider suspects which type of shock?
A. Distributive
B. Neurogenic
C. Cardiogenic D. Hypovolemic ✓
6
23- The nurse was caring for patient wound staples after operation. While
she was preparing the patient sneezed and one staple fell down the floor.
What should the nurse do?
A. Bring the staple from floor
B. Bring it and put back on wound
C. Apply dressing to the wound ✓

24- A 38-year- old patient is about to have lumber disk surgery. during
preoperative care, the nurse instructs the patient including the family.
members how to do "log rolling" to change patient position. One of the
family members ask why they must do such action in turning the patient.
postoperative. Which of the following is the nurse best response?
A. Facility good circulation
B. Avoid spinal movement ✓
C. Prevent post-operative bed sore.
D. Makes changing of patient position easier.

25- A 38-year- old patient is about to have lumber disk surgery during
preoperative care, the nurse instructs the patient including the family
members how to do log rolling to change patient position One of the
family members ask why they must do such action in turning the patient
postoperative Which of the following is the nurse best response?
A. Facility good circulation
B. Avoid movement
C. Prevent post-operative bedsore ✓
D. Makes changing of patient position easier

26- Patient will start TPN and should started gradually to avoid which?
A. Hyperinsulinemia
B. Hypoinsulinemia
C. Hypoglycemia
D. Hyperglycemia ✓

27- What is the complication of disc?


A. Continue muscle spasm
B. Impaired motor response ✓
C. Cerebrovascular accident

7
28- A 25 year old patient is admitted for observation in the surgical unit
with ad concussion after being involved in a road traffic accident. A nurse
performs a routine neurological assessment and notes a decrease in the
patient’s level of consciousness. BP 170/110 HR 52 RR 11 TEM 36.1 SAT
93% . Which of the following is the most likely cause?
A. Hypovolemia
B. Hypothermia
C. Pulmonary embolism
D. Increased intracranial pressure ✓

29-The doctor ordered for patient TPN How can we prevent infection
from central line if the nurse suspect that there is bacteria inside?
A. Give antibiotics 3 times per day
B. Apply dressing with alcohol on central line ✓
C. Apply one lumen for TPN
D. Apply dressing with antibiotics TPN daily

30- A Female patient will do the breast implant surgery and she tell the
nurse not to tell her family about the surgery. What is the nature of this
action?
A. Confidentiality ✓
B. Ethical dilemma
C. Illegal nursing practice
D. Medico-legal practice

31- The nurse is caring for a client who has had a right modified radical
mastectomy this morning. Which exercise should the nurse encourage the
client to perform this evening?
A. Hair combing exercises with the right arm.
B. Wall climbing exercises with the right arm.
C. Movement of the fingers and wrists of the right arm ✓

32- Radon can cause any cancer?


A. skin ✓
B. cervical
C. colon
D. prostatic

8
33- Radon cancer mostly affected?
A. Lung ✓
B. Cervical
C. Prostate

34- Radon cancer mostly affected?


A. pancreas ✓
B. Cervical Note: Causes lung or pancreas cancer also
C. Prostate

35- A 5-year - old child was brought to the Emergency Room with a
fractured of forearm. He had several bruises on his body but showed no
signs of pain while palpating them. He seemed scared and did not answer
any questions asked. How should the nurse initiate therapeutic
communication with the child?
A. Start interviewing
B. Encourage him to speak.
C. Explain about the fracture
D. Greet and show gentleness - nurse gives health.✓

36- You are assisting a MD with the removal of a chest tube. What
activity may the MD have the patient perform while the chest tube is
being removed?
A. Valsalva maneuver. ✓
B. Leopold Maneuver
C. Chest physiotherapy
D. Huff Cough Technique

37- A 76 years-old man is admitted to the Post-anesthesia Recovery unit


following chest surgery. The patient has a right chest tube that is attached
to low suction. Three hours after admission to the unit, the nurse observes
the drainage output from the chest tube is 300 milliliters. What is the
most appropriate initial intervention?
A. Notify the doctor ✓
B. Reduce IV infusion rate
C. Strip tube with roller device
D. Re-position in left lateral decubitus

9
38- Patient developed ascites which procedure will help relieve this
condition?
A. Paracenthesis ✓
B. Hemodialysis
C. Lumber puncture

39- 8 years -old child was admitted with suspected appendicitis a nurse
was evaluating the child’s condition and the mother stated that the child
did have his bowel movement for the past two days and requested for
natives. What is the risk of giving laxative to patient with appendicitis?
A. Pain
B. Fever
C. Rupture ✓
D. Diarrhea

40- A woman patient is admitted for abdominal pain. She complain


generalized plan nausea vomiting and constipation rebound tenderness
and abdominal rigidity the past hour her localized on right side BP
130/68 HR 92 RR 18 TEM 38.6 What should the nurse suspect?
A. Intestinal obstruction
B. Influenza
C. Appendicitis ✓
D. Pyloric stenosis

41- Which of the following nursing intervention assists in the prevention


of pressure ulcers following abdominal surgeries?
A. Frequent surgery site every two hours
B. Frequent moisturizing of skin
C. Use side rails as restrains
D. Encourage ambulation ✓

42- A 36 years old man ha undergone a subtotal thyroidectomy ago. he Is


suspected to develop tetany after the surgery. Which of the following
symptoms best indicates tetanus?
A. Tingling in the fingers ✓
B. Pain in hands and feet
C. Tension on the suture lines
D. Bleeding on the back of the dressing
10
43- What is the Location of rheumatic arthritis?
A. Wrist ✓
B. Elbow
C. Knee
D. Shoulder

44- A 28 year man admitted to orthopedic ward complaining of throbbing


pain in casted leg. Which of the following nursing intervention should be
taken first?
A. Remove the cast
B. Notify a doctor
C. Assess pedal pulse ✓
D. Administer PRN medication

45- During physical assessment of a male infant genitalia, the nurse found
that one of the testes are enlarged. Which of the following could be the
reason for swollen testes?
A. Chordee
B. Cryptorchidism
C. Hydrocele ✓
D. Hypospadias

46- A 55-year-old male has a myocardial infarction requiring admission


to the hospital. He is being treated with an anticoagulant. The healthcare
provider would be most concerned about Which of the following?
A. Trouble sleeping
B. Nausea
C. An elevated creatinine
D. Purpura ✓

47- The float nurse is transferring from surgical word to neurological


ward which of the following cases should delegate for her?
A. Patient with myasthenia gravis
B. Patient stable with fracture hip and casted ✓
C. Patient with chronic disease and on oxygen

11
48- A man is to be discharged from the General appendectomy the
precautionary measures, plans are discussed with him what the most
important desired outcome is after discharge?
A. Remain free of post-surgical complications. ✓
B. Report fever, redness or drainage from the wound site
C. Use pain management techniques apropos.
D. Resume gradual activities and avoid weight.

49- Which of the following considered a complex nursing assessment?


A. Laps time.
B. Admission.
C. Emergency ✓

50- There is crisis in hospital. Which of the following category of patients


should be evacuated first during fire?
A. Infant
B. Ambulate ✓
C. Elderly
D. Critically ill

51- Patient came to ER with dyspnea, Heart rate 136, Respiration rate 29.
After doing x-ray the patient diagnosed lung abscess. Which Unit should
the nurse have admitted the patient?
A. ICU
B. Medical, Surgical ward ✓
C. Cardiac ward

52- Patient have sore throat, lesions in mouth that didn't cure, dysphagia,
loss of weight. What's the expected diagnosis?
A. Gastritis
B. Tonsillitis
C. Laryngeal cancer ✓
D. Emphysema

12
53- A 21-year-old in oversized clothing presents to the hospital with of
felling dizzy and faint the hair and nail appear thin and dry the skin
appears pale, and she has sunk eye sockets and tenting skin. Her body
mass index is 16.She often induces vomiting after eating blood is collected
for analysis (see lab results). Blood pressure 90/52 mmHg, Heart rate118
/min Respiratory rate 26/min Temperature37.2C, Oxygen saturation
97%. ABG Test Result Normal value : - HCO₃ 31 22 – 88mmol/L -
PCO₂ 10.3 4.7-6.0 kPa - PH 750 7.36-7.45 . Which nursing problems stem
is the most appropriate?
A. Impaired nutrition
B. Decreased cardiac output. ✓
C. Infective airway clearance
D. Ineffective breathing pattern

54- The nurse care for a 60 year old woman who history hypertension,
hypothyroidism and elevated cholesterol levels. She takes tablets daily for
each of the health problem. The doctor orders a routine dual- x-ray
absorptiometry test that shows decrease bone density. Which medication
most likely contributed the test result?
A. Statins
B. Anti-hypertensive
C. Synthetic thyroid hormones ✓
D. Cholesterol absorption inhibitors

55- Client chest tube is connected to a chest tube drainage system with a
water seal. The nurse noted that the water seal c is fluctuating with each
breath that client takes. The fluctuation means that?
A. There is an obstruction in the chest tube
B. The client is developing emphysema
C. The chest tube system is functioning properly ✓
D. There is leak in the chest tube system

56- Which of the following tests is important assessment before


operation?
A. HIV screening
B. Blood grouping and cross matching ✓

13
57- The nurse is caring for a client who has had a chest tube inserted and
connected to water seal drainage. The nurse determines the drainage
system is functioning correctly when which of the following is observed?
A. Continuous bubbling in the water seal chamber
B. Fluctuation in the water seal chamber ✓
C. Suction tubing attached to a wall unit
D. Vesicular breath sounds throughout the lung fields

58- The nurse is caring for a client who has just had a chest tube attached
to a water seal drainage system. To ensure that the system is functioning
effectively the nurse should?
A. Observe for intermittent bubbling in the water seal chamber ✓
B. Flush the chest tubes with 30-60 ml of NSS every 4-6 hours
C. Maintain the client in an extreme lateral position
D. Strip the chest tubes in the direction of the client

59- The patient come to ER with head injury after one hour the expected
Vital/Signs for patient is?
A. Bp 126/85 RR 12 HR 78
B. Bp 158/92 RR 9 HR 60 ✓

60- Patient take anticoagulant daily what is the common risk factor
should the nurse concern?
A. Infection
B. Bleeding ✓
C. Pain

61- The doctor ordered to give nitroglycerine sublingual to a patient with


angina. What is the most likely effect that drug has on the patient body?
A. Increase heart rate
B. Increase blood pressure
C. Increase blood supply to the heart ✓
D. Increase myocardial oxygen consumption

14
62- Which is consider as normal blood pressure?

A. 90/60
B. 111/72 ✓
C.136/82

63- Patient with BP:145/95 . What


stage of hypertension?
A. Stage 1
B. Stage 2 ✓
C. Elevated
D. Crisis

64- Patient with BP: 230/180. What stage of hypertension?


A. Stage 1
B. Stage 2
C. Elevated
D. Hypertensive Crisis ✓

65- After subtotal thyroidectomy for patient. He complains restlessness


the nurse is assessing his vital signs: BP is 150/100 , Heart rate is120,
Temperature 40 .The nurse should expect that the cause for his condition
is?
A. Hyper-metabolism due to increase in T3&T4 ✓
B. Hypo metabolism due to decrease T3&T4
C. Hypo metabolism due to increase in T3&T4
D. Hyper metabolism due to decrease in T3&T4.

66- The nurse is assessing 50-year-old woman whose thyroid enlarged. A


blood sample was collected and an analysis confirm diagnosis. TSH 0.12
normal 0.4-6.5 Free 210 normal 50-140. Thyroxin T4 normal 4.5-11?
A. Thyroidectomy ✓
B. Incision and drainage
C. Polythyroidectomy
D. Adrenalectomy

15
67- Patient came to ER with high fever she had hyperthyroidism since 10
years ago and she has now symptoms of thyroid storm. What should the
nurse observe?
A. Bulging eyes.
B. Increased sensitivity to heat ✓

68- A 58 years- old woman present to clinic with tiredness and weakness.
She is tired throughout the day and usually sleeps for in the mid
afternoon. The last bowel movement was four days has a prescription for
levothyroxine but says she doesn’t remember to take it as prescribed. The
doctor arrange for specimen to be sent to the Lab. Which laboratory test
result are most likely?
A. Increase T4 and decrease TSH
B. Increase T4 and TSH unchanged.
C. Decrease T4 and TSH increase ✓
D. Unchanged T4 and TSH decrease.

69- A 48 years- old patient in the male Surgical Ward had his gall
bladder removed through laparoscopic cholecystectomy 24 hours ago.
While evaluating his general condition, the patient appears lethargic and
complains of severe nauseated feeling along with discomfort in the
abdomen. What nursing problem needs to be prioritized ?
A. Disturbed metabolism due to higher energy demand
B. Weak and lethargic due to low food and fluid intake
C. Nausea and vomiting due to slower gut movement ✓
D. Impaired comfort related to postsurgical effects

70- A Patient has a dissection aortic aneurysm. The patient’s surgery


would be categorized as?
A. Elective
B. Urgent
C. Emergency ✓

71- A patient is scheduled for an abdominal aneurysm repair. This is


what type of surgical intervention?
A. Diagnostic
B. Transplant
C. Curative ✓ D. Palliative
16
72- A cardiac care unit nurse decides to gather relevant data about a
myocardial infraction patient who has breathing difficulty and sever
bradycardia. Which of the following would be the most relevant and
suitable database?
A. Focused
B. Emergency ✓
C. Complete
D. Follow up

73- A 42 years-old patient went to the clinic for an eye consult. Patient
complained of blurred vision, ocular pain and head active. During
assessment tonometry was done (see results). Test Result Normal Values
Intraocular pressure 34 10-20mmHg. Which surgical procedure is the
most appropriate?
A. Laminectomy
B. Laser trabeculoplasty ✓
C. Incision and drainage
D. Extra capsular cataract extraction

74- What does time out mean?


A. Anesthesia is ready
B. All equipments are ready
C. Time for patient identity, the surgical procedure and the site
to start✓
D. Time for patient to go back to ward

75- A nurse is assigned to care for a Muslim female emergency caesarean


section. The patient asks the to wear a head scarf during the surgery.
Which of the following is the most appropriate?
A. Ask the patient to remove the scarf
B. Tell the patient that it is ok to wear the scarf
C. Ask the patient to wear a surgical cap ✓
D. Tell the patient that a request will be during the surger

17
76- The nurse begins a morning shift with the following see the doctor in
the waiting room (see table)?
1.A first time mother who delivered two days ago and is having
Difficulties breast feeding .
2. A60 year-old man who has a history of deep vein thrombosis and is
taking daily heparin .
3. A76 year-old women who was hospitalized three days Previously with
pneumonia .
4. A 56-year-old man who has used all his diuretic pills Coughing up
sputum .
Which patient should be seen first?
A. One B. Two
C. Three
D. Four ✓

77- Which of the following electrolyte cause cardiac arrest?


A. Chloride
B. Potassium ✓
C. Sodium
D. Calcium

78- A 46-year-old man diabetic patient is admitted to the surgical


department because he has gangrened leg due to uncontrolled blood
sugar and contamination of leg ulcer. The admission plan is to do under
knee amputation of the left leg. The nurse is explaining to the patient that
this will cause a permanent disability. Which of the following statements
is the best description for permanent disability?
A. Problems in body function and loss
B. Difficulties to experience life situations
C. Difficulties in executing activities ✓
D. Loss of interaction with community

79- During the evaluation at a community clinic, the patient completes the
medical history. Which of the following is NOT a risk factor for an acute
myocardial infarction?
A. Coronary artery disease
B. Smoking
C. Hemophilia ✓ D. Hyperlipidemia
18
80- After the pericardiocentesis, the doctor inserted a polyethylene
catheter left it in the pericardial sac. The patient asked the nurse about
the response of the catheter. What should be the nurse's response?
A. Monitor consistency of drainage ✓
B. Prevent movement of pericardial sac
C. Prevent recurrence of cardiac tamponed
D. Prevent increase in venous and blood pressure

81- A 71-year-old woman who resides in a long-term nursing home fell


while walking down stairs. The attending nurse arrives to find the patient
sitting motionless on the stairs. She is alert and oriented but wishes to rest.
While she rests, the nurse reviews the chart and notes that her medication
regimen includes metformin, loratadine, warfarin and diclofenac. Which
of the following sign most concern and should the nurse be alert?
A. Bleeding
B. Bone fracture
C. Brain concussion ✓
D. Hypoglycemia

82- A 62-years old patient present to the emergency department and


complains of muscle cramps nausea vomiting and fatigue. The outside
temp 45 and had been working outside day all day in the sun. he says that
his fingers are numb and cannot feel anything with them. BP 106/58 HR
99 RR 22 TEM 37.4 which of the following electrolyte will be below the
normal range?
A. Chromium
B. Magnesium
C. Potassium ✓
D. Bicarbonate

83- A patient has undergone pericardiocentesis as part of the


management of cardiac tamponed Which of the following would be most
indicative of cardiac tamponade recurring?
A. Facial flushing
B. Declining heart sounds
C. Muffled heart sounds ✓
D. Increasing blood pressure

19
84- At 6:00 PM while admitting a woman for a scheduled repeat C
section1, a patient tells a nurse that she drank a cup of coffee at 4:00AM
because she wanted to avoid getting a headache. Which of the following
actions should the take first?
A. Inform anesthesia care provider ✓
B. Ensure preoperative lab results are available
C. Start prescribed IV with lactated ringers
D. Contact patient's obstetrician

85- A patient has been experiencing recurrence of this media antibiotic


therapy and the patient was scheduled for preparative teaching, the
patient asked the nurse about the purpose of the procedure. What is the
best answer that the nurse should provide?
A. Stimulates motion of the ossicles
B. Detects permanent hearing loss
C. Allows drainage of purulent fluid from the middle ear ✓
D. Enables medication administration directly to the affected ear

86- The nurse has started intravenous fluid therapy on a child. Which of
the following action is appropriate?
A. Using a padded arm board if the child is active to restrict movement.
B. Apply lotion twice daily to prevent readiness and irritation
C. Determining the total volume infused every four hours.
D. Using an infusion pump to control the rate of infusion ✓

87- Height of enema is?


A. 50 cm ✓
B. 150 cm
C. Blow red edge

88- What is the best nutrition teaching instruction for patient with
Parkinson disease?
A. Choking
B. Drooling ✓
C. Aspirations
D. Dysphasia

20
89- A 44 years- old obese patient was subject for a surgery called
bariatric gastric bypass to specific. The nurse and the physician are
giving information about the procedure to the patient. Which of the
following statement is the best describing the patient understanding?
A. Same effect with liposuction
B. An opening will placed in my medication
C. The surgery will reduce the size of my stomach ✓
D. Easiest way to lose weight no need to exercise after surgery

90- A 43 year old woman accompanied by her son to the primary health
clinic and she speaks different language of the nurse. The nurse in the
clinic in the clinic does not speak the same language of the patient. In this
situation, what should the nurse do to assess the needs of this patient?
A. Ask the patient if it is OK to let her son translate for her
B. Try to use an Urdu translation application in her phone
C. Find out an interpreter employed the clinic ✓
D. Try to make flash with common phrases used

91- A 82 years old patient has Parkinson’s disease. During the assessment,
the nurse would expect which of the following actions to produce the most
tremor activity of the hands?
A. Eating with a fork
B. Resting hands in lab ✓
C. Standing with hands loose at sides
D. Rolling a small pill between the fingers

92- Patient with 88% O2 saturation What should use?


A. Facial mask ✓
A. Nasal cannula

93- Patient with COPD came to ER. The patient was very exhausted. He
was complaining shortness of breathing and cough. What is the most
appropriate PaO2 percentage should be given for him?
A. 55 or below mmhg ✓
B. 80 or high
C. 70 mmhg

21
94- Patient is being weaned off from the mechanical ventilator is about to
hook the endotracheal tube to oxygen at FiO2 of 40 Which of the
following oxygen administration device is the best the nurse in this
situation?
A. Ambo bag
B. Ventura mask
C. Tracheostomy collar
D. T-piece/ Briggs ✓

95- A nurse is assigned to care for a patient on oxygen. The nurse checks
the oxygen concentration using a pulse oximeter. Which of the following
reading indicates that the patient has adequate oxygenation?
A. Less than 90%
B. 91%-92%
C. 93%-94%
D. 95% and above ✓

96- What is the oxygen partial pressure?


A. Fio2
B. Pao2 ✓
C. So2

97- Ineffective airway clearance related to COPD as evidence persistent


cough. The problems refer to which of the following?
A. Airway clearance ✓
B. COPD
C. Persistent cough

98- Patient has Colostomy we transfer the Patient?


A-Belt abdomen
B- Belt chest ✓
C- Restrain

22
99- A nurse is caring for a 72-year-old man patient, is unsteady. The
patient requests the nurse to help nurse asked the patient to wait few
minutes device to transfer him. Which of the following transfer devices is
the nurse to use?
A. Belt ✓
B. Board
C. Handle
D. Mechanical lift

100- The nurse is preparing to transfer a 55-year-old patient on a the CT-


scan unit. The patient is too heavy for the nurse to transport, nurse went
to get an assistive device to transfer the patient. Which of the following
transfer device is the most appropriate for the nurse to use?
A. Board
B. Handle Note:
Too heavy or morbidity obese = mechanical lift
C. Trapeze Heavy or obese = board
D. Mechanical lift ✓ Unsteady unconscious = belt

101- What is most common symptoms of COPD?


A. Air hunger
B. Dyspnea
C. Cough with sweat night
D. Persistent cough ✓

102- A 78 years-old alert and oriented patient is returning from the


radiology department and the nurse is preparing to transfer the patient
from the wheelchair back into the bed. The nurse places the gait belt on
the patient and prepares to lift the patient from the chair. Which body
mechanics would be most appropriate?
A. Widen leg stance ✓
B. Bend over to left
C. Rotate from the waist
D. Maintain base of gravity in the feet

103- A patient teeth brush & bathing should be write in?


A. Daily care activity ✓
B. Medication chart

23
104- Patient is receiving chemotherapy. When should the nurse give
antiemetic to avoid side effect?
A. Before session ✓
B. During session
C. Half hour After session

105- A conscious victim of a motor vehicle accident arrives at the


emergency department. The patient is Gasping for air, is extremely
anxious, and has a deviated trachea. What diagnosis should the nurse
Anticipate?
A. Pleural effusion.
B. Tension pneumothorax ✓
C. Pneumothorax
D. Cardiac tamponed

106- A 16 years-old man is in the Emergency Department with S Ose


bleeding as a result of falling down while climbing up the wall. On
assessment, his nose appears slightly deviated and 2 swollen. He is
breathing from his mouth What sign helps evaluate that his nose is still
bleeding?
A. Complain of sharp pain in the nasal bone
B. Experience more difficulty in breathing
C. patient is swallowing frequently ✓
D. Increased swelling of the nose

107- What is the priority nursing assessment before administering


methergine for management of postpartum hemorrhage?
A. Blood pressure ✓
B. Uterine atony
C. Amount of lochia
D. Deep tendon reflex

108- The patient has Huntington's disease. Which of following is the most
expected symptoms of disease?
A. Weight gain
B. Sever Headache
C. CNS depression ✓

24
109- Patient came to ER after traffic accident and have sever bleeding
The doctor order she must enter operating room immediately what is
appropriate actions to do to obtain consent?
A. Call his husband to obtain telephone consent from him
B. Obtain order from nurse supervisor to enter operation room and do surgery
C. Enter pateint to operation immediately according to dr order without
obtaining consent ✓
D. Obtain consent from her friend who come with her to hospital

110- The patient has skull fracture. The patient is complaining racoon eye,
pain,and posterior neck fracture what is the type of skull fracture?
A. Basilar ✓
B. Depressed
C. Compound
D. Linear

111- A 24 years old patient brought to emergency room after an accident


and the nurse noticed a leaking of clear fluid from the nose. A nurse
anticipated the injury is?
A. Basilar ✓
B. Frontal lob
C. Neuropathic
D. Neuropathic

112- Patient came to ER after motor accident. The patient has sever
bleeding. Post operative head Surgery the vital signs are BP 90/60, HR
126 b/m, respiration 24 b/m). The doctor ask the nurse to assess good
tissue perfusion. Which of What is Indication of Improve tissue
perfusion?
A. decrease HR to 100
B. increase pulmonary wedge pressure
C. Increase Systolic BP to 86
D. 100 ml dark urine ✓

25
113- A nurse is caring for a child who sustained a head injury after
falling from a tree. On assessment of the child, the nurse notes the
presence of a watery discharge from the child's nose. The nurse will
immediately test the discharge for the presence of which of the following
substance?
A. Glucose ✓
B. Protein
C. White blood cells
D. Neutrophils

114- How to know it the left shoulder dislocated?


A. left arm longer than right arm ✓
B. patient cannot move the left shoulder
C. swelling in the left shoulder

115- What is the authority that issues licenses for workers in the health
sector in Saudi Arabia?
A. Saudi commission for health specialties (SCFHS) ✓
B. Saudi hospitals

116- Nursing staff If they take Saudi commission for health specialties.
Which is the Ministry is responsible for that?
A. Ministry of Health ✓
B. Saudi nursing Authority

117- Saudi Arabia neonatal screening for?


A. Hypothyroidism diseases ✓
B. Congenital diseases
C. Immunodeficiency disease

118- Multicultural team work on Saudi Arabia what's important thing


for the all team member to have?
A. Have the same value and belief in Saudi
B. Have the same professional guidelines
C. Have the same goal and objectives toward work ✓
D. Have the same qualification

26
119- A 1 year-old girl admitted to pediatric medical unit significant
weight loss ,diminished mid -arm circumference diarrhea, muscle like
stick .which of the following type of malnutrition do the nurse suspect?
A. Marasmus ✓
B. Spitting up
C. Kwashiorkor

120- A 1-year-old girl admitted to podiatric medical unit significant


weight loss, diminished mid-arm circumference diarrhea, and red hair.
Which of the following type of malnutrition do the nurse suspect?
A. Marasmus
B. Spitting up Note:
Kwashiorkor = Red hair
C. Kwashiorkor ✓ Marasmus = Muscle stick
D. Rickets

121- Nurse makes a clinical judgment that an African American male


patient in a stressful job is more vulnerable to developing hypertension
than White male patients in the same or similar situation. The nurse has
formulated what type of nursing diagnosis?
A. Actual
B. Risk ✓
C. Possible
D. Wellness

27
122-The patient has Renaud's syndrome on his toes. The patient has
yellow ulceration changed from blue to pale. What is the most common
cause?
A. Ascites
B. Palpitation
C. Digital sensitivity to cold ✓

123- Patient in triage area. What is the first subjective data should triage
nurse Obtain?
A. Chief compliance ✓
B. Level of pain
C. Family history

124-The nurse is teaching a patient about spironolactone (Aldactone).


Which of the following instructions should review with the patient?
A- Increasing the intake of foods that are high in potassium
B- Taking the medication right before going to sleep
C- Avoid seasonings that are labeled as salt substitutes ✓
D- Scheduling the medication so that a multivitamin is taken an hour later

125- Patient has a defect with the beta cells of the Islet of Langerhans.
Which of the following should the nurse most likely expect this patient to
exhibit?
A. Anemia
B. Appendicitis
C. Cholelithiasis
D. Hyperglycemia ✓

126- A 65-year-old women visited the gynecological outpatient history


reveals that she had 3 pregnancies, one abortion gestational age, had 2
normal deliveries. She smokes 20 Her complaint is that she wets herself
when she cough embarrassing for her? Which of the following can be
considered as risk factors pelvic floor muscles?
A. Chronic coughing
B. Diabetes mellitus
C. Excessive spot
D. Sedentary life style ✓

28
127- What is the intervention for open fracture?
A. Mobilize as much as possible
B. Left uncovered
C. Splinting leg ✓
D. Rise leg 90 degree

128- During the assessment of an adolescent's back, the nurse notice a


lateral deviation of the spine and an elevation of one shoulder blade than
the other. Which of the following is indicated by these findings?
A. Spinal fusion
B. Kyphosis
C. Lordosis
D. Scoliosis ✓

129- Which of the following is the best action to effectively increase


surgery capacity?
A. Adapt outpatient departments for inpatients ✓
B. Bring the injured people into the green zone
C. Provide care for the injured people with the available staff
D. Use the available supplies as it is difficult to arrange for additional

130- A newly admitted patient diagnosed with right-sided brain strok has
a nursing diagnosis of disturbed visual sensory perception related to
homonymous hemianopsia. Early in the care of the patient , what should
the nurse do?
A. Place objects on the right side within the patient’s field of vision. ✓
B. Approach the patient from the left side to encourage the patient to turn the
head.
C. Place objects on the patient’s left side to assess the patient’s ability to
compensate.
D. Patch the affected eye to encourage the patient to turn the head to scan the
environment.

131- A 12 years old child diagnosed with Diabetes mellitus. The doctor
ordered insulin for him. The nurse should teach how to take insulin.
Which of the following is the most appropriate nursing intervention?
A. Teach parent how to inject
B. Teach child how to inject himself ✓
29
132- A 15 year old girl diagnosed with DM and she tell the nurse she
could not give her self-injection. What is the nursing intervention?
A. Refer to home care
B. Teach her how to inject her self ✓
C. Instruct her to come to the hospital

133- A 4 years old diabetic patient went to party. He came back with
lethargic and tired . What is the First action?
A. Check blood glucose levels ✓
B. Administer juice

134- A 69 year-old man is admitted to the intensive care unit following


cardiac surgery. Two hours after admission, the nurse performs a routine
assessment and notes the patient's chest tube drainage is 200 milliliters
and a dark red color. He has had 60 milliliters output from the indwelling
urinary catheter Blood pressure 138/68 mmhg Heart rate 76/min
Respiratory rate 16/min Body temperature 37.0C oral Oxygen saturation
94% 6L/min nasal cannula . Which finding should be reported to the
doctor?
A. Dark red chest tube drainage
B. Urinary output
C. Oxygen saturation ✓
D. Chest tube output volume

135- A 57-year-old woman presents with complaints of pain in the bones


of her toes, foot and swelling on knees. What type of musculoskeletal
disorder is most likely?
A. Osteoarthritis.
B. Rheumatic arthritis
C. Osteomyelitis
D. Gout ✓

136- A 57 year-old woman presents with complaints of pain in the bones


of her both hands . What type of musculoskeletal disorder is most likely?
A. Osteoarthritis.
B. Rheumatic arthritis ✓
C. Osteomyelitis
D. Gout
30
137- A nurse is planning to assess the corneal reflex on unconscious client.
Which of the following is the safest? stimulus to touch the client’s cornea?
A. Cotton buds
B. Sterile glove
C. Sterile tongue depressor
D. Wisp of cotton ✓

138- The patient is unconscious and need emergency OR and the family is
2 hours before reach the hospital to whom to get the consent?
A. Call the family and get the consent
B. Do the procedure ✓
C. Let the supervisor do the consent
D. Nurse will sign the consent

139- During transferring for patient to operation room. The patient


changed his mind to do surgery. What should the nurse do?
A. Try to convince patient to do operation
B. Talk with relatives
C. Say that is not according to your mind
D. Inform operation staff and manager ✓

140- Classical signs of adrenal insufficiency?


A. Hyperpigmentation ✓
B. Hyponatremia
C. Hypernatremia

141- A 45-year-old man is admitted to the neurosurgery ward for the


surgical elevation of depressed skull fracture. He has episodic severe
headache with seizure and is unable to concentrate. Which of the
following initial nursing problem needs more attention?
A. Disturbed coping and anger spells
B. Risk of injury to seizure ✓
C. Disturbed communication and irritability
D. Pain management and comfort measure

142- Coombs test used to detect and screening for which of the following?
A. Sickle cell anemia ✓
B. Congenital anomaly C. Neural tube deficient
31
143- A 40 years old complains of severe pain, scheduled for surgery of
abdomen, medical history he smokes one packed of cigarettes per day.
Which of the following complications he might develop?
A. Atelectasis ✓
B. DVT
C. Constipation

144- A nurses is providing teaching session for pre-operative patient


before appendectomy about how to use spirometer, which of the following
nursing process?
A. Assessment
B. Evaluation
C. Implementation ✓
D. Diagnoses

145- A 45 year old patient has a colostomy bag attached as a result of


benign mass removal, while conducting the discharge teaching session,
the nurse assessed the patient's wife understanding of the teaching.
Which of the following steps of nursing process is exercised here?
A. Implementation
B. Planning
C. Assessment
D. Evaluation ✓

146- After teaching the deep breathing and coughing who is undergoing a
surgery, the nurse asked demonstration and then helped him in
correcting what part of therapeutic communication is used?
A. Evaluation ✓
B. Intervention
C. Identification
D. Demonstration

147- A nurse is providing session regarding lung cancer, which statement


indicates teaching was not effective?
A. I will eat fruit and vegetables
B. If I have COPD does not mean I will have lung cancer
C. No need to worry because I do not smoke, my husband does ✓

32
148- Patient with casted leg on discharge. Which of the following
instructions should the nurse include?
A. Rise your leg 90 degree
B. Remove cast after finishing treatment
C. Inform doctor for any abnormalities in circulation ✓

149- A patient is admitted to the hospital for a right total knee


replacement. during the preoperative assessment the patient reports no
known during allergies but the patient does report a severe allergic
reaction to shellfish. Why is this allergic reaction to shellfish of special
importance?
A. Shellfish allergies could indicate a negative reaction to anesthesia
B. This patient will not be able to receive donor packed red blood cells if
needed
C. A seafood allergy can be an indication of allergy to
provide one-iodine ✓
D. A seafood allergy can be precursor to malignant hyperthermia

150- During suction of child, ensure that each pass of the suction catheter
take no more than?
A. 5 Sec ✓
B. 10 Sec
C. 15 Sec

151- During suction of adult, ensure that each pass of the suction
catheter take no more than?
A-5 Sec
B-10 Sec ✓
C-15 Sec

152- The nurse is caring for a critically ill patient on mechanical


ventilation . The physician identifies the need for a bronchoscopy, which
requires informed consent. If the physician were to obtain consent from
the patient, the patient must be able to?
A. Be weaned from mechanical ventilation
B. Have knowledge and competence to decision ✓
C. Nod his head that it is okay to proceed
D. Read and write in English
33
153- Doctor order sputum culture how to collect?
A. Cough deeply ✓
B. Collect 30ml
C. Cough the first sputum in the throat
D. Cough in the morning and send it in the night

154- A 68-year-old diabetic women undergoes a below knee amputation


to vascular insufficiency and infection. On admission to care unit, the
nurse makes the priority diagnosis: Risk for ineffective peripheral tissue
perfusion. Which intervention is most appropriate?
A. Ensure adequate pain relief
B. Elevate residual limb on a pillow
C. Administer oxygen by facemask
D. Apply ice to the stump ✓

155- Post right-side mastectomy where to take blood pressure?


A. Right brachial
B. Left brachial. ✓
C. Right radial
D. Left radial.

156- Which of the following interventions is appropriate for the diagnosis


of risk for injury of patient that has a pacemaker?
A. Offer back rubs to promote relaxation
B. Instruct patient in dorsiflexion exercise of ankles
C. Have patient avoid exposure to magnetic resonance image ✓
D. Observe incision site for the redness purulent drainage warm ,soreness

157- The nurse is giving health teaching for patient with stage
pressure ulcer for discharge. What should the nurse tell?
A. Clean the wound with normal saline
B. Avoid using bedpan ✓

158- Patient with systemic Lepus arthritis complain pain in the joint She
has butterfly rash in her face what is most appropriate nursing action?
A. Maintain skin integrity
B. Manage pain discomfort relief ✓

34
159- Registered nurses must be able to measure blood pressure accurately.
What could possibly result in incorrect high blood pressure reading?
A. Bladder cuff is to wide
B. Cuff is deflated too quickly
C. Cuff is wrapped too loosely ✓
D. Arm is above the level of the heart

160- Nurse started shift and received two patient. One with thoracotomy
and the 2nd with pneumonia. Which of the following assess first?
A. Head to toe assessment for patient with pneumonia
B. Assess level of consciousness and vital signs for both patients ✓
C. Review for care plan and medication for both patients

161- A 38-year-old teacher who reported dizziness and shortness of


breath while supervising recess is admitted with a dysrhythmia. Which
medication if ordered requires the nurse to carefully monitor the patient
for asystole?
A. Atropine sulfate
B. Digoxin (Lanoxin)
C. Metoprolol (Lopressor)
D. Adenosine (Adenocard) ✓

162- The patient complained from stomachache continue after eating for
2 to 3 hours. The patient has increased weight 3 kg during short time.
Which of the following is the most important nursing diagnosis?
A. Imbalance nutrition
B. Acute pain ✓
C. Fluid volume deficit

163- How can the nurse give health education to prevent or reduce
sinusitis by health education?
A. Increase carbohydrate intake
B. Increase fluids intake
C. Avoid travel with air plane ✓

35
164- A nurse develops a plan of care for a child at risk for tonic-clonic
seizures. In the plan of care, the nurse identifies seizure precautions and
documents that which item (s) need to be placed at the child's bedside?
A. Emergency cart
B. Tracheotomy set
C. Padded tongue blade
D. Suctioning equipment and oxygen ✓

165- What should be your first concern at the scene where a person has
been seriously burned?
A. Checking the scene for safety. ✓
B. Checking the victims breathing and pulse.
C. Calling your local emergency phone number.
D. Cooling the burned area.

166- A nurse is completing the preoperative checklist for one of the


patients who ring is wearing a ring. What is the most appropriate action?
A. Give ring to security office
B. Lock ring with patient's valuables ✓
C. Call patient's family to give them the ring
D. Respect patient's choice and leave ring on patient's finger

167- Patient has difficulty in walking and lethargic, there decrease in


capillary refill and urine incontinence, has risk for?
A. Aspiration
B. Infection
C. Skin breakdown ✓
D. Body disturbance

168- A 17-year-old arrived at the emergency room complaining


abdominal pain on right lower quadrant. Pain was rated as 9 numeric
scale with positive rebound tenderness over the pain. Blood pressure
120/70 Heart rate 100 Respiratory rate 22 . Which of the following
interventions has the highest priority?
A. Keep NPO. ✓
B. Secure an IV access.
C. Prepares for ultrasound.
D. Prepares for abdominal surgery.
36
169- The nurse sustains a needle-stick injury after administration an
intramuscular to a patient. It is recommended that the nurse be
tested for human immune deficiency virus (HIV)?
A. Immediately with repeat testing 6 weeks. ✓
B. If patient refuses HIV testing.
C. If patient has a symptom of HIV infection.
D. A month after taking prophylactic antiviral.

170- MI patient in the cardiac care unit suffers from SOB and dyspnea.
the nurse gather data from relative. What type of database?
A. Focus database
B. Follow up database
C. Emergency database ✓

171- 74- A 56-year-old man with a history of COPD was rushed to the
Emergency Department with chest pain, shortness of breath, fever and a
cough. Upon assessment, crackles can be heard over the low lobes. The
patient looks pale and lethargic (see image). Blood pressure 110/70
mmHg Heart rate 130 /min Respiratory rate 9 /min Temperature 38.1C.
Oxygen saturation 85 % What is the most likely condition?
A. Bronchial asthma
B. Respiratory failure ✓
C. Pulmonary embolism
D. Myocardial infraction

172- Woman has a history of depression and chronic disease and asked
the nurse What is the best contraceptive?
A. Copper ✓
B. Combined
C. Hormonal

173- The patient is postoperative cardiac surgery. The pulse rate is


changed to bradycardia. What is the most common cause of decrease
Heart rate after operation?
A. Pain
B. Stimulate Vegas nerve ✓
C. Anxiety

37
174- A 19-year-old girl was scheduled for the extraction under general
anesthesia. Her pre-operative was done and the consent was signed by her,
but her nail polish and trim her long nails, requirement. What should be
the nurse's action in response to?
A. Explain why nails need to be cleaned ✓
B. Respect the patient's right to refuse
C. Record and inform physician
D. Remove as per protocol

175- A 28-year-old man and his wife involved in which his wife was killed.
The client is being Care Unit for multiple rib fractures and a broken
which room his wife is located. Which of the following is the most
appropriate?
A. "Your wife is not in the hospital"
B." I'm sorry, but your wife did not survive the
C. "I need to get your family so that you can wife”
D." The doctor will be talking to you about located” ✓

176- A patient presents to the clinic with 3+ edema of the lower


extremities, diagnosed neck veins, tachycardia, bounding pulse, weight
gain of 4 kilograms with 7 days, shortness of breath, and wheezing. Fluid
intake over the past 24 hours has been 3700ml output is estimated at
2400ml. which of the following is the most likely nursing diagnosis?
A. Fluid volume excess ✓
B. Urinary tract infection
C. Hypothyroidism
D. Ketoacidosis

177- A 25 year-old woman present to the Emergency Room with acute


gastritis and signs of moderate dehydration. Which is the most
appropriate nursing diagnosis?
A. Infection
B. Fluid volume deficit ✓
C. Activity intolerance
D. Fluid volume excess

38
178- A nurse is assigned to care for a group of patient in the medion
expected to review the medical records of these patient. What patient is at
risk for excess fluid volume?
A. Patient with ileostomy
B. Patient taking a loop diuretic
C. Patient with chronic renal failure ✓
D. Patient hooked to gastrointestinal

179- Patient with fluid volume deficit. What is the most likely symptoms
will be found by nurse?
A. Tachypnea
B. Tachycardia ✓
C. Nausea

180- A patient with congestive heart failure and severe peripheral edema
has a nursing diagnosis of fluid volume excess What are the two
MOST important interventions for the nurse to initiate?
A. Diuretic therapy and intake and output
B. Nutritional education and low-sodium diet
C. Daily weights and intake and output ✓
D. Low-sodium diet and elevate legs when in bed

UTI, Kidney
181- During the night shift routine rounds, a nurse found that a patient
complain of sleep disturbance due to frequent voiding several times at
night. Which of the following is the best condition that describes the
patient complain?
A. Dysuria
B. Polyuria
C. Nocturia ✓
D. Hematuria

182- Steal syndrome in hemodialysis symptoms?


A. Pain and hotness of skin
B. Pain and coldness of skin ✓

39
183- Renal failure patient came for dialysis, the nurse suspect that the
patient develop Steal Syndrome. What is the Steal Syndrome
characteristic?
A. Pain and warmth
B. Pain and coldness ✓
C. Redness and warmth
D. Redness and coldness

184- Child has urine incontinence two day ago with increase WBC in
urine. What is the cause?
A. Psychological abuse
B. Urinary tract infection ✓
C. Genitalia defect

185- A 60 years-old man is being discharged from the post-operative Care


Unit following a transurethral resection of the prostate. The nurse
provides discharge information regarding the care of the bladder
catheter. Which method would be most effective in bladder retraining for
this patient?
A. Scheduled urination every 2-3 hours ✓
B. Limit fluid intake before sleeping time
C. Perform pelvic floor exercises daily
D. Increase fluid intake during the daytime

186- Patient diagnosed with UTI. He has frequent urination . The urine
analysis is rescheduled to the next day. What is the best action for the
patient?
A. Increase fluid intake ✓
B. Decrease fluid intake
C. Normal fluid intake
D. Avoid fluid intak

187- Recurrent urinary tract infection what advice to give?


A. Drink 2-3 liters of water ✓
B. Wipe from back to front
C. Urinate only when you have the urge.

40
188- 10 years boy with polyuria and dysuria after assessment diagnosed
with urinary tract infections what should do to take urine sample?
A. Increase fluid intake ✓
B. Decrease urine intake
C. Regular intake of fluid
D. Zero intake of fluid

189- Patient with urine output less than 400 ml for 24h what is the
common diagnosis?
A. Dysuria
B. Polyuria
C. Oliguria ✓

190- A 36 year old man present to the hospital with complaints of


breathing difficulties and fever. He had undergone a liver transplant one
year before upon assessment a yellowish colored skin was noted on entire
body HR 102 RR 29 BP 110/80 TEM 39.2 HB 11 WBC ( white
blood cells) 4 RBC ( red blood cells) 3.5 Which complication is most
likely to occur?
A. Bleeding
B. Embolism
C. Superinfection
D. Organ rejection ✓

191- Patient has AV fistula in his leg. The patient came to dialysis unit
for session. When the nurse check the fistula, She found that fistula not
working. What is the first action?
A. Auscultate the fistula
B. Notify the physician ✓
C. Monitor vital signs

192- Patient with decrease Glomerular filtration rate GFR and small
kidney size 8cm, creatinine 8.5 mg/dl. What is the appropriate treatment?
A. Nephrotomy
B. Nephrectomy
C. Hemodialysis ✓

41
193- The patient with a kidney transplantation, then he came after few
days from discharge complaining from fever, tenderness and edema.
What should the nurse expect?
A. UTI
B. Organ rejection ✓

194- Which of the following is a sign of rejection after a renal


transplantation operation?
A. Decrease potassium
B. Decrease sodium
C. Decrease creatinine
D. Decrease urine out put ✓

195- Most common cause for acute renal failure?


A. pyelonephritis
B. Tubular destruction
C. Urinary tract obstruction
D. Dehydration ✓

196- The patient is receiving hemodialysis. The patient is experiencing


disequilibrium syndrome. What is the cause of the syndrome?
A. Increase urea and Na
B. Decrease urea and Na ✓

197- Disequilibrium syndrome post dialysis?


A. Increase urea and increase sodium
B. Decrease urea and decrease sodium ✓

198- A patient with chronic kidney disease receives hemodialysis week.


The nurse notices that the arteriovenous fistula is red (see lab results).
Test result normal value WBC 12.0 4-10.5x10^9/L Glucose 5.3 3.5-6.5
mmol/L HCT 0.38 0.41-0.50 Iron 10.7 11.7-31.8 mol/L Phosphorus 1.4
2.5 mg/Dl. Which of the following nursing diagnoses takes priority?
A. Activity intolerance related to generalized weakness se
B. Impaired skin integrity related to dry skin secondary hypophosphatemia
C. Constipation related to changes in iron level secondary hemodialysis
D. Risk for infection related to invasive ✓

42
199- Patient with chronic renal failure before performing peritoneal
dialysis the nurse should warm the solution before administering to?
A. Promote abdominal muscle relaxation
B. Maintain extra body warmth temperature
C. Encourage the removal of serum urea ✓
D. Stimulate potassium back into body cells

200- The dialysis solution is warmed before use in peritoneal dialysis


primarily to?
A. Encourage the removal of serum urea ✓
B. Force potassium back into the cells.
C. Add extra warmth to the body.
D. Promote abdominal muscle relaxation.

201- The patient with CRF should perform peritoneal dialysis at home .
The nurse is learning patient and instructing the client to warm the
dialyzing solution to 37 degrees Celsius. Why should the patient warm the
solution before using?
A. Dilate the peritoneal blood vessels ✓
B. Maintain a constant body temperature
C. Remove toxins from the body's cells
D. Relax the abdominal muscles

202- A 46 years-old patient is in the male Urology Ward after the surgical
removal of the stone from his left kidney through percutaneous
nephrolithotomy under general anesthesia. He has nurse and dull acting
pain in left lumbar region. His nephrostomy bag is attached through a
tube in his left kidney for a few days ( see image) What findings should
alert the nurse to? ‫السؤال له أربع اشكال مختلفة و نفس الصورة‬
A. Abdominal discomfort
B. Patient Very exhausted
C. Presence of blood and stone gravels in urine
D. Urine output less than 30 ml/hour ✓

43
203- A 46 year-old patient is in the male Urology Ward after the surgical
removal of the stone from his left kidney through percutaneous
nephrolithotomy under general anesthesia He has nausea and dull aching
pain in left lumbar region His nephrostomy bag is attached through a
tube in his left kidney for a few days (see image). What of the
following is definition of the procedure?
A. Surgical procedure
B. Incision of the skin to kideny for removal of the stone ✓
C. Destruction of a kidney stone using sound waves

204- A 46 year-old patient is in the male Urology Ward after


the surgical removal of the stone from his left kidney through
percutaneous nephrolithotomy under general anesthesia He has nausea
and dull aching pain in left lumbar region His nephrostomy bag is
attached through a tube in his left kidney for a few days (see
image). What of the following problems is needed to focus
on?
A. Risk of impaired skin integrity due to infection ✓
B. Disturbed life cycle related to nephrostomy bag
C. Knowledge deficiency for self-care management
D. Impaired social interaction due to altered life style

205- 46-year-old patient is in the male Urology Ward after the surgical
removal of the stone from his left kidney through percutaneous
nephrolithotomy under general anesthesia. He has nurse and dull acting
pain in left lumbar region. His nephrostomy bag is attached through a
tube in his left kidney for a few days (see image) What findings should
alert the nurse to report to the physician immediately?
A. Abdominal discomfort and conniption
B. Severe pain and discomfort at surgical site ✓
C. Presence of blood and stone gravels in urine
D. Urine output less than the identified amount

206- Nephritic syndrome urine color?


A. Bright red ✓
B. Dark Brown

44
207- A nurse is collecting a urine of a 4-year-old child with nephrotic
syndrome. Which of following observation about the color of the child's
urine the nurse expected to will chart?
A. Bright red
B. Amber
C. Dark, frothy ✓
D. Orange

208- Nephrotic syndrome urine color?


A- Bright red
B- Dark brown urine ✓

209- A nurse is collecting a urine of a 4-year-old child with nephritic


syndrome. Which of following observation about the color of the child's
urine the nurse expected to will chart?
A. Bright red ✓
B. Amber
C. Dark, frothy
D. Orange

210- Which patient is a contraindicated for enema?


A. Glaucoma
B. Hypertensive
C. Renal failure. ✓
D. liver disease

211- A nurse is caring for a 55 years patient with abdominal pain. On


assessment the nurse noticed lower abdominal distention on palpation
and was concerned about urinary retention. Which of the following
clinical indicators supports the nurses concerns?
A. Hyperthermia
B. Urine output of 35 ml/hour
C. Imbalanced electrolytes level
D. Voiding small amounts of urine at a time ✓

45
212- Skeletal traction urinary catheter were discontinued for a patient
who was immobilized in traction for sex weeks. The patient developed a
problem with urinary incontinence. Which of the following is the most
appropriate intervention?
A. Scheduled toileting
B. Bladder retraining ✓
C. Promoted voiding
D. Behavioral training
ICP

213- The nurse is assessing patient after craniotomy. The patient's blood
pressure is 180/65 ICP is 25.What is the patient's cerebral perfusion
pressure? Note:
A. 72mmHg CPP= MAP-ICP
B. 81mmHg ICP = 25
C. 78mmHg ✓
MAP (main arterial pressure ):
Systolic blood pressure +(2× Diastolic
blood pressure)/3
180 +(2×65) ÷3= 103
‫نطبق القانون‬
ICP= MAP-ICP >> 103-25 =78

214-The patient ICP 10 : 15 mmhg. What should the nurse expect to do?
A. Normal
B. Slight elevation ✓

215- Which one of the following signs and symptoms is associated with
increased intracranial pressure?
A. Restlessness and confusion
B. Bradycardia and hypertension ✓
C. Tachycardia and hypotension
D. Respiratory depression and headache

216- What is Signs of ICP?


A. Tachypnea
B. Intermittent tachycardia
C. Restlessness ✓

46
217- Which one of the following early signs and symptoms is associated
with increased intracranial pressure?
A. Restlessness and confusion ✓
B. Bradycardia and hypertension
C. Tachycardia and hypotension
D. Respiratory depression and headache

218- What is the initial sign should the nurse Assess indicates that ICP for
patient not increased?
A. Patient tolerate lower head of bed
B. Normal blood pressure
C. Absence of confusion and restlessness ✓
D. Absence of headache

219- A 10 month old infant is admitted to the surgical ward with


hydrocephalus. Which of the following indicate increased intracranial
pressure?
A. Bulging ✓
B. Decrease blood pressure
C. Rapid, shallow breathing
D. Increase body temperature

220- A nurse is monitoring a 3-year-old child for signs and symptoms of


increased intracranial pressure (ICP) after a craniotomy. The nurse
plans to monitor for which early sign or symptom of increased ICP?
A. Excessive vomiting ✓
B. Bulging anterior fontanel
C. Increasing head circumference
D. Complaints of a frontal headache

221- Child after Ventriculoperitoneal shunt surgery and the nurse note
the shunt not working , What sign of increase ICP?
A. Nausea and refund to eat
B. Change in neurological state ✓
C. Blood drain from ear

47
222- For an infant with hydrocephalus, a nurse should plan to monitor
for what sign or symptom of increased intracranial pressure?
A. High-pitched, shrill cry ✓
B. Decrease in systolic blood pressures
C. Depressed fontanel
D. Increase in respirations

Blood Transfusion

223- Which of the following religion is prohibited the blood transfusion?


A. Christian
B. Muslims
C. Jewish
D. Jehovah ✓

224- During blood transfusion. The patient suffering from low back pain
and chills. What should the nurse to do?
A. Give normal saline
B. Slow rate ✓
C . Review identification of blood

225- A nurse is caring for a client who is having an allergic reaction


febrile, flushed what the first nursing action?
A. Stop blood transfusion immediately ✓
B. Replace the blood with saline.
C. Administer an antihistamine.
D. Place the client flat with the feet elevated.

226- Patient with ongoing blood transfusion and the nurse observe that
patient has difficulty to breathing and shivering. What the nursing
priority?
A. stop infusion ✓
B. call physician
C. slow rate of infusion
D. give Intravenous fluid

48
227- During a patient's blood transfusion, which of the following are
associated with an allergic reaction the nurse should recognize that?
A. Diarrhea and rapid onset of chills
B. Itching and asthmatic wheezing ✓
C. Low back pain and acute renal failure
D. Distended neck veins and a rise in venous pressure

228- Patient with reaction from blood transfusion. What is the first
action?
A. Start open I.V infusion ✓
B. Change I.V set
C. Notify the doctor
D. Check vital signs

229- Patient with blood transfusions start to have blood transfusion


reactions the nurse stop the infusion, what should the nurse do next?
A. Check vital signs
B. Insert new IV set
C. Start normal saline ✓
D. Notify physician

230- The nurse is monitoring a client receiving a blood transfusion after


30 minutes when the client develops a cough with shortness of breath.
The client also complains of a headache and a racing heart. What should
the nurse do first?
A. Stop transfusion immediately
B. Replace the blood with saline.
C. Administer an antihistamine.
D. Slow the infusion rate ✓

231- A patient is to receive a prescribed blood transfusion using a


subclavian catheter. Catheter was inserted about 30 minutes ago. Which
of the following is the most appropriate step before starting blood
transfusion?
A. Check patency of catheter by flushing with normal saline solution
B. Position patient with the head of the bed elevated 30 degrees
C. Review result of chest radiograph completed 15 minutes ago ✓
D. Obtain patient's most recent complete blood count result.
49
232- The common complication for patent has osteoporosis Hip fracture
Patient has fracture in right arm with cast. The left arm has blood
transfusions IV and patient need another cannula for medication
which area should we put the new cannula?
A- Right arm
B- Left arm ✓

233- A 60 years old patient was diagnosed as having chronic


inflammatory bowel disease and was scheduled for colonoscopy. He was
ordered for the packed cells transfusion which to prior to colonoscopy
procedure. Before starting the infusion, the patient name, identification
number, date and blood type are cross checked by the nurse. What else
must be assessed before starting the blood infusion?
A. Drug allergic
B. Blood screening
C. Patient condition (V/S) ✓
D. Patient identification

234- A hospitalized patient has received transfusions of 2 units of blood


over the past few hours. A nurse enters the room to find the patient
sitting up in bed, dyspnea and uncomfortable. On assessment, crackles
are heard in the bases of both lungs, probably indicating that the patient
is experiencing a complication of transfusion. Which of the following
complications is most likely the cause of the patient’s
symptoms?
A. Febrile non-hemolytic reaction.
B. Allergic transfusion reaction.
C. Acute hemolytic reaction.
D. Fluid overload. ✓

235- A Girl come hospital with thalassemia . She is given blood


transfusion every 3 weeks. She came to hospital for blood transfusion and
the doctor ordered lab investigation before administration, he found on
results increase ferritin level. What should the nurse give for her to
prevent complication?
A. Deferasirox ✓
B. Iron supplement
C. Sodium chloride
50
236- A 50 years-old woman is receiving a blood transduction during the
post- operative period following a surgical removal of the uterus. The
patient had begun following a surgical removal of the uterus. The patient
had begun bleeding profusely four hours after the operation and the
surgeon ordered a blood transfusion. Thirty minutes after starting the
infusion of blood products, the patient begins to complain of lower back
pain and chills. Blood pressure 90/60 mmHg Heart rate 114/min
Respiratory rate 26/ min Temperature 38.4ْ C. What is the most likely
underling problem?
A- Hemolytic reaction ✓
B. Allergic reaction

237- Patient with blood type AB-ve for blood transfusion what are the
compatible blood group for this patient?
1- A-
2- B-
3- O-
4- AB- ✓

238- A 45 years old with severe upper GI bleeding is admitted to the


medical ward. The doctor orders the nurse to give the patient blood
transfusion Which peripheral IV catheter gauge is best for this
procedure?
A. 14 gauge
B. 16 gauge ✓
C. 20 gauge
D. 24 gauge

239- A nurse has an order to transfuse a unit of packed red blood cells to
a client who does not currently have an intravenous (IV) line inserted.
When obtaining supplies to start the IV infusion, the nurse selects an
Angio catheter with a size of?
A. 18 gauge ✓
B. 21 gauge
C. 22 gauge
D. 24 gauge

51
240- What will happen to the patient if whole blood transfused exceeds
four hours in room temperature?
A. Hypokalemia
B. Hyperkalemia ✓
C. Hyponatremia
D. Hypernatremia

241- A 24 Years-old woman a diagnosed as having hemolytic anent


physician ordered to transfuse one unit of packed red blood transfusion,
the patient starts flushing and complained of dyspnea generalized body
itching (see lab results) TEST Result Normal values Hb 130 142-176 g/L
WBC 4.5 4-10.5 X 10°/L RBC 3.0 3.8-5.1 X 102/L which following nursing
diagnosis is related to blood and occludes the patient's problem?
A. Bacteremia
B. Fluid overload
C. Hypovolemic shock
D. Transfusion reaction ✓

242- A 42 year old man with thalassemia received a packed cell


transfusion. The nurse assess the stability of his condition after
transfusion by monitoring the vital signs and general condition every two
hour. When should the nurse immediately report the patient condition?
A. Sever headache and raised blood pressure
B. Raised body temperature and flushed skin ✓
C. Joint pain, body ache listlessness
D. Restlessness & bradycardia

243- Medical Ward manager was evaluating a blood transfusion to a


patient admitted with vehicle accident. The nurse knows that the patient
post blood transfusion. What is the time interval the student should asse
hour of the blood transfusion process?
A. As ordered by the physician
B. As stated in the hospital policy ✓
C. As instructed by the registered nurse in
D. As directed by the blood bank staff

52
244- The hospital director attended to ward for evaluation of staff
regarding vital signs follow up during blood transfusion. What is the
most appropriate action for monitoring vital signs during blood
transfusion depends on?
A. According to physician order
B. According to nursing supervisor instructions
C. According to hospital policy ✓
D. According to blood bank staff instruction

245- A patient receives a blood transfusion for severe anemia after


surgery. While evaluating the patient, the nurse finds that the patient’s
oral temperature has begun to rise from 36.8C to 38.3 C. Which of the
following should the nurse do?
A. Discontinue the intravenous line and restart at another site
B. Use a blood cooling device to cool the blood as it infuses
C. Give the patient antipyretic medication and continue the transfusion as
ordered
D. Stop the transfusion, keep the vein open with normal saline, and notify
the doctor immediately ✓

246- Postoperative patient the doctor ordered blood transfusion. The


nurse assessed the patient after 15 minutes. The patient feels freezing and
tight breathing. What should the nurse do first?
A. Stop infusion, start normal saline and notify the doctor ✓
B. Stop infusion , cover patient with blanket and notify the doctor
C. Slow infusion rate to half and give anti allergic medication

247- How much of blood volume for patient has a 10% blood loss and his
weight is 50kg?
A. 300ml Note:
B. 400ml ✓
Formula of Blood
Transfusion =50*80/10= 400
C. 3000ml Adult weight kg *70
D. 4000ml Child weight kg *80

53
248- 5 years old boy experienced an accident. A stranger taken him to
hospital, after stabilization he needs blood transfusion. From whom you
should take the consent?
A. Just wait for the parents ✓
B. From the stranger
C. From the child
ABG

Note: Normal value


PH: 7.35 – 7.45
PaCO2: 35 – 45 mm Hg
HCO3: 22 – 26 mEq/L

249- Nurse is caring for a client with a nasogastric tube that is attached to
low suction the nurse monitors the client, knowing that the client is at risk
for which acid-base disorder?
A. Metabolic acidosis
B. Metabolic alkalosis ✓
C. Respiratory acidosis
D. Respiratory alkalosis

250- Patient ABG PH 7.38, Paco2 50, Hco3 6 normal. What is the ABG
interpretation?
A. Compensated Metabolic alkalosis
B. Uncompensated Metabolic acidosis
C. Compensated Respiratory acidosis ✓
D. Uncompensated Respiratory alkalosis

251- ABG reading was low PH,high PCO2, normal Hco3 what the
interpretation?
A. Compensated respiratory acidosis
B. Uncompensated respiratory acidosis ✓
C. Metabolic acidosis
D. Metabolic alkalosis

54
252- Gastric suction can cause?
A. Metabolic acidosis
B. Respiratory acidosis
C. Metabolic alkalosis ✓
D. Respiratory alkalosis.

253- Patient with ABG PH low, pco2 high, hco3 normal what is expected
signs and symptoms for the patient?
A. Vomiting and difficult breathing SOB ✓
B. Headache and vomiting
C. Shallow, rapid breath and vomiting
D. Kaussmal respiration

254- Patient when entered ER he said, my heart will get out of my chest I
fell that I will die he diagnosed with panic attack what is the medical
problem that will be developed if panic not controlled?
A. Respiratory acidosis
B. Respiratory alkalosis ✓
C. Metabolic acidosis
D. Metabolic alkalosis

Nasogastric Tube
255- A surgical unit nurse assessed a 35 years-old post appendectomy
observe abdominal distention with absent bowel sounds. Which of the
following interventions is most appropriate?
A. Encourage ambulation
B. Provide liquid diet as tolerated
C. Ensure patency of nasogastric tube ✓
D. Check surgical site for signs of infection

256- A child with tracheoesophageal fistula is scheduled for an operation.


What should the nurse do preoperatively?
A. Insert NGT
B. Suction periodically ✓
C. Prepare tracheostomy set if necessary

55
257- The nurse is preparing to administer an enteral feeding to client via
a nasogastric feeding tube the most important actions of the nurse?
A. Averify correct placement ✓
B. Check the feeding solutions matches the dietary order
C. Aspirate abdominal content to determine the amount of last remaining in
stomach
D. Ensure that feeding solutions is at room temperature

258- W hat is the goal for NGT insertion after cholecystectomy?


A. Prevent aspiration
B. Decompress stomach ✓
C. Suction gastric content

259- What is rational of insertion therapeutic NGT of patient in the


hospital?
A. Lavage ✓
B. Decompression
C. Juice suction

260- The comatose patient with NGT for feeding and positioned in low
fowler. The nurse enter the patient room. She found the patient in supine
position. She auscultated lung sound with diventure. What is the most
appropriate nursing diagnosis?
A. Risk for injury
B. Risk for aspiration due to NGT ✓

261- 80 years old patient married from 70 years old woman. He suffers
from Inability to do daily activities, bedridden with diaper and NGT for
feeding. The patient need long term care. Which of the following
developmental stage for both of them?
A. Aging ✓
B. Middle age
C. Adulthood

262- What is the most common Complication of NGT?


A. Hemorrhage
B. Aspiration ✓
C. Infection
56
263- Patient with ulcerative colitis the doctor order to give him 300ml
through NGT. What is nursing intervention?
A.Warm the feed before giving ✓
B. Give every 15 minute

264- Nurse is preparing to insert a nasogastric tube into a client. The


nurse places the client in which position for insertion?
A. Right side
B. Low Fowler's
C. High Fowler's ✓
D. Supine with the head flat

265- A nurse is starting the morning shift and has received the patient’s
handover from night shift nurses and completed the morning rounds. The
nurse is preparing to perform physical assessment for the patients Which
of the following patients should be assessed first?
A. A postoperative patient with total hip replacement ✓
B. A patient with a stroke who is on nasogastric tube.
C. A patient with congestive heart failure
D. A patient scheduled for appendectomy who was admitted.

266- Patient with abdominal pain and abdominal distention and unable to
pass stool. When preforming rectal examination there is no stool and
rectum clean. After that the nurse inserted nasogastric tube for patient.
What is the purpose for Nasogastric tube For this patient?
A. Obstruction
B. Enteral feeding
C. Decompression ✓

267- Patient admitted to ICU with tracheotomy and Nasogastric tube.


The vital signs are normal. The nurse was doing suction for patient and
yellow secretion coming out. What is the cause?
A. Diet imbalance
B. Infection around tracheotomy ✓
C. Iatrogenic infection

57
268- Patient with NGT and intermittent suction, suddenly the patient
complaining of pain and the abdomen is distended, at 10:00am NGT
output=120 cc, 14:00 NGT output= 60 cc, what is the first action should
be taken by the nurse?
A. Increase the suction
B. Administer pain medication
C. Irrigate NGT to check patency ✓
D. Notify the physician

269- Patient connected with NGT. The nurse noticed that the patient is
dry and crackles. What is a first action before start feeding?
A. Aspiration content.
B. Check place of NGT.
C. Check line is patent. ✓

270- The patient is postoperative and sedated. Which of the following


signs require immediate intervention?
A. Urine bag is full
B. Hemovac bag is full with serosanguineous drainage
C. Intermediate nasogastric suction is not connected ✓
D. Nasal cannula 4L without humidification

271- A patient underwent gastric procedure. The nurse provides post-


operative care and an hour after the operation the patient’s a face
becomes pale and she is distressed. She reports feeling nauseous and
requests a vomit bowl. The nurse notes that she frequently pushes the
button of the patient - controlled analgesia pump?
A. Administer a post-operative anti-emetic
B. Check the patency of the Nasogastric tube ✓

272- The nurse prepares the client for the removal of a nasogastric tube.
during the tube removal, the nurse instructs the client to take which
action?
A. Inhale deeply.
B. Exhale slowly.
C. Hold in a deep breath. ✓
D. Pause between breaths

58
273- A 53 years old man patient in transferred to the recovery room after
coronary bypass surgery. He is on mechanical ventilation and chest tube
drainage is attached. Intravenous line, nasogastric tube and urinary
catheter are intact. What short term goal of care needs priority?
A. Monitor heart sounds and report abnormality
B. Monitor kidney functioning by urinary output
C. Appropriate positioning and promote rest ✓
D. Prevent infection at the incision site

274- Patient admitted to ICU with tracheotomy and Nasogastric tube.


The vital signs are normal. The nurse was doing suction for patient and
yellow secretion coming out. What is the cause?
A. Diet imbalance
B. Infection around tracheotomy ✓
C. Iatrogenic infection

275- A 73 years old bed ridden man is admitted in the geriatric ward in a
semi-conscious state accompanied by one of his neighbors. He has loose
motions on and off for the past few months. He is wearing diaper and has
nasogastric tube inserted. He has bruises on his forearms and is
developing bedsores at the sacral area. In physical examination, mild
crackles sounds are auscultated bilaterally in the upper part of his chest
and at sternum . Blood pressure 106/86mmHg Heart rate 96/min
Respiratory rate 28/min Temperature 39.6 C. Which of the following
nursing problems should be prioritized?
A. Self-care deficit due to dependency on others
B. Altered conscious level related to disease process
C. Impaired physical mobility related to weakness
D. Ineffective airway clearance due to chest congestion ✓

276- An 81-year-old bed ridden patient in the Geriatric Ward was pyretic
through his nasogastric tube. After an hour, the patient deep sleep, his
breathing pattern changed and he did not rep calling his name. Which of
the following should be the immediate nursing intervention?
A. Call the physician to examine the patient
B. Check for the correct dosage given
C. Check vitals and inform physician ✓
D. Try to wake up the patient
59
Infection Controle

277- A 32 years old man develops chronic productive cough. He has not
been feeling hungry and has lost three kilograms body weight in the past
three weeks. On examination of his lung fields, there was diminished
breath sound and widespread crackles. An early morning sputum culture
was sent to the lab?
A. Droplet
B. Contact
C. Airborn ✓

278- Diphtheria which type of isolation?


A. Droplet ✓
B. Airborne

279- A 45-year-old patient admitted with pulmonary tuberculosis. The


unit nurse placed the patient in an isolation room with negative air
pressure and prepared all the personal protective equipment at the
entrance of the room. What type of precaution measure has the nurse
activated?
A. Contact
B. Droplet
C. Airborne ✓
D. Standard
60
280- 8 month-old child diagnosed with bronchiolitis due to respiratory
Syncytial virus (RSV) is admitted to the pediatric hospital. The nurse
should initiate which of the following isolation precaution?
A. Contact ✓
B. Airborne
C. Standard
D. Droplet

281- The nurse is assigned to care for the a patient with Ebola virus
disease. Which of the following is the most common mode of transmission
for Ebola virus?
A. Vector
B. Airborne
C. Direct contact ✓
D. Common vehicle

282- Nurse receives a telephone call from the admission office of the
hospital and is told that a patient with streptococcal meningitis will be
admitted to the Medical Unit. The nurse is planning to apply infection
control measures for the patient. Which type of isolation precaution the
nurse must observe?
A. Droplet precautions ✓
B. Contact precautions
C. Airborne precautions
D. Standard precautions

283- TB transmission?
A. Air drop ✓
B. Exposure to mouth
C. Physical

284- A 35 year old patient was admitted to a medical ward with


confirmed agnosies of meningococcal infection. Which of the following
infection control preventions the nurse should implement?
A. Droplet precaution ✓
B. Contact precaution
C. Airborne precaution
D. Standard precaution
61
285- In order to reduce the risk of disease transmission from a patient
with diphtheria, which of the following standard precautions would the
nurse implement?
A. Airborne
B. Contact
C. Droplet ✓
D. Ventilatory

286- You have just admitted a patient with bacterial meningitis to the
medical-surgical unit. The patient complains of a severe headache with
photophobia and has a temperature of 102.60 F orally. Which type of
isolation should be suitable for the patient?
A. Air borne
B. Contact
C. Droplet ✓

287- A 5 years-old child is seen in the primary care clinic with mild fever,
headache, and malaise for about 2 days and today he has a rash filled
with fluids. which of the following is the best suggested diagnosis?
A. Chicken pox ✓
B. German measles
C. Measles
D. Scarlet fever

288- What is the type of isolation to Pt with Meningitis?


A. Negative pressure
B. Positive pressure ✓
C. Reserve

289- HIV patient will be risk for which of life threating disease?
A. Pneumonia
B. Tuberculosis ✓

62
290- Suspect patient with TB what is the highest priority nursing action?
A. Isolate the patient in private negative pressure room. ✓
B. Take nasal swab
Note:
Airborne precautions = Negative pressure room
Droplet precautions = Positive pressure room

291- HIV patient will be risk for which of life threating disease?
A. Pneumonia
B. Tuberculosis ✓

292- What is the organism that causes cervical cancer?


A. HPV ✓
B. Herpes 2
C. HIV

293- Which of the following diagnostic tests is definitive for TB?


A. Chest x-ray
B. Mantoux test
C. Sputum culture ✓
D. Tuberculin test

294- What is the transmission method of HAb virus?


A. Food ✓
B. Blood
C. Sexual transmission

295- Patient came to ER and diagnosed with meningitis. How should the
nurse handle with patient?
A. Keep patient in same isolation room ✓
B. Use mask N95 all times
C. Limit the visitors

296- Incubation period of infection is?


A. Person is most infectious and nonspecific sign and symptom. ✓
B. Organism growing and multiplying.
C. Recovery from infection.
D. Presence of specific sign and symptoms
63
297- The hospital observed MERS-COV infection speed to many persons.
The hospital not identified the affected from non-affected person. What
should do to reduce the risk?
A. Isolate all patients
B. Prevent visiting
C. Apply standard precautions to all patients ✓
D. Wear protective equipment

298- A nurse working in medical unit is preparing to with droplet


precaution measures in place. The following personal protective
equipment; eyewear. What is the correct sequence for putting the
equipment on?
A. Face Mask, Gown, Eyewear, and Gloves
B. Gown, Face Mask, Eyewear, and Gloves ✓
C. Eyewear, Cloves, Face Mask, and Gown
D. Gloves, Gown, Face Mask, and Eyewear

299- What is the Minimal PPE for aids?


A. Gloves + nonsterile gown ✓
B. Sterile gloves +sterile gown ,
C. Mask +sterile gloves

300- A nurse working in medical unit is going out (removing) with


droplet precaution measures in place. The following personal protective
equipment; eyewear. What is the correct sequence for putting the
equipment off?
A. Face Mask, Gown, Eyewear, and Gloves
B. Gown, Face Mask, Eyewear, and Gloves
C. Eyewear, Cloves, Face Mask, and Gown
D. Gloves, Eyewear , Gown, , and Face Mask ✓

301- The doctor touches the bed by the sterile gloves the nurse should?
A. Ask him to change the gloves and give him new one ✓
B. Let him to complete the Central line procedure.

64
302- To make cornea reflux you should use?
A. Straily gloves.
B. Swap cotton ✓
C. Cotton tap
D. Sterile tongue depressor

303- Mr. X attended in outpatient clinic with symptoms of shortness of


breath, diarrhea and severe respiratory distress Which of the following is
the best diagnosis of Mrs. A case?
A. Corona virus ✓
B. Swine Flue
C. Zika virus
D. Hepatitis

304- A nurse is caring a patient diagnosed with pulmonary tuberculosis


and she has to wear N95 mask when she enters to the parent room. Based
on the cycle of infection, which of the following is the reason to use the
mask?
A. Break the transmission chain at the portal of entry. ✓
B. Break the transmission chain at the portal exit

305- Patients eat contaminated food and the diagnosed with hepatitis A as
result of that food which of the followings related to hepatitis A disease?
A-The disease is transmitted through the blood.
B- The disease is transmitted through sexual contact.
C- Incubation period 3-5weeks. ✓
D- Incubation period 4-5 month

306- Patient with smallpox. He became fine and no symptoms but still
isolated for 12-14 days. What is the type of the period?
A. Incubation period ✓
B. Infective period
C. Latency period
D. Communicable period

65
307- A patient in surgical was transferred to isolation room after the
wound swab confirmed to have methicillin Resistant staphylococcus
Atreus MRSA. Which of the following measures should the nurse take to
prevent infection in the ward?
A. Clean the Room three times a day
B. Discard all soiled dressing into waste bag
C. Instruct the patient to wash hands regularly
D. Wear gloves and gown on every entry into the room ✓

308- The nurse administered a prescribed intramuscular medication to a


patient during a home health visit. How should the nurse dispose of the
used needle and syringe?
A. Recap the needle, then place the needle and syringe into a waterproof.
container until safe disposal can be made.
B. Bend the needle back towards the barrel of the syringe before putting The
and syringe in metal trash container.
C. Wrap the needle and syringe in disposable paper before putting the needle
and syringe into the dirty section of the nurse’s equipment bag.
D. Put the needle and syringe directly into a puncture-resistant
plastic.container that has a lid ✓

309- When planning discharge teaching for a patient hospitalized for


treatment of the 3rd burns over 30% of the body, a nurse knows it is most
important to include instructions regarding the loss of large amounts of
serum occurring with burns and the resulting loss of immune function.
Which of the following instructions should be include?
A. Wash hands frequently each day ✓
B. Wear supplemental oxygen at night
C. Wear masks while in public spaces
D. Take a multiple vitamin tablet night

310-Incubation period of infection is?


A. Person is most infectious and nonspecific sign and symptom. ✓
B. Organism growing and multiplying.
C. Recovery from infection.
D. Presence of specific sign and symptom.

66
311- A 53 years old man is diagnosed as HIV positive. He had a history of
and off diarrhea and progressive weakness for past few months. He
sigmoidoscopy. Which of the following measure should the healthcare
providers evaluate?
A. Reverse isolation and optimum nutrition ✓
B. Confidentiality and social support system
C. Health maintenance and follow up regimen
D. Family screening and prevent spread of infection

312- Patient with chlamydia she delivers then baby given erythromycin
ointment to prevent?
A. Bilateral cataracts
B. Opthalmic neontorum ✓

313- A patient visiting the 10 days after a sinus surgery for a follow up
complains of having a bad taste in the mouth. The nurse smells of foul
odor while examining the patient mouth. What does the suspect the
patient may have?
A. Pulmonary decompensation
B. Hemorrhage
C. Aspiration
D. Infection ✓

314- A 33-year-old woman has come to the outpatient clinic for treatment
of a vaginal infection. Physical assessment reveals yellowish excessive,
thin offensive and frothy discharge. Which of the following is the most
likely diagnosis?
A. Candidiasis
B. Trichomoniasis ✓
C. Bacterial vaginosis
D. Chlamydia

315- Patient with a colostomy complains of itching of the peris tornal skin.
On assessment, the skin is covered in a red rash with white patches visible.
What is the most likely cause of this condition?
A. Not changing the pouch regularly
B. Candidiasis ✓
C. Consuming acid-producing foods D. Dehydration
67
316- An infection control nurse is responsible to conduct the surveillance
on how the hospital staff take precautionary measures against flu, based
on her findings, she is responsible to conduct the awareness razing for
the doctors and nurses in the in-patient areas. Which of the following is
the goal of the nursing practice here?
A. Infection control management
B. Research and survey
C. Primary prevention ✓

317- Before preparing a client for the first surgical case of the day, a part-
time scrub nurse asks the circulating nurse if a three minute surgical
hand scrub is adequate preparation for this client. Which response
should the circulating nurse provide?
A. Inform the nurse that hand scrubs should be three minutes between cases
B. Direct the nurse to continue the surgical hand scrub for five minute
duration ✓
C. Validate that the nurse is implementing the OR policy for a surgical hand
scrub.
D. Ask the experienced nurse to perform this scrub since it is the first one of
the day

318- A patient diagnosed with septic shock had an upward-trending


glucose level (180-225 mg/dL) requiring control with insulin. The
patient's spouse asks why is insulin needed as the patient does not have
diabetes Which of the following is the most appropriate nursing response
to educate the patient's spouse?
A. "It is common for critically clients to develop type diabetes. We give
insulin to keep glucose level under control (less than140 mg/dL)

B. " Patient had diabetes before, you just didn't know it. we give insulin to
keep glucose level in the normal range (70-110 mg/dL)

C. " Increase in glucose is a normal response to stress by the body . We


give insulin to keep the level 1t 140-180- mg/dL" ✓

D. " Increase is common in critically ill clients and effects their ability to fight
off infection. We give insulin to keep the glucose level in the normal range
(70-110 mg/dL)"
68
3 Questions Jizan
319- Jizan province is poised to achieve malaria elimination. There is a
need to change from a policy of passive case detection to reactively and
proactively detecting infectious reservoirs that require new approaches to
surveillance. For discovering health problems and diseases inside the
community. What is the priority of surveillance?
A. Social surveillance
B. Public surveillance and prevent infection. ✓
C. Financial surveillance and previous health diseases.

320- Jizan province is poised to achieve malaria elimination. There is a


need to change from a policy of passive case detection to reactively and
proactively detecting infectious reservoirs that require new approaches to
surveillance. For discovering health problems and diseases inside the
community. What is the priority of surveillance?
A. Social surveillance
B. Financial surveillance and previous health diseases.
C. Incidence and prevalence of the health diseases. ✓

321- Jizan province is poised to achieve malaria elimination. There is a


need to change from a policy of passive case detection to reactively and
proactively detecting infectious reservoirs that require new approaches to
surveillance. For discovering health problems and diseases inside the
community. What is the priority of surveillance?
A. Social surveillance
B. Surveillance for previous and new health problems ✓
C. Financial surveillance and previous health diseases

Note: Social and financial ‫في أسئلة جيزان نستبعد‬

69
Community and Leadership question

322- Health education about accident in school, role of who?


A. Nurse ✓
B. Doctor
C. Social worker
D. Administration

323- Nurse supervising on assistant nurse and delegate assignment to him


and say "diving up patient's assignment". What type of leadership?
A. Autocratic
B. Laissez-faire ✓
C. situational
D. transformational

324- A nurse explains to a patient that an endoscopy tube will be inserted


down the throat during a gastroscopy which will cause gagging, but it
must be swallowed. What type of communication the endoscopy nurse
has used here?
A. Assertive ✓
B. Interactive
C. Formative
D. Informative

325- When the nurse knows the strengths and weaknesses point in her
personality. What is consider?
A. Self-awareness
B. Self-understanding
C. Self – concept ✓

326- The Saudi Ministry of health is developing education health


program. All clerks not aware for the program and not have skills to
deal. What should the authority do?
A. Survey ✓
B. Gap analysis
C. SWOT analysis

70
327- Patient admitted to hospital for first session of chemotherapy. The
patient WBCs is low and the nurse informed doctor. What is the nurse?
A. Patient advocate ✓
B. Professional communication
C. Professional judgment
D. Critical thinking

328- The nurse was giving sufficient information for parent regarding
health for their baby What is this considering?
A. Positive denial
B. Detachment
C. Empowerment ✓

329- The nurse checking patient file to transfer him to operation room
She found that the doctor didn't get the patient's signature in informed
consent. What is this consider?
A. Negligence ✓
B. Malpractice

330- The doctor was busy. He requested from the nurse to see certain
patient. What is this called?
A. Practitioner
B. Responsibility ✓

331- A head nurse of an intensive care unit wrote a report about the
needs of her department and gave it to the nursing supervisor. Which of
the following types of communication is this?
A. Upward ✓
B. Horizontal
C. Diagonal
D. Downward

332-The head nurse needs to increase her staff nursing the next year.
What is this considering?
A. Selection
B. Recruitment
C. Downsizing
D. Organizing ✓
71
333-The head nurse needs to increase her staff nursing the next year.
What is this considering?
A. Selection
B. Recruitment
C. Downsizing
D. Rightsizing ✓

334- Head nurse want to increase size of staff next year, what is the first
thing done by human resources?
A. Downsizing
B. Recruitment
C. Selection
D. Rightsizing ✓

335- The head nurse notice conflict between the stuff nurses and try to
solve this conflict. She told them that one side give in the patient
satisfaction what type of this solution?
A. avoidance
B. accommodation ✓
C. Compromise
D. Collaboration

336- Patient admitted to hospital for first session of chemotherapy. The


result WBC is low and the nurse informed doctor. What is the nurse?
A. Patient advocate ✓
B. Professional communication
C. Professional judgement
D. Critical thinking

337- A 40-year-old man was admitted to a male medical department with


pneumonia. A nurse in the department explained and obtained informed
consent for a chest CT scan procedure with contrast. With of the
following ethical principles underpinning informed consent?
A. Autonomy ✓
B. Non maleficence
C. Beneficence
D. Respect

72
338- Charge nurse hears a nurse speaking to patients with disrespect.
which of the following is the best action for the charge nurse?
A. Apologize to patient
B. Tell nurse to be kind
C. Take over the care
D. Speak privately with nurse ✓

339- When the employees assess the performance of their managers. They
give positive feedback for them. What is the type of the evaluation?
A. Supervisor evaluation
B. Subordinate evaluation✓
C. Director evaluation
D. Manager evaluation

340- Highly registered nurses work in Community health agencies was


visiting people in certain area for observation and assessment. They
found high risk for lead toxicity families in their home. They connected
them with suitable agencies for testing. What is the type for that?
A. Screening ✓
B. Surveillance
C. Assessment
D. Case finding

341- Newly promoted (nurse manager). what's priority for her?


A. Supervise patient care delivered by staff
B. Complete evaluation of nursing staff ✓
C. Make unit plan and operational pla

342- Community health nurse visited Alzheimer's disease patient. The


patient complained from bruises and many injuries. The caregiver denied
any fallness caused for patient. What is the nurse response?
A. Restrain the patient
B. Blame to patient that he is the reason.
C. The cause is a cognitive impairement
D. Arrange room and provide lightnight in room ✓

73
343- The head nurse of a Coronary Care Unit delegated the staff a senior
nurse in that unit what initial step must the head nurse implement
before?
A. Check the hospital policies for delegating tasks. ✓
B. Explain the task to the senior nurse.
C. Negotiate with the senior nurse.
D. Take the signature of the senior nurse

344- Nurse plan community program to those who experienced a disease


And assess the recovery or rehabilitation or disability Which level of
prevention?
A. Primary
B. Secondary
C. Tertiary ✓

345- What is an example of secondary level of prevention?


A-How to be smoking cessation .
B-Given education on HIV to high school students
C. Nurse Combine to medication ✓

346- A new community nurse was caring for a long - term disabled
patient. The patient said, " I prefer to have my care given this way “. the
nurse explained more alternative approaches were clinically more
effective. However, the patient was Insisting on his opinion and said, " I
am more comfortable if my care is given this way " What should the
nurse do in this case to manage the situation?
A. accept that the patient knows best what works for him.
B. review the recent research with the patient to convince him.
C. give evidence-based care according to current findings.
D. Compromise between patients preference and research approach of
care ✓

347- It is a managerial function that indicates leading the staff in the most
effective method?
A. Planning
B. Directing ✓
C. Organizing
D. Controlling
74
348- Which of the following describes functional nursing?
A. Task and activity-oriented care. ✓
B. Involves a group of nurses working together to identify, plan, implement,
and evaluate patient centered care
C. Ratio of patient care
D. Emphasizes on continuity of care responsibility

349- Nursing supervisor newly elected Wants to use disciplinary


evaluation to nurses behavior What type of power she is using?
A. Reward
B. Coercive ✓
C. legitimate
D. Formal

350- Nurse manager prepares unit clinical operational plan What is top
priority in the plan?
A. Infection control
B. Staff orientation
C. Quality projects
D. Safe patient care ✓

351- The nurse is caring for 6 patients. Providing for them all care and
activities. What is the type of nurse according to nursing care delivery
system?
A. primary ✓
B. Case management
C. Functional

352-She plans of assigning competent people to fill the roles designed in


the hierarchy. Which process refers to this?
A. Staffing ✓
B. Scheduling
C. Recruitment
D. Induction

75
353- Which of the following best describes the role of a case
management?
A. promotes healthy lifestyle
B. provides coordination of care to ensure continuity ✓
C. possesses highly skilled communication methods
D. promotes the comfort of the patient

354- The community health care Facility observed low response for
immunizations activities. Then it decided to teach people about
immunizations and the importance for applying it. What would improve
the effect of response?
A. Modify immunizations program schedule.
B. Provide more health teaching programs. ✓

355- The head nurse gives low evaluation to the nurse because of her late.
What is the type of her evaluation effect?
A. Halo effect
B. Horn effect ✓
C. Central tendency

356- The head nurse gives high evaluation to the nurse because she is
active and cooperative with her colleague. What is the type of her
evaluation effect?
A. Halo effect ✓
B. Horn effect
C. Central tendency
D. Central effect

357- The head nurse is too busy to give evaluation to her staff. She gave to
everyone average evaluation. What is the evaluation type that the head
nurse used?
A. Halo effect
B. Horn effect
C. Central tendency ✓

76
358- Broken infusion machine. What to do?
A. Put a sticker and report. ✓
B. Put in the broken machines area.
C. Inform charge nurse.

359- The community nurse was providing home visit for a family to give
health instruction. The nurse observed while she entered the garage, she
was finding the car's oil had expired . What would you do?
A. get rid of oil and throw it away junk.
B. Call to the company and change it. ✓
C. Use the oil
D. Give it to the neighbors

360- The head nurse ensures that nursing staff give nursing care to
patients according to criteria. What is this consider?
A. Leading ✓
B. Supervisor
C. Delegation
D. Give orders

361- Nurse said to patient your identity will not be linked to responses
What's the ethical principles?
A. Privacy
B. Confidentiality
C. Anonymity ✓
D. Debriefing

362- Nurse manager tells one of the staff, “I don’t have time to discuss the
matter with you now. See me in my office later” when the latter asks if
they can talk about an issue. Which of the following conflict resolution
strategies did she use?
A. Smoothing
B. Compromise
C. Avoidance ✓
D. Restriction

77
363- The nurse use internet to see patient file outside hospital?
A. Telehealth
B. Telecommunication ✓

364- The nurse receives ECG for patient via Internet. What is consider?
A. Telehealth
B. Telecommunication ✓
C. Telemedicine

365- Why should the community nurse know about the disease in other
countries?
A. To give patient needs accordingly ✓
B. To know the perveance of the disease in certain country.
C. To help countries for solutions
D. To make research and studies

366- During haji nurse is caring for non-Arabic speaker What is most
appropriate action?
A. Use google translation.
B. Ask family members to interpret.
C. Call hospital Interpreter services ✓

367- What must be known on legal points of delegation when process to a


new nurse?
A. Evaluation of performance of delegate by clients
B. Actual time it takes to complete the task by delegat
C. Institution definition of the job description ✓
D. Number of times that the delegate has previously task
Note:
according to the hospital policy
‫أهم شرطين فيهم‬
1- Job prescription
2- Scope of practice

78
368- Managers create a new position patient educator work with head
nurse, what organizational relation ship?
A. square
B. solid line
C. matrix
D. Dotted line ✓

369- Joey plans to revisit the organizational chart of the department. He


plans to create a new position of a Patient Educator who has a
coordinating relationship with the head nurse in the unit. Which of the
following will likely depict this organizational relationship?
A. Box
B. Solid line
C. Broken line (Dotted) ✓
D. Matrix

370- Joey plans to revisit the organizational chart of the department. He


plans to create a new position of a Patient Educator who has a
coordinating relationship with the head nurse in the unit. Which of the
following will likely depict this organizational relationship?
A. Box
B. Dotted line
C. Broken line ✓
D. Solid line

371- Student nurse is asking the head nurse about touching


communications. What should the Manager response?
A. Don't touch patient head
B. You can touch patient as much you need
C. Each patient has different congnition for touching ✓

372- Stephanie is often seen interacting with the medical intern during
coffee breaks and after duty hours. What type of organizational structure
is this?
A. Formal
B. Informal ✓
C. Staff
D. Line
79
373- There is a new community nurse assigned to work ina Makkah,one
of the most multicultural cities in Saudi Arab has tointeract with
multicultural population every day. What is the most important step the
nurse should do before with the clients?
A. Asking the client about his/her cultural background
B. Conducting an appropriate culturological assessment ✓
C. Ensuring that the client has his/her a privacy.
D. Looking at the client's file to take history

374- CHN is a community -based practice. Which best explains this


statement?
A. The service is provided in the natural environment of people
B. The nurse has to conduct community diagnosis to determine nursing
needs and problems ✓
C. The service are based on the available resources within the community
D. Priority setting is based on the magnitude of the health problems identified

375- During fluid therapy on infusion, the infusion pump or Blood


pressure device is malfunctioned. What should the nurse do?
A. Put the pump aside inside the pt room ,
B. Put pump in utility room ,
C. Remove the pump and put it with other pumps
D. Place Broken sticker on pump and put it with other malfunctioned
pumps ✓

376- A nurse responsible for narcotic medication loses on ampule


morphine. She reports the problem to nurse manager. Which of the
following is the first thing she should say according to the SBAR
approach?
A. I have lost one ampule of morphine ✓
B. We should ask the nurse about the morphine
C. I think I left the lid of narcotics box open
D. We should open an investigation into problem

80
377- The community midwife has just finished an educational session
with a group of women regarding maternal health and annual
gynecological examination along with mammogram and Pap smear
testing. What outcome is the most desired?
A. Disease prevention.
B. Increased healthy living.
C. Early detection of related issues. ✓
D. Strengthened marital relationship

378- She takes pride in saying that the hospital has a decentralized
structure. Which of the following is NOT compatible with this type of
model?
A. Flat organization
B. Participatory approach
C. Shared governance
D. Tall organization ✓

379- Which of the following guidelines should be least considered in


formulating objectives for nursing care?
A. Written nursing care plan
B. Holistic approach
C. Prescribed standards
D. Staff preferences ✓

380-What is The term that use to describe people communicate with


same culture goals, values and ethical?
A. Intrapersonal communication
B. Interpersonal communication ✓
C. Intergroup communication
D. Interorganization communication

381- What is The term that use to describe people communicate their
own belief same culture, goals, values and ethical?
A. Interpersonal communication
B. Intrapersonal communication ✓
C. Intergroup communication
D. Interorganization communication

81
382- Stephanie considers shifting to transformational leadership. Which
of the following statements best describes this type of leadership?
A. Uses visioning as the essence of leadership. ✓
B. Serves the followers rather than being served.
C. Maintains full trust and confidence in the subordinates.
D. Possesses innate charisma that makes others feel good in his presence.

383- As a manager, she focuses her energy on both the quality of services
rendered to the patients as well as the
welfare of the staff of her unit. Which of
the following management styles does she
adopt?
A. Country club management
B. Organization man management
C. Team management ✓
D. Authority-obedience management

384- Joey orients his staff on the patterns of reporting relationship


throughout the organization. Which of the following principles refer to
this?
A. Span of control
B. Hierarchy ✓
C. Esprit d’ corps
D. Unity of direction

385- He emphasizes to the team that they need to put their efforts
together towards the attainment of the goals of the program. Which of
the following principles refers to this?
A. Span of control
B. Unity of direction ✓
C. Unity of command
D. Command responsibility

386- A 14- nurse manager assigns tasks according to clinical competencies


of the nurses. Which of the following is the management function?
A. Delegating ✓
B. Evaluating C. Planning
82
387- He discusses the goal of the department. Which of the following
statements is a goal?
A. Increase the patient satisfaction rate. ✓
B. Eliminate the incidence of delayed administration of medications.
C. Establish rapport with patients.
D. Reduce response time to two minute

388- A patient fell in the bathroom and his left leg was fractured, in order
to communicate information about the patient to the next shift. Which of
the following documentation should be used by the nurse at the nurse at
the end of the shift?
A. Kardex record ✓
B. Assignment record
C. Shift report
D. Incident report

389- A patient fell in the bathroom and his left leg was fractured, in order
to communicate information about the patient. Which of the following
documentation immediately should be used by the nurse?
A. Kardex record
B. Assignment record
C. Shift report
D. Incident report ✓

390- He likewise stresses the need for all the employees to follow orders
and instructions from him and not from anyone else. Which of the
following principles does he refer to?
A. Scalar chain
B. Discipline
C. Unity of command ✓
D. Order

391- A nurse manager includes staff in decision-making process what unit


structure is used?
A. Divisional
B. Centralized
C. Functional
D. Decentralized ✓
83
392- What is the main limitation of democratic style of leadership?
A. It ignores the incompetent personnel
B. Cannot be used in emergency situations ✓
C. Decreases collaboration among the team
D. May end up in inappropriate decision making

393- On orientation day, a nurse educator said to a new nurse" our


hospital provides the highest level of specialized healthcare in an
integrated educational and research setting" which of the following
aspects of the hospital's strategic plan is indicated by this statement?
A. Vision
B. Policy
C. Values
D. Mission ✓

394- The statement, "The Holy Spirit Medical Centre aims to provide
patient-centered care in a total healing environment" refers to which of
the following?
A. Vision
B. Goal ✓
C. Philosophy

395- Stephanie is a new Staff Educator of a private tertiary hospital. She


conducts orientation among new staff nurses in her department. Joseph,
one of the new staff nurses, wants to understand the channel of
communication span of control and lines of communication. Which of the
following will provide this information?
A. Organizational structure ✓
B. Policy
C. Job description
D. Manual of procedures

396- In the hospital digital dashboard, what types of data displayed?


A. Staffing
B. Financial
C. Performance ✓
D. Knowledge-based

84
397- During Cardiopulmonary Resuscitation (CPR) for a 75-year-old
man in the Emergency Department, the doctor introduced himself as a
leader for the CPR. What is the most appropriate leadership style for this
situation?
A. Autocratic ✓
B. Democratic
C. Bureaucratic
D. Laissez-faire

398- A nurse manager was not happy about low results of patient’s
survey. Which of the following is the first step for nurse manage?
A. Schedule meeting with staff
B. Start changes to improve
C. Review evaluation details ✓
D. Report to director

399-A nurse manager is open minded, listens to the team, understands


others, makes changes to improve unit operations and procedures. What
type of leadership style is this?
A. Situational
B. Democratic ✓
C. Compassionate
D. Transformational

400- The nurses in the Male Medical Unit took a signature on the from a
patient who needs an abdominal CT with contrast. What is the best
nursing practice?
A. Consent should be taken after the procedure
B. Verbal consent is acceptable for this procedure
C. Consent should be taken from the patient's relatives
D. Consent should be taken according to the organization's ✓

401- A nurse manager make booklet to let the staff practice in safe and in
sequence . What is the benefit from booklet?
A. Formulated clear guidelines and policy ✓
B. Follow organization orders

85
402- Newly nursing director assigned to a hospital . That is the first
should he do?
A. Evaluates the staff ✓
B. Change the head nurses
C. Change the roles
D. Nothing to do

403- The head nurse meets with staff nurses to discuss ways to improve
communication among shifts Which of the following statement best
exemplifies the final stage of conflict management?
A. "We need to clearly define the nature of the conflict”
B. "I will evaluate the outcomes of the strategies used monthly ✓
C. " Let us create a time line for the implementation of our strategies”
D. " I have to force you to follow the rules to resolve the issue”

404- The head nurse of a Coronary Care Unit delegated the staff a senior
nurse in that unit What initial step must the head nurse implement
before?
A. Check the hospital policies for delegating tasks ✓
B. Explain the task to the senior nurse
C. Negotiate with the senior nurse
D. Take the signature of the senior nurse

405- The nursing director wants to evaluate the quality of nursing care at
the in-patient areas The management team will evaluate on quarterly
basis the documentation and the relationship between the patient's length
of stay and the quality of care which of the following is the most
important data source to identify the quality of care?
A. Patient's complaints and time taken to resolve them
B. Patient's satisfaction level at the time of discharge ✓
C. Details of nursing notes for patient's progress

406- The nurse manager received complaints from some nursing to


improper work distribution and discrimination How should the nurse
manager handle the situation?
A. Investigate the complaints ✓
B. Individual counselling
C. Identify list of problems D. Continue to observe
86
407- There are times when manager evaluates her staff as she makes her
daily rounds. Which of the following is NOT a benefit of conducting an
informal appraisal?
A. The staff member is observed in natural setting.
B. Incidental confrontation and collaboration is allowed.
C. The evaluation is focused on objective data systematically ✓
D. The evaluation may provide valid information for compilation of a formal
report

408- Nurse manager first days in new hospital will?


A. Introduce plan and strategy to top manager in hospital
B. Introduce himself to staff ✓
C. Observation for 3 month before begin work

409- Which of the following is the most appropriate action for a head
nurse starting at a new hospital?
A. Make immediate change at the unit
B. Plan and coordinate new strategies ✓
C. Assess unit activities for at least three months
D. Ask about the previous head nurse managerial style

410- Nurse accidently dropped a medication ampoule, informed the


charge nurse, and completed an incident report form. The charge nurse
arranges medication replacement. Which of the following is the
immediate nursing action required?
A. Revise protocol for medication related incidences
B. Allocate senior nurses to medication assignments
C. Provide missed medication dosage to patient first ✓
D. Incident reporting must be given the priority

411- What is the Role of head nurse?


A. Ambulate the pt.
B. Monitor ECG for the patient
C. Recruiting the nurses
D. Allocate the nurse to the patient ✓

87
412- The nursing assistant with 20 years of experience approaches a
recently graduated nurse who recently passed the licensing examination.
The nursing assistant states the only difference between you and me is the
size of our pay checks. Which of the following is the most appropriate
response for the newly graduated nurse?
A. assert a hierarchical position
B. emphasize the additional education received
C. explain the legal difference in the scope of practice
D. focus on the need to work together for quality client care ✓

413- After accessing patients' medical records. Which behavior nurse


shows that patients confidentiality has been breached?
A. Reviews patients medical record
B. Read patients care plan
C. Disclosing patients information ✓
D. Documents medication administered.

414- Patient with expressive aphasia has impaired communication. What


is the short term goal for him to increase verbalization?
A. Able to communicate effectively within a week
B. Anticipate all patients needs
C. Able to determine basic needs ✓

415- During the assessment phase of a preoperative interview, the patient


reports feeling nervous. The patient conveys to the nurse that a parent
died in surgery due to malignant hyperthermia, to whom would this
information be most important?
A. Pacu nurse
B. Scrub nurse
C. Anesthesia team ✓
D. Charge nurse

416- A charge nurse in the nurse was noted poor staffing schedule in one
of the shift. The schedule caused problems among staff members in the
unit. The women the charge nurse should bring the problem to?
A. Supervisor ✓
B. Chief nurse
C. Follow charge nurse D. Grievance committee
88
417- A nurse intervention a patient recently admitted to long term care
facility to obtain information on the patient health perception. The nurse
encourage the patient to elaborate this change. Which of the following
type of question would be the most effective in this situation?
A. Analytical
B. Focused
C. Close
D. Open ended ✓

418- Which step in community organizing involves training of potential


leaders in the community?
A. Integration
B. Community organization
C. Community study
D. Core group formation ✓

419- In which step are plans formulated for solving community


problems?
A. Mobilization
B. Community organization ✓
C. Follow-up/extension
D. Core group formation

420- Rayed is provided new wheelchair program needs. Which of the


following Priority should include in the program?
A. Elderly ✓
B. Psychiatric
C. Pre-eclampsia
D. Measles

421- Rayed is provided frame work wheelchair program needs. Which of


the following Priority should include in the program?
A. School age
B. Psychiatric
C. Pre-eclampsia ✓
D. Measles

89
422- A nurse is showing stress and anxiety due to long duty hours. Which
of the following should the nurse manager do?
A. give day off ✓
B. give light tasks
C. offer vacation
D. talk about coping

423- Which of the following can be considered as a major development in


nursing home visits in terms of information technology?
A. Medication dispensing
B. Telehealth ✓
C. Patient monitoring
D. Prevention of epidemiological disease

CPR

424- Adult CPR?


A. 30:2 100 at 5 cm ✓
B. 15:2 120 at 5 cm
C. 15:2 120 at 5 cm
D. 30:2 100 at 4 cm

425- CPR depth?


In Infant: one third of chest = 1.5 INCH =4CM in infant
In Adult : 2 INCH = 5CM ✓

426- During a CPR of an admitted patient cardiac arrest a family


member tasks the unit nurse to be at the bedside and say final words to
their loved one The nurse explains to the family member that the scene is
very disturbing and the medical team is doing its job. The family member
still insists in witnessing the resuscitation efforts what would be the most
appropriate action by the unit nurse?
A. Allow family member to be at bedside ✓
B. Wait and ask permission from team leader
C. Call security to escort family member out
D. Refuse because there is not enough space in the room

90
427- What is the updated technique for CPR According to AHA?
A. 30:2120 at 5cm ✓
B. 15:212 at 5cm
C. 15:2120 at 5cm
D. 30:2100 at 4cm

428- 15. CPR site is?


A. Sternum ✓
B. Chest

429- CPR Technique is?


A. 30:2 120 (one secure 30 Compression and 2 breath, 120 compression in
one minute) ✓
B. 15:2 100

430- An infant arrives in the Emergency Department not breathing and


does not have a pulse. When starting CPR , which of the following
arteries is appropriate to perform the pulse checks?
A. carotid
B. Radial
C. Brachial ✓
D. temporal

431- While performing cardiopulmonary resuscitation (CPR) on a 5-year-


old child, the nurse palpates for a pulse. Which of the following sites is
best for checking the pulse during CPR in a 5-year-old child?
A. Femoral artery
B. Carotid artery ✓
C. Radial artery
D. Brachial artery

432- You are performing CPR on an infant when a second rescuer


appears. What is the next step in management?
A. Immediately transport the patient
B. Wait until exhausted, then switch
C. Have the second rescuer help with CPR, to minimize fatigue ✓
D. Have the second rescuer begin ventilations; ratio 30:2

91
433- A nursing instructor teaches a group of students about basic life
support. The instructor asks a student to identify the most appropriate
location to assess the pulse of an infant under 1 year of age. Which of the
following if stated by the student, would indicate that the student
understands the appropriate procedure?
A. Carotid
B. Popliteal
C. Radial
D. Brachial ✓

434- Patient came to ER with cardiac problem. The patient need CPR
suddenly. Who will do the procedure?
1. Care giver ✓
2. Surgeon

435- During her break time, the nurse was eating in the hospital cafeteria
when she noticed a female colleague grasping her neck and unable to
speak . which of the following maneuvers is the best to be performed by
the nurse?
A. Epley
B. Heimlich ✓
C. Valsalva
D. Leopold's

436- A 69 years old female patient in the Medical Word is in a semi


conscious state . Her pancreatic cancer is metastasized to her liver and
lungs and she is admitted for supportive treatment .Her physicians
discussed with the family that she will not be given the CPR to save her
life if she goes into the cardiac arrest . Her two sons agreed but the
daughter is indecisive . Which of the following is the critical thinking
behind not providing cardiopulmonary resuscitation?
A. Unilateral judgment of health professionals
B. Refusal of patients right to treatment
C. Ethical dilemma and is indecisiveness
D. Mercy killing to ease suffering ✓

92
437-Why should the nurse while doing CPR ensure from complete chest
recoil?
A. Allow the lung to expand ✓
B. Increase cardiac output

438-A nurse cares for a group of clients in a long-term care facility.


Which situation represents a situation in which the nurse supports the
client’s autonomy?
A. Patient refused to get I. V line
B. Doctor order for patient no CPR but patient insists to do CPR ✓

439- The doctor is allow woman to go in with her husband during CPR
resuscitation. What should the nurse tell her?
A. Try to stand away in seperate place to prevent stoping work because
the work at this time very fast ✓
B. Don't cry or move during CPR and don't try to share

440- When ventricular fibrillation occurs in a CCU, the first person


reaching the client should?
A. Administer oxygen
B. Defibrillate the client
C. Initiate CPR ✓
D. Administer sodium bicarbonate intravenously

441- You find a victim arrested in the street according to car accident you
found a clear fluid dropping from his ear, and you must give him rescues
breathing, what you will do?
A. Chest thrust
B. Jaw thrust ✓
C- Head tilt chin lift
D. Abdominal thrust

442- During infant CPR the nurse use two fingers but the nurse noticed
there is no progress?
A. Continue ✓
B. Use heel hand
C. Stop and giving breath

93
443- The nurse is doing CPR for patient. The nurse notes that patient
chest not raise properly (no proper recoil). What is the next action?
A. Squeeze oxygen bag harder
B. Change with another nurse ✓

DNR

444- Doctor writes DNR on a patient with end stage cancer, the patient
tells the nurse and express his feelings he wants to be resuscitation, what
is your nursing action?
A. Explain to the patient to except DNR
B. Ignore the patient feeling
C. Follow the hospital policy ✓
D. Report the doctor to Saudi Commission for Health Specialties

445- The doctor decided DNR for patient. The patient refused that. He
insisted to do CPR. What should the nurse do?
A. Continue to produce care and reduce pain ✓
B. Discontinue the treatment

446- For old man patient with poor prognoses the health care provider
decides to put him DNR, Patient have 3 children 2 of them agree with
DNR the 3rd one refused and ask to continue treatment. What the nurse
should do?
A. provide care and comfort
B. provide palliative care
C. Discuss with family regarding DNR and patient needs ✓

447- The doctor decided DNR for patient. The patient refused that. What
should the nurse do?
A. Discuss patient about DNR
B. Comfort and palliative care ✓
C. Prepare spiritual equipment

94
448- A nurse cares for a group of clients in a long-term care facility.
Which situation represents a situation in which the nurse supports the
client's autonomy?

A. A client falls and fractures a hip. The nurse contacts the health care.
provider for a prescription for pain medication prior to transfer for treatment

B. A client reports to the nurse regarding observing staff smoking on facility


grounds when it was banned for residents and family members.

C. A competent client who has received a terminal diagnosis request. The


nurse to not reveal the diagnosis to the family due to fear of them. seeking
long-term mechanical ventilation.

D. A client wishes to have a do not resuscitate (DNR) order to prevent


heroic measures by the health care team in the event of cardiac or
respiratory arrest. ✓

449- The doctor decided DNR for patient. The patient refused that. What
should the nurse do?
A. Follow hospital policy and respect his desire ✓
B. Give the care, continue treatment
C. Ignore the patient need
D. Stop all medication

Medication

450- A 62 year man with a history of intracerebral bleeding three months


ago, was referred primary health care following acute starch symptoms.
Brain CT scan is normal and the patient is receiving oxygen by nasal
cannula at 4LPM BP 185/105 HR 82 RR 18 TEM 36.6 SO2 93%.
Which of the following medication would be ordered first?
A. RtPA
B. Aspirin ✓
C. Nicardipine
D. Dopamine

95
451- Which of the following MARSA medication?
A. Vancomycin ✓
B. Gentamycin
C. Ampicillin
D. Streptomycin

452- A 33 year-old women with a history peptic ulcer disease presents to


the clinic with complains of bilateral knee and hip pain that worsens with
activity .The pain began one year ago and has been progressively
worsened. She now feels that she cannot take her daily walks and must
limit activity . The pain is absent at night and during rest .The nurse
counsels her regarding medication options . Which medication would be
most appropriate?
A. Aspirin
B. Ibuprofen
C. Naproxen
D. Acetaminophen ✓

453- A 3 year- old child with an elevated body temperature is


administered oral aspirin. The nurse records the body temperature of the
child two hours and finds that it is still elevated. What is the most likely
underlying physiology for the delayed response in action of the aspirin?
A. High gastric pH ✓
B. Thin epidermis
C. Low muscle tone
D. Short intestines

454- A 66- year- old woman with a history of unstable angina and
hypertension present to the emergency department with a dull chess pain
that she describes as similar to heartburn. The pain radiates down the
left arm. She had taken sublingual nitroglycerine tablets with any relief
any electrocardiograph is perform and shows elevated S T segments.
Which medication is most likely to given to dissolve the thrombus?
A. Heparin
B. Warfarin
C. Streptokinase ✓
D. Aspirin

96
455- Nitroglycerin administration route?
A. Intramuscular
B. Sublingual ✓
C. Subcutaneous
D. Oral

456- A nurse checks the medication chart of a patient, which has the
following order, nitroglycerin Tab sublingual. Which of the following
medication administration routes should the nurse use?
A. In the nose
B. In the eyelid
C. Under the skin
D. Under the tongue ✓

457- A nurse checks the medication chart of a part, which has the
following order; Nitroglycerin tab Buccal at stat. Which of the following
medication administration routes should the nurse use?
A. In the ear
B. Intravenous
C. Under the skin
D. Between cheek and gum ✓

458-The doctor ordered to give nitroglycerine sublingual to a patient with


angina. What is the most likely effect that drug has on the patient body?
A. Increase heart rate
B. Increase blood pressure
C. Increase blood supply to the heart ✓

459- What is the goal for giving Corticosteroids in Asthma?


A. Bronchodilator
B. Decrease airway swelling ✓
C. Chest clear and Remove secretions
D. Vasodilation

460- What is the side effects of Depakote ( Depakine )?


A. Nausea
B. Vomiting
C. Hair loss ✓
97
461- What should the nurse measure for patient on mannitol during
increase ICP?
A. Serum glucose
B. Serum electrolytes ✓

462- Patient admitted to medical ward. Which medicine prescription the


nurse should question?
A. Lasix 40mg oral BID
B. Codeine oral PRN ✓
C. Insulin S. C 50 IU
D. Ampicillin I. V 1000g

463- 76-year-old woman presents to the clinic with complaints of fatigue.


She feels her heart skips beats and becomes irregular with activities such
as climbing stairs and walking long distances. She has chest pain with
exertion that she rates at a level 5 on a 1-10 scale. The pain subsides with
rest. Her skin and nail beds appear pale. An assessment of gait shows
imbalance and she admits to episodes of numbness in the hands and feet.
The nurse suspects anemia and prepares a care plan. Which test would be
the most appropriate?
A. Schilling ✓
B. Folic acid levels
C. Lymph node biopsy
D. Bone marrow aspiration

464- A 62 year old woman presents to the clinic with a primary compliant
of fatigue. An assess shows a smooth and reddened tongue with a loss of
papillae and pallid mucous membranes. She complaints of a sense of
constant fullness of the stomach, with a decreased appetite and two three
loose bowel movement per day. A neurological assessment shows
numbness in the feet and lower legs. The patient type of test would the
most likely?
A. Schilling
B. Erythropoietin
C. Folic acid levels
D. Vitamin B12 levels ✓

98
465- Patient with TB going on Isoniazid medication. To prevent
peripheral Neuropathy .What should be included in care plan advice?
A. Instruct low protein diet
B. Avoid sun exposure too much
C. Provide vitamin B6 intake ✓
D. Increase fluid intake

466- Patient feeling hand numbness which medication should receive?


A. Vitamin C
B. Vitamin K
C. B12 ✓
D. Folic acid

467- Doctor write new order restrain (PRN) for Psychiatric patient that
he is overly aggressive. Which of the following indicate that the nurse
should do?
A. Call doctor while patient aggressive to do that by himself.
B. Apply the order one time only or if needed. ✓
C. Close patient room and do not apply restrain.
D. Express that is not acceptable and complains about your supervisor.

468- Physician orders an intravenous fluid of D5NS at 100cc/hr. This is


an example of which of the solution?
A. Hyper alimentation
B. Hypertonic ✓
C. Hypotonic
D. Isotonic

469- A mother of a patient who is on antipsychotic drug asked a doctor


about some more detail about the drug. The doctor told the mother that
her son is taking an atypical antipsychotic drug. Which of the following is
an examples of this type of drug?
A. Thioridazine
B. Clozapine ✓
C. Chlorpromazine
D. Haloperidol

99
470- A nurse cares for a client with an epidural catheter for labor pain
management and requests additional medication a student registered
nurse anesthetist (SRNA) responds and administers medication via the
epidural catheter. Which action performed by the SRNA Require the
nurse to stop administration of medication?
A. Aspirating bloody fluid into a syringe
B. Obtaining a negative aspiration into a syringe
C. Cleansing the port with alcohol for 15 seconds ✓
D. Assessing the client’s blood pressure

471 - A patient with a history of atrial fibrillation has an order for 0.25
milligrams of drug (X) the available supply/dose is 0.125milligrams.
Which of the following doses in correct?
A. Dispense two tablets to the patient ✓
B. Administer one tablet twice daily
C. Dispense one half tablet to the patient
D. Return tablets to the pharmacy and re-order

472- A patient with a history of atrial fibrillation has an order for 25


milligrams of drug (X), the available supply/dose is 0.25 milligrams.
Which of the following doses in correct?
A. Dispense two tablets to the patient
B. Administer one tablet twice daily
C. Dispense one half tablet to the patient
D. Return tablets to the pharmacy and re-order ✓

473- A nurse is preparing scheduled medications due at 6 pm. If a doctor


orders paracetamol tab 1g QID, and it was supplied from the pharmacy
in 250 mg tablets. Which of the following is the most appropriate nursing
actions?
A. Ask the pharmacy to provide 1g tablets
B. Call the doctor to recheck the dosage
C. Give the patient four 250 mg tablets ✓
D. Hold the medication and document in nursing notes

100
474- Soldier was brought to triage area after being exposed to chemical
weapons. Signs and symptoms of nerve gas exposure were noticed. A
nurse prepares for medical management. Which medication should the
nurse prepare for the patient?
A. Atropine ✓
B. Adrenaline
C. Sodium nitrate
D. Sodium thiosulphate

475- Which of the following types of iv fluid is considered hypertonic?


A. 0.33% Nacl solution
B. 5% Nacl solution ✓
C. Ringer's lactate solution
D. Ringer' solution

476- Patient scheduled to perform vena cava filter surgery. The doctor
ordered to stop medication before one day of operation for preoperative
preparation. Which of the following drug should be stopped?
A. Warfarin ✓
B. Sodium bicarb
C. Potassium

477- 13-years old patient is admitted for diarrhea and vomiting. He looks
pale and lethargic. A nurse is preparing to give IV hypotonic solution.
Blood pressure 110/70 mmHg Heart rate 76 /min Respiratory rate 18
/min Temperature 36.1°C . Which IV solution is most appropriate?
A. 0.9% saline
B. Lactated ringers
C. 10% dextrose in water
D. 0.45% sodium chloride ✓

478- Physician orders an intravenous fluid of D5NS at 100cc/hr. This is


an example of which of the solution?
A. Hyper alimentation
B. Hypertonic ✓
C. Hypotonic
D. Isotonic

101
479- Physician orders an intravenous fluid of D5 in water at 100cc/hr.
This is an example of which of the solution?
A. Hyper alimentation
B. Hypertonic
C. Hypotonic
D. Isotonic ✓

480- What is drug that prevent recurrence of rheumatic fever?


A-Penicillin ✓
B. Corticosteroids
C. Salicylate

481- A nurse enters the room of a patient named Ahmed Saeed to


administered medication but the patient not
wear his identification bracelet. Which of the following is the most
appropriate nursing action?
A. Ask the patient are you Ahmed Saeed?
B. Ask the patient what is your name ✓
C. Ask the patient hat mediation do you take

482- DM patient at night shift with cold skin, tachycardia, diaphoresis


what you will do first?
A. Check blood glucose.
B. Give him cup of orange juice ✓

483- Which of the following statement by the nurse describes the effect of
clomid as an ovulation inducing drug?
A. Given for the first 15 days in each cycle
B. Maximum dose is 50 mg daily for a month
C. It increases the risk of birth defects
D. It increase the risk of multiple pregnancies ✓

102
484- Clomiphene citrate (Clomid) is prescribed for a 32-year-old
infertility treatment. The nurse should understand that this medication is
used for following actions?
A. Induce ovulation ✓
B. Decrease prolactin level
C. Reduce endometriosis
D. Stimulate the release of Follicle-Stimulating Hormone

485- The nurse administered a dose of morphine sulfate as prescribed to a


patient who is in the post anesthesia care unit (PACU). The patient
appears to be resting comfortably, the respiratory rate is 8 and the O2
saturation is 21 oxygen via cannula is 86%. The nurse should
IMMEDIATELY administer?
A. Flumazenil ( Romazicon )
B. Midazolam (versed)
C. Naloxone (Narcan) ✓
D. Ondansetron (Zofran)

486- A nurse is caring for a patient who had Coronary Artery bypass
Graft Surgery (CABG) four hours ago. The nurse notices that the patient
has increased confusion and is restless. The patient reports nausea,
weakness and paresthesia in the extremities (see lab results)
Normal Values Result Test: 134-146 mmol/L145 Sodium
3.5-5.2 mmol/L 6.8 Potassium 2.15-2.62 mmol/L 2.50 Calcium.
Which of the following is the best medication?
A. Naloxone ( Narcan )
B. Hydralazine ( Apresoline )
C. Potassium chloride (KCI)
D. Sodium polystyrene sulfonate ( Kayexalate ) ✓

487- Digoxin not given to patient who has which of the following?
A. Tachycardia
B. Hypertension
C. Tachypnea
D. Bradycardia ✓

103
488- Atropine is indicated for increase?
A. Heart rate ✓
B. Respiratory rate
C. Increase Temperature

489- patient with ongoing magnesium sulfate the nurse should be alert to?
A. Temperature
B. Respiratory rate ✓
C. Heart rate

490- Morphine side effect?


A. Tachycardia
B. Hypertension
C. Tachypnea
D. Bradypnea ✓

491- A nurse is caring for a client with bipolar disorder, who is receiving
Lithium carbonate. Before Administration of the next dose, the client
complains of nausea and vomiting and the nurse finds that the client’s
lithium blood level 1.5mEq/L. Which of the following actions is
considered Apriority?
A. call the client’s physician immediately
B. withhold the next dose. ✓
C. Administer IV fluids

492- A patient took his NPH insulin at 6 Am when will he be


hypoglycemia?
A. 7 AM
B. 8 AM
C. 9 AM
D. 10 AM ✓

493- Child post appendectomy with severe pain. What is the most
appropriate nursing intervention?
A. Give pain medication as ordered ✓
B. Rest read book
C. Walk with parents
D. Talk with same group age child
104
494- MAOI drug connected to patient then develop severe nausea,
vomiting during infusion. What should the nurse do?
A. Stop infusion ✓
B Give thiamine
C. Notify physician

495- A patient was on a regular dose of lithium carbonate. The nurse


noticed he has hand tremor, polyuria, diarrhea and vomiting. What
immediate action should be taken by the nurse?
A. Diuretics
B. Withholding lithium ✓
C. Calling the psychiatrist
D. Monitoring serum lithium level

496- A patient was on a course of lithium carbonate drug. During the


nurse found that he complained from nystagmus visual hallucination, and
oliguria Which of the following drug related complications best
symptoms?
A. Overdose
B. Mild toxicity
C. Severe toxicity ✓
D. Moderate toxicity

497- A patient is being prepared for discharge following hip replacement


surgery. The nurse is providing him with discharge education. Which of
the following information should be taught to him as an effective pain
management principle?
A. Avoid giving pain medication prior to participating in physical therapy
B. Give a double dose of pain medication if pain is intolerable
C. Give pain medication before pain becomes severe ✓
D. Delay giving pain medication as long as possible.

105
498- A male client has received a prescription for orlistat for weight and
nutrition management. In addition to the medication, client states plans
to take a multivitamin. what teaching should a nurse provide?
A. Be sure to take the multivitamin and the medication at leasr
two hours apart ✓

B. As a nutritional supplement, orlistat contains all the recommended daily


vitamins and minerals

C. Multivitamins are contraindicated during treatment with weightcontrol


medications such as
orlistat

D. Following a well-balanced diet is a much healthier approach to good


nutrition than depending
on a multivitamin

499- A 45-year-old distressed and restless patient in the Psychiatric Ward


was unable to sleep during the nights for the last two days. He was
ordered sleep medication which was to be administered at 10 pm. At the
time medicine administration, the patient was found asleep in bed .
Which of the following action should the nurse take regarding the
dictation?
A. Discard medicine and cancel the order
B. Leave it at the bed side for the patient
C. Wake him up and administer
D. Hold, record and report ✓

500- A nurse is given health education to the parent how to reduce febrile
avulsion in the child at home. Which of the following medication in the
safest intervention?
A. Analgesic
B. Antipyretic ✓
C. Antibiotic
D. Antiemetic

106
501- A home care patient with chronic obstructive pulmonary disease
(COPD) reports an upset stomach. The patient is taking theophylline
(Theo-Dur) and triamcinolone acetonide (Azmacort ), the nurse should
instruct the patient to take ?
A. Theo-Dur on an empty stomach
B. Theo- Dur and Azmacort at the same time
C. Theo- Dur and Azmacort 12 hours apart
D. Theo-Dur with milk or crackers ✓

502- A 78-year-old woman who lives in a long-term care facility has been
having repeated episodes of urinary tract infections. She is prescribed
then amine mandelate one gram to be taken by mouth four times per
before meals and at bedtime. The nurse advises the patient that she need
to remove milk from the diet while taking the medication. What is the
primary purpose for this dietary advice ?
A. To prevent mal absorption of medication
B. Decrease risk of gastrointestinal upset
C. Reduces effectiveness of medication
D. To make the urine acidic ✓

503- A nurse received the serum digoxin level result for the patient the
day and notes that the result is : 2.6 ng/mL (see lab result) Test result
normal Value Digoxin (men) 2.6 0.8-2ng/mL . Which of following
nursing actions is the most important?
A. Notify the physician ✓
B. Check previous vital signs of patient
C. Record normal value on nursing note
D. Administer scheduled dose of medication

504- A nurse received the serum digoxin level result for the patient the
day and notes that the result is : 2.6 ng/mL (see lab result) Test result
normal
Value Digoxin (men) 2.6 0.8-2ng/mL . Which of following nursing actions
is the most important?
A. Notify the physician
B. Check previous vital signs of patient
C. Withhold next do ✓
D. Administer scheduled dose of medication
107
505- A patient is being admitted in medical unit and has orders for fluid
restriction of 1400ml / 24 hours. He is receiving IV medications in 50 ml
solutions QID. How much fluid should the caring nurse allocate for care
intake in 24 hours?
A. 400 ml
B. 800 ml
C. 1000 ml
D. 1200 ml ✓

506- A nurse is preparing an order of Amydramine 30 mg PO BD. The


available check is 15 mg /5 ml. What is the correct dosage of the
medication to be administered?
A. 5 ML
B. 10 ML ✓
C. 15 ML
D. 20 ML

507- A nurse is preparing a medication order of 100 mg IV stat. The


available hydrocortisone is 200mg per 5 ml distilled water for injection.
Which of the following is the correct administered?
A. 4.5 ML
B. 3.5 ML
C. 2.5 ML ✓
D. 1.5 ML

508- A nurse is preparing to administer an intravenous infusion of 2000


ml of Ringer's lactate over 12-hours. The administration set has a
chamber that delivers 15 drops per ml. How many drops per minute
should be administered?
A. 14
B. 24
C. 42 ✓
D. 56

108
509- Doctor order to give medication x 300mg and available is 900mg/6ml
how many ml the nurse will give?
A. 1.2
B. 2 ✓
C. 4
D. 6

510- A 56-year-old man was admitted with complaint working for three
days. The nurse is preparing to administer infusion of saline 1000 ml over
six hours. What is the hourly infusion rate that the nurse needs to
infusion therapy in mL/hour?
A. 155
B. 167 ✓
C. 190
D. 217

511- Doctor Order: Heparin 10000 units OD S/C Standard Solution


40000 units/ml How many ml would you administer?
A. 0,52 ml
B. 1 ml
C. 0,5. ml
D. 0,25. ml ✓

512- The doctor order 600 ml of drug during 10 hours. How many
minutes for 15 ml of the drug?
A. 15 ✓
B. 20
C. 25
D. 30

513- A Physician has ordered clindamycin phosphate 300 mg to be


administered intravenously the available stock was labelled. clindamycin
phosphate 900 mg in 6 ml what is the correct dose to be administered by
the nurse?
A. 2 ml ✓
B. 4 ml
C. 6 ml
D. 8 ml
109
514- A nurse check the medication chart of a patient, which has the
following order; Paracetamol 500 mg P.O. QID. How many times a day
should the nurse administer the medication?
A. Once
B. Twice
C. Three times
D. Four times ✓

515- The doctor order 600 ml of drug during 10 hours. How many
minutes for 15 ml of the drug?
A. 15 ✓
B. 20
C. 25
D. 30

516- Vitamin k dose in newborn?


A. 0. 25 mg
B. 0.5mg
C. 1mg ✓
D. 2mg

517- A nurse is assigned to care for a patient in medical ward who has a
physician order of 1500ml of normal saline 0.9% to be administered over
12 hour period. Which of the following is the infusion rate?
A. 75 ml/hour
B. 100ml/hour
C. 125ml/hour ✓
D. 150ml/hour

518-The following syringe contains 2 gm of morphine sulphate, how many


grams are there in 1 ml?

A. 0.5
B. 1 gm ✓
C. 2 gm
D. 4 gm

110
519- The following syringe contains morphine sulphate in a concentration
of 2 mg/cc, how much total morphine this syringe contains?

A. 0.5 mg
B. 1 mg
C. 2mg
D. 4 mg ✓

520- Doctor order to give 75ml per hour over 12hours of normal saline
how many ml total should be given?
A. 450ml
B. 600ml
C. 900 ml ✓
D. 1000 ml

521-The patient weight 67 kg. He is receiving drug 30mg /kg/24hr. How


many grams should the nurse give for 24 hr?
A.5 g
B. 1.5 g
C. 1 g
D. 2 g ✓

522- MR Ahmed admitted to ICU with congestive heart disease his vital
signs BP 110/60 HR 120. stroke volume 80 the nurse expected cardiac
output to be?
A. 180 Note:
B. 1700 Cardiac Output =
Heart Rate * Stroke Volume
C. 2400
120 * 80 = 9600
D. 9600 ✓

523- The doctor ordered medication 5000 mg. The nurse diluted in 250 ml
of D5N5 and modified dose on infusion pump to 20mg/hr. How many ml
should the nurse give per hour?
A. 5
B. 10 Note: 5000 /250 = 20
C. 15
D. 20 ✓

111
524- What is the drug to minimize intracranial pressure?
A. Warfarin
B. Morphine
C. Potassium
D. Dulcolax ✓

525- A 37year old female prevented to the Emergency room with


complain of symptomatic bradycardia appropriate nursing intervention
include establish intervention access what is the possible drug to be given
in the IV solution?
A. Digoxin
B. Atropine ✓
C. Anticoagulant
D. Calcium channel blocker

526- A 25 year old woman present to the emergency room with decrease
reflexes, hypoventilation, hypotension, and focus pupils, a family member
who is accompanying the patient has bottle of diazepam which the label
states was recently the family member also indicates that the patient has a
depression. What intervention should the nurse expect to administer?
A. Naloxone
B. Active charcoal ✓
C. Tap water enema
D. Magnesium sulfate to reduce the risk seizure

527- Cortisone administration time?


A. Early morning ✓
B. Afternoon
C. Before bed time

528- Hydrocortisone route of administration?


A. Ear
B. Eye
C. Nose ✓
D. Under the tongue

112
529- A nurse is about to perform venipuncture to initiate intravenous
therapy with 0.9% normal saline solution. Before venipuncture, the
nurse collects the require supplies and plans to Perform certain action.
Which of the following actions is the most important?
A. Apply a tourniquet below the site of venipuncture
B. Place an arm board at the joint above the venipuncture
C. Apply cool compress over the vein to be used for venipuncture
D. Inspect 0.9% normal saline solution date of expir ✓

530-A client was admitted in the Emergency Room due to mild metabolic
acidosis associated with dehydration and potassium the doctor
administered Hartmann’s (lactated Ringer’s) intravenous fluid and
electrolyte replacement. Which of the following elements of the lactated
Ringer’s solution highest value?
A. Calcium
B. Sodium ✓
C. Potassium
D. Magnesium

531- What should you do if I.V line infiltration happen?


A. Remove I.V line ✓
B. Cold compress
C. Slow fluid

532- An older adult client with renal failure comes to the emergency
department with a report of nausea and vomiting. The client's heart rate
is 45 beats/min. The nurse is most concerned about which medication that
the client takes?
A. Nitroglycerin
B. Digoxin ✓
C. Doxorubicin
D. Furosemide

113
533- Nurse gives health education to patient In outpatient department
regarding a medication. she told him "avoid working outdoors in a hot
weather, excessive sweating ad during l water daily Which of the
following is most likely medication?
A. Diazepam
B. Haloperidol
C. Lithium carbonate ✓
D. Monoamine oxidase inhibitor

534- During CPR of an admitted patient following the cardiac arrest.


Two Dc shocks and one dose of epinephrine was given. A third dose was
delivered followed by antiarrhythmic medication. What medication is the
most likely to be given next?
A. Adenosine
B. Lidocaine 2 mg
C. Epinephrine 1 mg ✓
D. Sodium bicarbonate

535-A patient with an allergy to cats arrives at the emergency department


with symptoms of pruritus, and difficulty in swallowing after visiting
some who has three cats. Addition to confirming airway patency, which
of the following is it most appropriate to prepare the patient for?
A. Intravenous line insertion
B. Intravenous glucocorticoid
C. Subcutaneous epinephrine ✓
D. Application of ice throat

536- A nurse is giving discharge planning instructions to the parents of a


one year old child acute otitis media which of the following discharge
instruction takes priority?
A. Administrate antibiotics as prescribed ✓
B. Administer influenza vaccination
C. Breast feeding as long as possible
D. Continue using of pacifier

114
537- A 4 years old girl, was playing outside, she came to her mom crying
and holding her right upper arm, she went to the hospital with swelling
over the upper arm, pain and itching, the appropriate management is?
A. Maintain patent airway
B. Administer S/C Epinephrine ✓
C. Prepare for intubation
D. Administer I.V Epinephrine

538- The nurse is caring for a 60-year-old in the Medical-Surgical Unit.


On entering the room the nurse finds the patient alert, oriented and. The
skin color appears healthy and pink. The nurse observes
electrocardiogram and notices absent P-waves. Which initial action is the
most important ?
A. Defibrillate
B. Cardio version
C. Pacemaker preparation
D. Intravenous amiodarone ✓

539- During health education for a patient on Monoamine oxidase


inhibitor (MAOI) the nurse instruct him to avoid certain foods that can
interact with the MAOI. Food containing with of the following contents
should be avoided?
A. Alcohol
B. Caffeine
C. Tyra mine ✓
D. Folic acid

540- A 45 year old client who diagnosed with brain was schedule for
craniotomy. It is important to preventing the developing of cerebral
edema after surgery. What medication would the nurse expect to
prescribed the client?
A. Steroids ✓
B. Diuretics
C. Ant convulsions
D. Antihypertensive

115
541- What is the side effect of Thyroxine?
A. Depression
B. Fever
C. Weight gain
D. Increase appetite ✓

542- Which of the following drug should the doctor give Instruction for
patient when using?
A. Antiemetic's
B. Steroids ✓
C. Antibiotic
D. Paracetamol

543- After teaching a 54 year old patient with angina on how to take
nitroglycerin sublingual PRN. Which of the following statements reflect
the patient understanding?
A. I have to take this medication once i need it only ✓
B. It is ok to take one tablet daily to prevent the heart attack
C. I can take two tablets together at once if the attack is
D. This medication will regulate my heart beats and I will be

544- When a patient was first diagnosed with schizophrenia, one of his
family members asked the nurse about the possible causes. The nurse
said that one reason is that he may have had an excess secretion of
neurotransmitter. Which of the following neurotransmitters?
A. Serotonin
B. Dopamine ✓
C. Glutamate
D. Endorphins

545- A patient who has pain localized in the chest, now down the left arm.
What is this type of pain called ?
A. Phantom
B. Regimented
C. Shooting
D. Radiating ✓

116
546- The doctor ordered morphine to patient. The nurse should assess
which of the following with the medication?
A. O2 saturation 95%
B. Respiratory rate 11 b/m ✓
C. Hear rate 65 b/m
D. Blood pressure 118/75

547- A 71 year-old woman who resides in a long-term nursing home fell


while walking down stairs. The attending nurse arrives to find the patient
sitting motionless on the stairs. She is alert and oriented but wishes to rest.
While she rests, the nurse reviews the chart and notes that her medication
regimen includes metformin, loratadine, warfarin and diclofenac. Which
medication is most likely to increase the patient's risk of injury ?
A. Metformin
B. Loratadine ✓
C. Warfarin
D. Diclofenac

548- A 3 year is brought by the mother to the emergency Department


with fever diarrhea and vomiting. She passed four loose motions and
three vomiting the last 24 hours , she is anorexic, irritable , has dry lips
and moderate skin turgor. She is given ORS to drink, but she refused it
after the first stip. O2 sat 96 HR 36 TEM 38.8 What is the immediate
nursing intervention is required to encourage the baby to drink the ORS?
A. Give In a cup once cold a day
B. Help her to drink with a syringe
C. Give in a small amount frequently ✓
D. Engage in playing and help to her drink it

549- Doctor order to give Ofloxacin Otic route the nurse understand that
medication will be given?
A. Nasal
B. Eye
C. Ear ✓

117
550- The nurse should assess which of the following tests before giving I.
V furosemide?
A. Bp , Bun , liver enzyme
B. Bp , Bun , potassium
C. Bp, HR , urine output ✓

551- Community health nurse is making preparation for crisis plan.


What is the most accurate information to make a disaster plan in
community?
A. Check stored water every year and change it if expired
B. Store bag with 3 gallons of water for each person for 3 days ( 1 gallon
per day for each person). ✓
C. Inform that animals don't need plan. It can survive
D. Prepare bag and Storing of food water and other stuff for 7 days

552- A 45 year old man is admitted to the neurosurgery ward for the
surgical elevation of depressed skull fracture. He has episodic severe
headache with seizure and is unable to concentrate. Which of the
following initial nursing problem needs more attention?
A. Disturbed coping and anger spells
B. Risk of injury to seizur ✓
C. Disturbed communication and irritability
D. Pain management and comfort measure

553- A nurse inserted an intravenous cannula into a patient's right


forearm and began to administer 500mg of erythromycin through a
pump delivery system. Following hospital protocol the nurse remains at
the bedside for the first 10minutes to assess the patient's response. After
five minutes of the infusion, there is a small hematoma at the cannula
insertion site. The intravenous injection flow a shower than expected
when the arm is elevated. The skin proximal to the catheter site is warm,
reddened and painful on palpation. Which of the following is the most
appropriate nursing action?
A. Apply ice pack for twenty minutes at site
B. Discontinue and re-establish a new site ✓
C. Increase infusion of the pump
D. Position right below level of heart

118
554- What is the side effects of isoniazid?
A. Hyperuricemia
B. Photosensitivity ✓
C. Peripheral neuritis and nerve inflammation
D. Blurred vision.

555- A nurse is preparing to administer 25 mg iron dextran inject patient


with iron deficiency anemia. the nurse knows this d to subcutaneous
tissue and wants to administer the drug safely which of the best
administration techniques?
A. Z-track ✓
B. Deep IM
C. c. use large gauge
D. insert needle at 45 angle

556- What is the Isoniazid side affect?


A. Hyperuricemia
B. blurred vision
C. Prephral neuritis ✓

557- A patient was on regular dose haloperidol. The nurse noticed that he
start developing side effects in the form stopped posture with shuffling
gait and pill-rolling movement of his hands. Which of the following is
most likely the
side effect?
A. Akathisia
B. Acute dystonia
C. Tardive dyskinesia
D. Pseudo parkinsonism ✓

558- A patient was brought to the emergency room because of a sudden


onset of difficulty in breathing. The result to a stat X ray revealed fluid
accumulation in the pleural space. The doctor order for an immediate
thoracentesis. Which of the following sites should be prepared ?
A. Between 5th and 6th
B. Between 6th and 7th
C. Between 7th and 8th ✓
D. Between 8th and
119
559- Patient is receiving chemotherapy. When should the nurse give
antiemetic to avoid side effect?
A. Before session ✓
B. During session
C. Half hour After session

560- The doctor ordered human chorionic gonadotropin hormone


for woman with uterine fibroid. What is the Side effect for HCG
hormone?
A. Depression ✓
B. Anorexia
C. Osteoarthritis
D. Menopause

561- Patient came to ER complained with headache after taking


Nitroglycerin. What should nurse do?
A. Explain that is Side effect of medication ✓
B. Go-to the pharmacy
C. Ignore the patient

Maternity

562- A woman was rushed to the emergency room because if vaginal


bleeding mild cramps, tenderness over the uterus, and closed cervix.
After the assessment, it was suspected that the patient may be lead to
inevitable abortion. What is the possible management of this patient?
A. Bed rest
B. Surgical management
C. Induction of oxytocin
D. Dilatation and curettage ✓

563- Ovarian cyst complications?


A. Amenorrhea
B. Dysmenorrhea
C. Infertility ✓

120
564- Polycystic ovary syndrome (PCOS) Complications?
A. Amenorrhea
B. Dysmenorrhea
C. Infertility ✓

565- Polycystic ovary syndrome (PCOS) Common Signs and symptoms?


A. Dysmenorrhea
B. Infertility ✓
C. Polymenorrhea

566- A 40 years old female is diagnosed with chlamydia with no signs and
symptoms?
A. Subclinical stage ✓
B. Clinical stage
C. Susceptibility

567- For which of the following issues should the nurse observe the movie
closely during the 4TH stage of labor?
A. Uterine irritability ✓
B. Signs of infection
C. Signs of bleeding
D. Unwillingness to breastfeed

568- What does the relationship of the site of the presenting part to the
location on maternal pelvis refer to?
A. Fetal lie
B. Fetal position ✓
C. Fetal presentation
D. Fetal attitude

569- 29-year-old man is in the Surgical Ward on his first post-open


thyroidectomy. He appears drowsy but he is. able to respiration by
nodding head. He is developing mild dy restlessness. What is the initial
recommended goal of care ?
A. Monitor vital signs of thyroid storm
B. Assess for bilateral vocal fold mobility ✓
C. Monitor for swelling on the neck.
D. Monitor for vocal cord paralysis
121
570- Patient has cystic fibrosis and the nurse wants to perform postural
drainage when is the best time for posture drainage?
A-Before meals
B-After meals after half an hour of meals ✓
C-Before sleeping
D. Afternoon

571- Female patient has been advised that laboratory tests confirm herpes
simplex virus (HSV), type 2. The nurse should teach the patient that a
Papanicolaou test (Pap smear) is recommended?
A. Every 6 months if symptoms persist despite treatment. ✓
B. Every year even if asymptomatic whenever symptoms recur.
C. Every 3 years if other Pap smears have been negative

572- Which of the following condition is a Contraindication for a woman


oral Contraceptives?
A. Dysmenorrhea
B. Menorrhagia
C. Thrombophlebitis ✓
D. Toxic shock syndrome

573- What causes spontaneous abortion in 12 gestational age?


A. Maternal stress
B. Infection
C. Fetal abnormalities ✓

574- A nurse is caring for a pregnant patient who is diagnosed with


abruption placenta. Which of the following findings of assessment would
indicate a concealed hemorrhage?
A. Maternal tachycardia
B. Decrease in fundal height
C. Rigid, board like abdomen ✓
D. Acceleration in fetal heart rate

122
575- A 62-year-old male patient, admitted in the surgical Ward is
scheduled for the surgical removal of polyps from his descending colon
under general anesthesia. he is experiencing fatigue, abdominal pain and
blood streaked. stools for a couple of months. he is worried whether the
bleeding in his stools is going to stop after surgery. What is most
appropriate response by the nurse for the patient concern?
A. Surgery often relieves the symptoms.
B. Let us have a detail discussion with your physician ✓
C. Your condition may or may not resolve, it depends.
D. In fact surgery is the only treatment for the problem.

576- In placenta Previa marginalia, the placenta is found at the ?


A. Internal cervical os partly covering the opening
B. External cervical os slightly covering the opening
C. Lower segment of the uterus with the edges near the internal cervical✓
D. Lower portion of the uterus completely covering the cervix

577- community health nurse visited a postnatal primigravida mother


who want back to work. The nurse instructed the woman the different
positions breast-feeding in order to lessen the burden of the mother at
night. Which of the following position will the nurse recommend for the
mother night ?
A. Cradle hold
B. Football hold
C. Reclining position
D. Side lying position ✓

578- A 26 years old patient in the gynecological ward complained of pain


and swelling in her episiotomy stitches on her second post-natal day. On
examination the localized swelling redness foul smell and pussy discharge
were identified at the stitches site after examination the nurse checked
vital sings BP 116/28 HR 132 RR 28 TEM 39.8C. Which of the following
should the nurse do prior to the patient examination by the gynecologist ?
A. Arrange dressing instrument
B. Maintain patient privacy ✓
C. Arrange stitch removal kit
D. Collect detailed history

123
579- A woman is 5 weeks gestation diagnosed with hyperemesis gar what
is the most important nursing action?
A. Psychological support
B. Folic acid supplementation
C. Strict intake and output record ✓
D. Providing the woman with a high protein diet

580- A pregnant women with varicose vein. What is the Nursing


intervention?
A. Elastic stock ✓
B. Elevation leg on pillow
C. Wear flat shoes ( not wearing heels )
D. Appropriate position while sitting with support back

581- A gravid 8 para 8 women has just delivered a 4.5Kg infant a


pregnancy. Which of the following is a possible complication?
A. Postpartum depression
B. Maternal hypoglycemia
C. Postpartum hemorrhage ✓
D. Pregnancy-induced hypertension

582- A 32 weeks pregnant patient present for her follow up appointment


in the antenatal clinic. She complains of experiencing frequent heartburn.
Which of the following is the most appropriate advice to the patient?
A. Drink plain water between meals
B. Raise head of the bed ✓
C. Eat favorite foods
D. Lie down for one hour after taking food

583- A nurse is preparing to administer Rhogam to Rh-negative women


who has delivered a Rh-positive newborn. Which of the following is
prevented by this intervention ?
A. Maternal illness
B. Neonatal illness
C. Production of antibodies ✓
D. Re- occurrence of Rh positive baby in next mother

124
584- In the 12th week of gestation, a client completely expels the products
of conception. Because the client is Rh-negative, the nurse must?
A. Administer RhoGam within 72 hours ✓
B. Make certain she receives RhoGAM on her first clinic visit
C. Not give Rhogam since it is not the birth of a stillborn
D. Make certain the client does not receive RhoGAM since the gestation only
lasted 12 weeks

585- A pregnant woman of 15 week gestation age and Rh-ve has an


abortion admitted in the ward what should the nurse do for this woman?
A. Administer Rheum within 72hour ✓
B. Do not give Rheum since it is not used with abortion
C. Do not give Rheum since the pregnancy is more than 12
D. Make certain she received Rheum on her first clinic visi

586- Rhogam is most often used to treat mothers that have a infant?
A. RH negative, RH negative
B. RH negative, RH positive ✓
C. RH positive, RH negative
D. RH positive

587- During a prenatal examination. The nurse draws blood from a


young Rh negative client and explain that an indirect Coombs test will be
performed to predict whether the fetus is at risk for?
A. Acute hemolytic disease ✓
B. Respiratory distress syndrome
C. Protein metabolic deficiency
D. Pathologic hyperbilirubinemia

588- A 20 weeks pregnant, primary gravid woman visits the antenatal has
sickle cell anemia trait and worried this disease transmitted to her baby
which of the following should be initial intervention?
A. Plan for the fetal genetic screening
B. Educate mother that her disease is inactive
C. Discuss the chances of genetic disease in the fetus
D. Gather data about the other family members having the disease ✓

125
589- A13-weeks-pregnant, multi gravida women is anxious and
apprehensive she has five children and is not willing to continue with this
pregnancy She is requesting the midwife to abort the fetus. She is under
weight malnourished and is over worked. BMI 17 Kg/m2 What
intervention is desired immediately?
A. Admission and intravenous line management for induction
B. Family planning and birth control measures
C. Dietary management and supplements
D. Support, reassurance and counselling ✓

590- A pregnant mother with sickle cell anemia is in labor. What should
the nurse to ensure the safety of the infant?
A. Control pain
B. Administer oxygen ✓
C. Monitor fetal heart rate
D. Monitor maternal vital signs

591- A 24 weeks-pregnant mother is in the Antenatal Clinic. She is upset


and crying about having her third baby girl as her family wants a boy.
She is requesting an abortion immediately before her family finds out
about the sex of the baby. Which of the following is the most appropriate
plan for the mother?
A. Calm her down and reassure for an appropriate solution
B. Provide moral support and book her for procedure
C. Repeat ultrasound and wait for a few more weeks
D. Family counselling and follow religious guidance ✓

592- A community Nursing nurse is planning to conduct prenatal


teaching and community assembly for pregnant adolescents. Which
teaching strategy would be most effective?
A. Offering open sessions for pregnant adolescents and anyone else who
wants to attend
B. Designing posters that girls can view individually in community Nursing
center
C. Preparing group class sessions for teaching pregnant adolescents
together ✓
D. Conducting one to one teaching sessions for both mothers and daughters

126
593- A primigravida woman who is pregnant at 30 weeks gestation told
the nurse that she is worried that anything happens to her baby. Which
of the following should be the proper nurse's response?
A. Ask the woman not to worry
B. Ask the woman to express her concerns ✓
C. Attract the woman's attention to other issue
D. Reassure the woman about the baby condition

594- Which of the following should be included in the nursing care to a


woman during 2nd stage of labor?
A. Shave the perineum
B. Administer enema to the woman
C. Careful evaluation of prenatal history
D. Watch breathing, bear down with each contortion ✓

595- woman who is 32 weeks gestation. Her weight was 66 kg last month
and today it is 78 kg. Which of the following is the best nursing action?
A. Assess the size of her fetus
B. Give health education good nutrition
C. Advise her to exercise and lose some weight
D. Check her blood pressure and test her urine for protein ✓

596- A nurse plans to teach agroup of 20 to 25 -year-old women about


oral contraceptives. The nurse should instruct that oral contraceptives
may?
A. Increase the risk of pelvic inflammatory disease
B. Cause acne to worsen
C. Decrease the risk of breast and cervical cancer
D. Decrease the risk of endometriosis ✓

597- Upon reviewing the pregnant client’s blood test results, the nurse
that traces of mercury are present even after the nurse had healthy
dietary modifications. Which action of the client does the nurse discuss to
reduce risk client?
A. Client has five soaked walnuts every day
B. Client consumes king mackerel very often ✓
C. Client eats one medium bowl of flax seeds daily
D. client has cooked soybean seeds as an evening snack
127
598- Postpartum patient was in labor for 30 hours and had ruptured
membranes for 24 hours. For which of the following would the nurse be
alert?
A. Endometritis ✓
B. Endometriosis
C. Salpingitis
D. Pelvic thrombophlebitis

599- A 34 years old woman was diagnosed with breast cancer and
underwent surgery. She is currently receiving monthly chemotherapy she
telephone the clinic and notifies the nurse that she developed a sore throat.
Which of the following foods would be the most appropriate to
recommended?
A. Fresh fruits and vegetables ✓
B. Seafood
C. Dried fruits nuts
D. pasteurized cheese

600- A full term mother presents in the antenatal clinic with mild lower
abdominal contraction and show watery discharge for her vagina. She
told the midwife that she may be starting her child birth. She is admitted
in the labor and delivery unit, support well and is lying down on the bed.
What knowledge guide is necessary as first intervention?
A. Assessment of cervix ✓
B. Medication and induction
C. Reassurance and support
D. Mode of delivery

601- A 17 year old mother after normal delivery have anemia and
bleeding. Which of the following education the nurse do?
A. Mass media
B. Group education
C. Individual education ✓

128
602- The nurse is receiving Post-operative patient from operating room
after general anesthesia to transfer him to the ward which of the
following is the priority nursing care for the patient?
A. Wear comfortable, loosely clothes
B. Ensure to keep side rails up ✓
C. Administration of morphine

603- A pregnant woman visits the Outpatient clinic complaining of


excessive vaginal secretion. Which of the following is the appropriate
nursing assessment?
A. Fetal heart rate
B. Fundal height
C. Signs of infection of labor ✓
D. Fetal presentations and position

604- To prevent and relieve fetal distress related to maternal hypotension.


Which of the position should apply?
A. Left side ✓
B. Right side
C. Semi fowler
D. Knee chest position

605- During A Vaginal Delivery of Woman 38 Years Old the Nurse


Should Consider the Risk of Which of The Following?
A. Acute bleeding and coma
B. Low potassium
C. Brain Injury
D. Fetal heart rate ✓

606- A 40 year-old female is diagnosed to have stress Incontinence , one of


nursing diagnosis stated by the nurse is "Stress incontinence related to
decrease pelvic muscle tone" What is the most appropriate nursing
intervention?
A. Apply adult diapers
B. Catheterize the patient
C. Teach Kegel exercises ✓
D. Initiate a bladder emptying program

129
607- A 34-weeks-pregnant mother experiences a sudden gush from her
vagina and mild uterine contractions. She informs about her condition
and requests if she could wait until the delivery. Which of the following is
the best desired response for report to the hospital?
A. Intravenous fluids and medicines need to be administer
B. Observation is necessary to identify premature labor ✓
C. Pain and fluid flow both need to be controlled
D. Fetal heart sound monitoring is necessary

608- The postpartum mother is breastfeeding newborn. The neonate took


5 Ounces. he mother is worry about the amount and She is asking the
nurse. What should the nurse reply?
A. That’s normal capacity ✓
B. The capacity for baby stomach same amount
C. That’s proper amount in first time

609- Vaginal assessment is contraindication in?


A. Placenta previa ✓
B. Placenta abruption
C. Hydatidiform mole
D. Ectopic pregnancy

610- A nurse in the postnatal ward is assigned for a multiparous patient


has just delivered a healthy newborn. When should the nurse plan to take
the patient vital signs?
A. Every hour for the first 2 hours
B. Every 30 minutes during the first hour.
C. Every 15 minutes during the first hour ✓
D. Every 5 minutes for the first 30 minutes

611- A post-operative nurse assesses the newly formed stoma and skin of
a patient who is one day postoperative following a proctocolectomy with
the formation of a permanent ileostomy. Which of the following clinical
findings would necessities immediate care?
A. Bright red and moist stoma
B. Dark red and purple skin ✓
C. Bloody liquid in pouch
D. Ulcerations with a rash
130
612- Pregnant woman 34 weeks has hypertension this problem is
identified as?
A. Hypertension
B. Pregnancy induced DM
C. PIH (Pregnancy induced hypertension) ✓
D. Placenta Previa

613- Which of the following is considered as the main maternal


postpartum hemorrhage complication?
A. Death ✓
B. Candidacies
C. Cervical cancer
D. Uterine prolapsed

614- A primigravida client at 25 weeks gestation visits the clinic and tells
the nurse that her lower back aches when she arrives home from work.
The nurse should suggest that the client perform?
A. Tailor sitting ✓
B. Leg lifting
C. Shoulder circling
D. Squatting exercises

615- Pregnant client is making her first Antepartum visit. She has a two
year old son born at 40 weeks. A 5 year old daughter born at 38 weeks
and 7 year old twin daughters born at 35 weeks. She had a spontaneous
abortion 3 years ago at 10 weeks. Using the GTPAL format the nurse
should identify that the client is?
A. G4 T3 P2 A1 L4
B. G5 T2 P2 A1 L4
C. G5 T2 P1 A1 L4 ✓
D. G4 T3 P1 A1 L4

131
616- A 17-years-old mother presented to the primary health after delivery.
She is suffering from fatigue, anemia, fever vaginal discharge (see lab
results) 81/50 mmHg Blood pressure , 98 /min Heart rate , 39.6 C
Temperature Result Test: 4.6 RBC 4.7-6.1 × 1012/L (Male) Normal
Values 4.2-5.4 ×1012 /L (Female) 88 Hb 130-170 g/L (Male) 120-160 g/L
(Female) 2.50 Calcium 2.15-2.62 mmol/L . Which of the following is
considered as the main maternal postpartum haemorrhage
complication ?
A. Death ✓
B. Candidacies
C. Cervical cancer
D. Uterine prolapsed

617- A 17-year-old mother presented to the primary health center ten


after delivery. She is suffering from fatigue, anemia, fever and vaginal
discharge (see lab results) Blood pressure 80/50 mmHg Heart rate 112
/min Respiratory rate 35 /min Temperature 39.6 C, Test Result
Normal Values RBC 4 4.7-6.1 × 1012 /L (male) 4.2-5.4 × 1012 /L (female)
Hb 90 130-170 g/L 120-160 g/L (female) HCT 0.29 0.42-0.52 (male) 0.37-
0.48 (female) WBC 12.8 4.5-10.5 × 109/L . Which of the following is the
best diagnosis of health problem in this case?
A. Severe urinary track infection
B. Vesicovaginal fistula
C. Puerperal sepsis ✓
D. Post-partum hemorrhage

618- A 17-year-old women went to the Emergency Department


complaining of fever, sore throat, and hoarseness of voice. Laboratory
tests were done (see lab results) Test Result Normal values
- HB 120.5 120-158 g/L - - HCT 0.39 0.36-0.47 g/L
- RBC 4.0 3.8-5.1 × 1022/L - WBC 14.2 4-10.5 × 109/L
Throat culture positive Beta Hemolytic Nonfound Streptococcus. Which
of the following. Would be the possible complication for the patient
condition?
A. Cellulitis
B. Bacterial cholangitis
C. Infective endocarditis
D. Rheumatic heart disease ✓
132
619- A gravida 3 para 2 presents to the Maternity Triage Unit after the
amniotic membranes ruptured at home. The fluid is noted to be clear.
The neonates head is engaged into the pelvis and the patient is having
contractions every 5 to 7 minutes. Each contraction lasts for 60-90
seconds. An examination of the cervix finds 4 centimeters dilatation and
90% effacement. She is uncomfortable during contractions and rates the
pain at a level 7, on pain scale of 1-10. Which of the following is most
indicative that she is in true labour to?
A. Level of pain.
B. Cervical dilatation and effacement ✓
C. Engagement of presenting part.
D. Frequency and length of contractions

620- A pregnant woman is 36 weeks pregnant was admitted to antenatal


ward for observation after being involved in a car accident the refuses
when the nurse tried to touch her abdomen to perform examination
saying it is painful. She also present mild vaginal bleeding. Which of the
following is the most appropriate diagnosis?
A. Placenta previa
B. Tubal pregnancy
C. Abruptio placenta ✓
D. Inevitable abortion

621- Which of the following is described as premature separation of a


normally implanted placenta during the second half of pregnancy.
Usually with severe hemorrhage?
A. Placenta Previa
B. Ectopic pregnancy
C. Incompetent cervix
D. Abruptio placenta ✓

622- Which of the following classifications of placenta previa is applicable


when the placental edge is 5 cm away from the internal cervical OS ?
A. Total
B. Partial
C. Marginal ✓
D. Complete

133
623- A 28-year-old pregnant woman at 9 weeks presents to the with
vaginal bleeding. During assessment, the nurse found height is 12cm.
Which of the following is the most likely diagnosis?
A. Placenta previa
B. Abruptio placenta
C. Ectopic pregnancy
D. Hydatidiform mole ✓

624- A 25 years old patient with history of amenorrhea for two month
was admitted for hydatidiform mole investigation. Which signs and
symptoms would the nurse observe?
A. Hypotension
B. Hyperglycemia
C. Rapid uterine growth ✓
D. Painful uterine contraction

625- A women was discharged from gynecological ward after gestational


trophoblastic disease molar pregnancy. Which of the following is the best
advice to give her?
A. Never to fall pregnant again
B. To request the doctor to sterilize her.
C. To consider having her uterus removed
D. To avoid falling pregnant for at least one year ✓

626- 30 years old women absent her menstruation for 5 months and her
menstrual cycle come every 28 days, she controlled her diet and do heavy
exercise this women condition?
A. Pregnancy
B. Primary amenorrhea
C. Secondary amenorrhea ✓

627- Mother 32 week is worry about her fetus and want to ensure her
fetus health is well being and growth. What is the most appropriate and
best test?
A. Measure fundal hight ✓
B. Ultrasonography
C. Non stress test
D. Abdominal girth
134
628- The Vitro fertilization is related to ( indications )?
A. Blockage of fallopian tube ✓
B. Donner of sperm
C. Absence of sperm
D. Immunologic hormone

629- Postpartum woman came to ER and complaining from fatigue. She


told nurse that "I am so exhausted, I am so worried, I am hoppeless and I
can't do anything. The mother diagnosed with postpartum blues. What is
the next question The nurse should ask her?
A. Are you blame yourself you are not be coping to motherhood ? ✓
B. Is there a family member you can talk with him?
C. Are you have any thoughts about harm yourself or harm your baby

630- A 30-year-old pregnant lady in her 33 weeks pregnancy. When the


nurse assesses the health condition of the lady, she provides nursing care
and health teaching. Which of the following types of home visiting is the
best to be conducted?
A. Selective
B. Follow up. ✓
C. Field trip.
D. Systemic

631- A Pregnant woman came to ER complain with sever pain in leg.


When the nurse assessed her, the nurse found that woman has varicose
veins. What should the nurse do?
A. Instruct her to Put pillow behind back while setting
B. Instruct her to put pillow under head and elevate the shoulder
C. Instruct her to wear elastic stocking with suitable size ✓

632- Postpartum mother is to be discharged on the second day of her


forceps delivery. She had sutures on her vaginal and perineum tears. She
is breastfeeding her baby and eating the special food provided by her
family. What discharge teaching needs more emphasis?
A. Diet management and exercise plan
B. Newborn care and vaccination records
C. Hygiene practices and alert signs to report ✓
D. Family planning and child growth monitoring
135
633- Pregnant woman 12 week after motor accident. She came to ER with
vaginal bleeding. What is the most appropriate Diagnosis?
A. Placenta previa
B. Inevitable abortion ✓
C. Abruptio placenta

634- A 30 year-old married female has dilatation and curettage as a


therapeutic abortion. A nurse was preparing for discharge instructions.
What should the nurse include in the discharge instructions?
A. Take high protein diet
B. Use tampons during swimming
C. Avoid sexual intercourse for two weeks ✓
D. Continue on bed rest for two weeks at home

635- Woman come to follow up in the third week after delivery. nurse
observe that uterus is slight palpable Perineal pad full soaked with blood.
Bp 100/60 temperature 39 what should nurse anticipate?
A. Perineal laceration
B. Retained placental fragment ✓

636- A 26-year old patient came for lithotripsy of her kidney. She was
accompanied by her husband. Her vital signs were stable and was
overweight. Her investigations released her pregnancy test as positive and
were two moths pregnant. Which of the following action the nurse should
take for this patient?
A. Prepare for physical examination
B. Hold procedure and inform ✓
C. Have the consent signs
D. Prepare for lithotripsy

637- A pregnant 33 week came to ER with sudden vaginal bleeding and


soft abdomen painless. What is the most appropriate Diagnosis?
A. Threatened Abortion
B. Abruptio placenta
C. Placenta previa ✓

136
638- A 66- year-old woman was admitted to the hospital with a history of
hypertension he presents with breathing difficulties that worsen with
activity and while sleeping. He is weak and feel that her heart misses
beats. An electrocardiogram confirms atrial fibrillation, right ventricular
hypertrophy and deviation towards the right. Which of the following is
the most likely origin of the disorder?
A. Hypertension ✓
B. Rheumatic fever
C. Atherosclerosis
D. Genetic predisposition

639- A pregnant woman in labor. She has sever abdominal pain extend to
symphysis pubis. Contraction frequency every 3 minutes and Duration 40
- 50 seconds. Cervical dilatation 3 cm. What is the most appropriate
action?
A. Encourage her to walk ✓
B. Give nitro oxide
C. Give analgesic

640- 19 years old woman needs urgent Cesarean section. The informed
consent should taken from which person?
A. Parents
B. Patient ✓
C. Husband
D. Patient and Husband

641-The nurse is performing a prenatal examination on a client in the


third trimester. The nurse begins an abdominal examination that
includes Leopold maneuvers. What information should the nurse be able
to determine after performing the assessment’s first maneuver?
A. Fetal descent
B. Placenta previa
C. Fetal lie and presentation ✓
D. Strength of uterine contractions

137
642- Which of the following is necessities for caesarean section?
A-Preterm labor
B-Severe preeclampsia
C-Total placenta previa ✓
D-Partial placenta previa

643- A pregnant woman had an accident and come to ER with brown


bleeding abdominal distension cramps. In abdomen fetus heart rate not
heard ?
A. Inevitable ✓
B. Threatened
C. Abruptio placenta

644- Mother has inevitable abortion. What should the nurse monitor?
A. Hemorrhage ✓
B. Uterine contractions

645- A pregnant mother at early pregnancy was admitted in Emergency


Room with leakage of amniotic fluid, vaginal bleeding and lower
abdominal cramping pain. What is the possible diagnosis should the
nurse suspected?
A. Missed
B. Inevitable ✓
C. Incomplete
D. Threatened

646- Mother with hyperthyroidism high risk for?


A. Preterm labor
B. Pre-eclampsia ✓
C. Hemorrhage
D. Congenital anomalies

647- Pregnant woman has convulsions associated with eclampsia which of


following drug is expect to administer?
A. Diazepam ✓
B. Midazolam
C. Magnesium sulfate

138
648- Newborn has diagnosed Development Dysplasia of hip DDH and is
using a Pavlik Harness as treatment?
A. Multiple
B. Acetabular dysplasia
C. Subluxation
D. Dislocation ✓

649- You have learned that in babies and children with developmental
dysplasia of the hip (DDH), the hip joint has not formed normally which
of the Following is the most common form of DDH?
A. acetabular dysplasia
B. dislocation
C. Preluxion
D. Subluxation ✓

650- A mother, who is planned for the labour induction, is started on


intravenous medication. She is in the first stage of her labour and is
having regular and increasingly stronger uterine contractions. Her cervix
is 1 cm dilated for the past few hours; both the mother and the baby are
being monitored. Which of the following signs should alert the midwife?
A. Baby's head not engaged
B. Decreasing heart rate of the baby ✓
C. Mother's blood pressure 110/60 mmHg
D. Mother's perspiration and increased thirst

651- When the bag of water ruptures, the nurse should watch for which
of the following risks?
A. Fetal distress
B. Cord prolapse ✓
C. Respiratory distress
D. Bleeding

652- A pregnant woman in active phase with cervical dilatation 3 cm. The
midwife induced labor by amniotomy. What is the next action?
A. insertion catheter to empty bladder
B. Assess contraction for full minute
C. Assess FHR at least full minute ✓

139
653- The community midwife has just finished an educational session
with a group of women regarding maternal health and annual
gynecological examination along with mammogram and Pap smear
testing. What outcome is the most desired?
A. Disease prevention.
B. Increased healthy livin
C. Early detection of related issues. ✓
D. Strengthened marital relationship

654- Surfactant secreted in lung at gestation age of ?


A. 22
B. 24 ✓
C. 26
D. 28

655- Which week lung (surfactant) will be mature?


A. 20 week
B. 24 week
C. 16 week
D. 28 week ✓

656- When does mature surfactant begin to be Produced or secreted?


A. 21 week
B. 24 week ✓
C. 28 week
D. 16 week

657- The size of uterus at the end of pregnancy:


A. 900 g ✓
B. 1000 g
C. 1500 g

658- The uterus size after delivery:


A. 900 g
B. 1000 g ✓
C. 1500 g

140
659- A nurse is performing an assessment of women who is delivery
which assessment finding would indicates a need to physician?
A. Hemoglobin of 11.0 g/dl
B. White blood cell count of 12,000
C. Fetal heart rate of 180 beats per minute ✓
D. Maternal pulse rate of 85 beats per minute

660- A nurse who is caring for a woman in labor, prepares to auscultate


fetal heart rate by using Doppler ultrasound device. How does the nurse
determine that the fetal heart sounds are correctly?
A. Notify if the heart rate is greater than 140 BPM
B. Placing the diaphragm of the Doppler on the woman abdomen
C. Palpating the maternal radial pulse while listening to the rate ✓
D. Performing Leopold's manoeuvres first to determine the fetal heart

661- Postpartum Client called to midwife after 10 days of discharge. She


told her that lochia become before 3 days brown. What is the most
appropriate action?
A. Informe her to return hospital to reassesment
B. Reassure the mother and tell her thats normal and lochia changes in
color from day to day ✓
C. Ask her about if she breastfeed her baby

662- What the important considerations that the nurse must checked
after the placenta is delivered?
A. Checked if the cord is long enough for baby
B. Check if the umbilical cord has 3 blood vessels
C. Check if the cord has a meaty portion and a shiny portion.
D. Check if the placenta is complete including the membranes ✓

663- During vaginal examination the nurse palpated the posterior


fontanel to be at the right side and upper quadrant of the maternal
pelvic?
A. ROP
B. LOP.
C. ROA ✓
D. LOA

141
664- During vaginal examination the nurse palpated the Anterior
fontanel to be at the right side and upper quadrant of the maternal
pelvic?
A. ROP ✓
B. LOP.
C. ROA
D. LOA

665- Which of the following of maternal age and other factors that affect
negatively on labor and cause severe complications?
A. Female 50 and above ✓
B. Female 16 or less
C. Hypertension
D. Anemia

666- What is the most common cause of fetal hypoxia?


A. fetal descent
B. Full Dilation of cervical
C. Contraction of uterine ✓
D. cervical effacement

667- Which of the following statement described the latent phase ?


A. First 3 cm of cervical dilatation ✓
B. From onset of labour to full dilatation
C. When the cervix is 100% effaced
D. Time of progress from 4 cm to 7 cm

668- Which of the following hormones are responsible for implantation of


ovum inside Uterine wall?
A. Progesterone and estrogen ✓
B. LH and. FSH Hormones
C. Oxytocin

142
669- Mastitis is an infection of the breast that occurs most often 2-4 after
child birth, which of the following is considered first line treatment of
mastitis?
A. Drainage of breast abscess
B. Antibiotic therapy and cessation of breast-feeding
C. Antibiotic therapy and continuation of breastfeeding ✓
D. Advise mother to stop breastfeeding until infection is clean.

670- Postpartum mother was complaining of mastitis on the Right breast,


and she told the nurse that it's safe feeding by my left side, What should
The nurse response?
A. Breastfeed from both sides it is safe ✓
B. The left side is only safe
C. The right side is only safe

671- Post-operative patient to be discharged and his temperature is 37.6


at 8:00 - everything else is normal what to do?
A. informs the physician to delay discharge.
B. gives aspirin dose earlier than scheduled.
C. temperature is normal because of the time. ✓
D. suspected infection

672- A nurse is providing instructions to a mother who has been


diagnosed with mastitis. Which of the following statements if made by the
mother indicates a need for further teaching?
A. I need to take antibiotics. And I should begin to feel better in 24-48 hours.
B. I can use analgesics to assist in alleviating some of the discomfort.
C. I need to wear a supportive bra to relieve the discomfort.
D. I need to stop breastfeeding until this condition resolves ✓

673- A nurse is giving health education for a mother who has mastitis.
Which of the following if stated by the mother about what she needs to do,
indicate the additional education is needed?
A. Take antibiotics
B. Use analgesics
C. Wear a supportive bra.
D. Stop breast-feeding. ✓

143
674- A client in the second stage of labor is unable to push and lacks to
bear down. What is the most appropriate next step?
A. Assess fetal descent ✓
B. Infuse intravenous fluids
C. Empty the client’s bladder
D. Administer oxygen to the mother

675- A 22-years old gravida 2 para 1 with gestational age 38 week


admitted to the hospital. The chief complaint is decreased the fetal non-
stress test revealed decreased variability and fetal movement. The next
morning as part of the antenatal, the nurse checks the fetal heart rate by
Doppler Sonic aid decreased the fetal heart rate to less than 100 /min.
which of the following action the nurse should do first?
A. Reassure the mother that the FHR is Ok.
B. immediately notify the physician or midwife. ✓
C. Reposition the patient to left lateral position.
D. Ask the mother about the pattern of fetal movement

676-Which hormones is responsible for uterine wall change and breast


enlargement during pregnancy, leucorrhea and hyperemesis. Which of
the following placental hormone is responsible?
A. Progesterone
B. Estrogen ✓
C. HCG

677- A primigravid client who is at 14 weeks gestation has been diagnosed


with hyperemesis gravidarum. The nurse explains to the client that the
condition is related to high levels of?
A. Testosterone
B. Estrogen ✓
C. Aldosterone
D. Progesterone

678- Pregnant 36 weeks gestational with irregular painless contraction


this signs called?
A. Hegar's sign
B. Braxton Hicks ✓
C. True labor
144
679- Causes of hemodilution in pregnancy?
A. Anemia
B. Increase plasma ✓
C. Increase WBCS
D. Decrease RBCS

680- A 3 Months Pregnant woman came with her husband to ER with


fatigue and confusion. Her husband inform the nurse that she doesn't
have any disease. The doctor order maternal drug screening test (drug
abuse) Test for Her. What should the nurse do before the Test?
A. Request for blood test
B. Get informed consent from husband ✓
C. Ask for medical history

681- Neonate borned with vernix caseosa. The nurse should identify it
protects baby from What?
A. From injuries and diseases
B. From trauma during delivery or infection ✓

682- What is the food should be limited during pregnancy?


A. Raw meat ✓
B. Cocked meat
C. Cooked fish
D. Well wash vegetables

683- Which dietary intake should be initiated in pregnant woman?


A. Yogurt
B. Soft cheese
C. Processed cheese ✓
D. Pasteurized

684- What is the food that a pregnant woman should not eat because it
can lead to infection for her?
A. Pasteurized milk
B. Unprocessed cheese ✓
C. Processed cheese
D. Yogurt

145
685- What is the food should be limited during pregnancy?
A. Pasteurized Milk
B. Processed Cheese
C. Soft cheese ✓
D. Yogurt

686- Mother has 8 months child wants to give her baby an egg to eat what
is the kind of egg she should give?
A. Give whole egg
B. Don’t give for child until 1 year
C. Give white egg without yolk
D. Give yolk egg without white ✓

687- Mother of 10 month old baby is complaining of breast engorgement.


What should the pt avoid in the dietary recommendation for baby?
A. Increase fluid intake
B. Start Weaning
C. Give baby cows milk ✓

688- The nurse is teaching a client diagnosed with Raynaud phenomenon


about how to avoid recurrent episode which instruction should be
included?
A. Warm bath
B. Aspirin
C. Wear gloves if handling cold objectives
D. Avoid Cold temperature ✓

689- Woman has experienced recurrent vaginal infection and she is


asking the nurse how to prevent the infection. What is the response if she
say the nurse expect she need further instructions and consider alter
preventive measures that kill good bacteria?
A. Wear cotton untiat underwear
B. Wipes front to back each toilet
C. Wear condom to prevent infection ✓
D. Avoid douch and perfume spray

146
690- Woman has recurred vaginal infection after prescribing treatment
by the doctor she asked the nurse about how she could prevent infection.
Which of the following is most appropriate answer to prevent infection
and kill normal flora?
A. stop vaginal doush and perfume sprays ✓
B. wear cotton underwear
C. wash from back to front
D. Wear condom to prevent infection

691- A nurse was educating a group of woman on prevention of infection.


The nurse asked each woman to state one preventive of vaginal infection.
Which woman needs more education?
A. First woman "keep vaginal area clean & dry
B. Second woman "wear cotton under wear"
C. Third woman "wipe from front to back after urination
D. Fourth woman "Do vaginal douche twice a day ✓

692- A 30-year-old pregnant lady in her 33 weeks' pregnancy. When the


nurse assesses the health condition of the lady, she provides nursing care
and health teaching. Which of the following types of home visiting is the
best to be conducted?
A. Follow up ✓
B. Selective
C. Systematic routine

693- A couple asked the nurse which of the first investigation they should
do for infertility. Which of the following should be the proper a nurse
answer?
A. Hysterosalpingogram
B. Serum progesterone
C. Semen analysis ✓
D. Endometrial biopsy

694- Postpartum Mother 2 days after delivery her baby Blood collection
from heel is done to PKU. She has appointment next day for
circumcision . What is the most important attention for circumcision?
A. Avoid tighted diapper ✓
B. Notify first day of first voiding urination
147
695- A nurse is educating a primigravida woman who is pregnant at 30
weeks on breast feeding. Which of the following statement by the woman
indicates that she needs additional teaching?
A. Breast milk can stored at room temperature ✓
B. Breast feeding should be based on baby demand
C. Baby can beheld in different ways during feeding
D. Breast feeding helps the uterus to return to prepregnancy size

696- A nurse is educating a primigravida woman who is pregnant at


30weeks on breast feeding. Which of the following statement by the
woman indicates that she needs additional teaching?
A. Breast milk can stored at room temperature
B. Breast feeding should be based on baby demand
C. Baby can beheld in different ways during feeding
D. Breast feeding helps the uterus to return to pregnancy size ✓

697- After 3 days of breast feeding a post partial patient reports nipple
soreness. To relieve her discomfort the nurse should suggest that she?
A. lubricate her nipples with expressed milk before feeding. ✓
B. dry her nipples with soft towel after feeding.
C. Apply warm compresses to her nipples just before feeding
D. Apply soap directly to her nipples, and then rinse.

698- Which test should be performed to screen for cervical neoplasia


during antenatal assessment?
A. Papanicolau (PAP test) ✓
B. Vaginal rectal culture
C. Rapid plasma regain test (PPR)
D. Venereal disease research laboratory test (VDRL)

699- Female patient has been advised that laboratory tests confirm
genital warts and recurrent many times. The nurse should teach the
patient that a Papanicolaou test (Pap smear) is recommended?
A. Once Every 6 months ✓
B. Once Every year
C. Once Every 3 years

148
700- A women should get their (pap smear test) each :
A. 6 month
B. 1 year
C. 3 years ✓
D. 5 years

701- The nurse is giving health education about copper IUCD


disadvantages. Which of the following is consider disadvantage for
IUCD?
A. Cause Iron deficiency anemia
B. Short term cost
C. Painful Insertion ✓

702- Side effect of HCG ( human chorionic gonadotropin ) during


pregnancy?
A. Anorexia
B. Osteoarthritis
C. Menopause
D. Depression ✓

703- What is the action of combined oral contraceptives?


A. Induce thinning of endometrium and prevent ovulation ✓
B. Decrease the production of luteinizing hormone and follicle and prevent
ovulation.

704- A 45 year-old woman is receiving chemotherapy for breast cancer.


Two weeks after the initial treatment she telephoned the nurse at the
cancer center and reports, she has hair loss, nausea, tiredness, a body
temperature is 38.1 C, and air hunger. Which finding most likely
indicates she needs to report the clinic?
A. Pyrexia
B. Nausea
C. Hair loss
D. Air hunger ✓

149
705- A postpartum woman who was admitted for 24 hours, community
nurse visited her after 2 days of discharge. Which of these are abnormal
findings?
A. Frequent urination.
B. Lochia serosa.
C. Uterus below umbilical level.
D. Both breast full of milk. ✓

706- A postpartum woman who was admitted for 24 hours, community


nurse visited her after 2 days of discharge. Which of these are abnormal
findings?
A. Frequent urination.
B. Lochia serosa.
C. Uterus below symphysis pupils. ✓
D. Both breast full of milk.

707- Multiparous woman is admitted to postpartum ward after vaginal


delivery. Assessment showed, lochia: steady trickle of bright red blood
and fundus firm. Blood pressure 110/70 mmHg Heart rate 80 /min
Respiratory rate 20 /min Temperature 37.5 C . Which of the following is
the most likely diagnosis?
A. Endometritis
B. Uterine atony
C. Vulvar hematoma
D. Laceration of the genital tract ✓

708-The Postpartum mother was asking the nurse about timing for
restating sexual intercourse activity. What should the nurse response?
A. 3 weeks after delivery
B. As long as taking contraceptives
C. After stop of lochia discharges ✓
D. Any time she wants

150
709- Which of the following assessment findings indicates laceration of
the canal in the fourth stage of labor?
A. Red-brown lochia
B. Firm contracted uterus
C. Fundus is palpated at the level of the umbilicus
D. More than 1 saturated perineal pad per hour ✓

710- A mother is the outpatient Clinic for her first post-natal visit on the
15th day her normal vaginal delivery. Her physical examination reveals a
stable condition, breasts are soft and her sanitary napkin has bright
colored rubra. Which of the following needs further evaluation?
A. Amount and frequency of breast-feeding
B. Hydration level and bleeding breast-feeding
C. Activity, exercise and resting periods
D. Uterine size and position ✓

711- Which of the following would the nurse included in her discharge
education for a postpartum patient regarding signs and symptoms that
should be reported immediately?
A. Lochia rubra persists ✓
B. Nipples become red
C. Lochia decreases in amount
D. After pains increase with breast feedin

712- Which type of lochia should the nurse expect to find in a client 2
days PP?
A. Foul-smelling
B. Lochia serosa
C. Lochia alba
D. Lochia rubra ✓

713- Lochia red sometimes happens during postpartum to bleed heavily


with Foul smell for the first three to ten postpartum days, the nurse
should expect that may to?
A. Lochia pink
B. Sign of abnormal hemoglobin
C. Infection ✓
D. Bleeding

151
714- Alba normally disappears after how many days postpartum?
A. 5 days
B. 7-10 days ✓
C. 18-21 days
D. 28-30 days

715- Lochia red fleshy odor mean?


A. Normal ✓
B. Infection
C. Bleeding

716- A nurse is caring is caring for a day 1 postpartum patient assessment


revealed in addition to red lochia with fleshy odor Blood pressure 110/70
temperature 38 respiratory rate 20 heart rate 90 Which of the following
would be the proper nursing interpretation of these findings?
A. Normal ✓
B. Infection
C. Dehydration
D. Hemorrhage

717- A nurse is providing postpartum care for a GSP4 mother who had a
rapid labor of an infant weighing 4000 gm. Assessment revealed a boggy
uterus, heavy lochia and stable vital signs. After fundal massage and
bladder evacuation. the fundus remains soft. which of the following is the
most appropriate next nursing action?
A. Inform the physician. ✓
B. Reassess the vital signs
C. Continue fundal massage
D. Take venous blood sample

718- pregnant women G1 P0 vaginal delivery observed in the second


postpartum day that the perineal pad saturated with bright red lochia
rubra what is the priority nursing intervention?
A. Massage fundus
B. Obtain vital signs
C. Inform physician
D. Inquire about time of pervious saturated perineal pads ✓

152
719- A nurse is caring for child who is post tonsillectomy and
adenoidectomy. The nurse should plan to assess which of the following
complication?
A. Pulmonary hypertension
B. Hemorrhage ✓
C. Hearing loss
D. Orthopnea

720- A 40 year-old woman present with painful leg cramps and muscle
twitching, which wakens her at night. Her menstrual cycles are irregular
and cause painful abdominal cramping. The hair, skin and nails appear
dry and brittle (see lab results) Test Result Normal Values Calcium 2.09 2.
15-2.62mmollL Phosphate, inorganic 1.69 0.82- 1.51mmollL Magnesium
0.55 0.7-1.0mmollL what type of lifelong diet would be most beneficial for
this patient?
A. Increased magnesium and vitamin C.
B. Increased calcium and vitamin D ✓
C. Increased dairy and iron
D. High protein and high calorie
721- Which of the following consider contraindication of tonsillectomy?
A. Child with adenoid infection more than 4 times per year
B. Child with tonsillitis more than 4 times per year
C. Child age less than three years old ✓
D. Child with hypertrophied adenoids

722- The nurse is receiving a child postoperative tonsillectomy. Which of


the following nursing action is suitable for the postoperative care?
A. Encourage the child to cough spontaneously
B. Observe for subtle signs of hemorrhage
C. Place the child in the prone position ✓
D. Suction the mouth to clear the airway Because of violence

153
723- The nurse is receiving a child postoperative tonsillectomy. Which of
the following nursing assessment is suitable for the postop care?
A. Encourage the child to cough spontaneously
B. Observe for subtle signs of hemorrhage ✓
C. Place the child in the prone position
D. Suction the mouth to clear the airway Because of violent behavior

724- After tonsillectomy, a child begins to vomit bright red blood .the
Initial nursing action is to?
A. Notify the physician
B. Turn the child to the side ✓
C. Maintain an NPO status
D. Administer the prescribed antiemetic

725-The nurse is caring for a 5 years old boy in the postoperative unit
after he underwent tonsillectomy. The child’s gag reflex is intact and he
lies in the side lying supine position. The nurse notices that he is
repeatedly swallowing. He rates his pain at a level 8 using the smiling face
pain scale for kids . Blood pressure 90/50 mmHg Heart rate 150/min
Respiratory rate 26/min Temperature 37.1C What is the most
appropriate initial nursing response?
A. Notify the doctor ✓
B. Elevate the head of bed
C. Administer analgesic
D. Administer infusion bolus

726- 22 week pregnant women diagnosed with molar pregnancy. The


nurse should expect On ultrasound to see which of the following?
A. Empty of gestational sac
B. Abnormal fetus
C. Grape like cluster ✓

727- The nurse cares for a client who has undergone a tonsillectomy. The
nurse is most concerned about which postoperative finding?
A. Lack of appetite
B. Throat pain
C. Frequent swallowing ✓
D. Nausea
154
728- A 6 years old pre-school returned from the operating room after
tonsillectomy. A nurse received the patient from the recovery room.
Which sign the nurse should assess first for bleeding?
A. Frequent swallowing ✓
B. Decreased pulse rate
C. Complaints of discomfort
D. Elevation of blood pressure

729- Patient post surgery of ovarian removal have rapid increase in


abdominal size and complain of increased pain what the complication
developed?
A. Pneumonia
B. Expulsions of surgical content
C. Internal hemorrhage ✓

730- A 16 year-old boy is in the Post-Operative Care Unit two hours


after a tonsillectomy. He is alert and oriented but complains of
severe throat pain and difficulty swallowing. He rates the pain at a
level 7, on a scale of 1-10. The urine output from the folly catheter
is 45 ml over the past two hours. Blood pressure 130/74 mmHg
Heart rate 64/min Respiratory rate 18/min ,Oxygen saturation 98 % on
room ai, Which clinical finding is most important to report to the
doctor ?
A. Oxygen saturation
B. Difficulty swallowing
C. Urinary output ✓
D. Pain level

731- The nurse is preparing to discharge an 8 months old recovering from


gastroenteritis. Which of the following topics the nurse should include in
session?
A. Weaning
B. Toilet training
C. Accident prevention
D. Dietary requirements ✓

155
732- How many times shower the baby?
A. twice a week with warm water ✓
B. Every day with warm water
C. Every day with warm water and acidic soap
D. Every day with warm water and alkaline soap

733- How many times shower the baby?


A. Twice a week with warm water
B. Every day with warm water
C. Every day with warm water and acidic soap
D. Twice a week with warm water and alkaline soap ✓

734- Neonatal mortality rate?


A. 1:100
B. 1:1000 ✓
C. 1:100000
D.1:1000000

735- A nurse is assessing a 2 days old full-term male neonate circumcision.


She observed that the circumcised area is re a large amount of fresh
blood. Heart rate 110 /min Respiratory rate 40 /min Temperature 36.6 C.
Which of the following action should the nurse take?
A. Apply antibiotic ointment on the affected area
B. Give the infant another injection of vitamin K
C. Clean the area with betadine to prevent infection
D. Apply gentle pressure with a sterile gauze ✓

736- A nurse is performing physical examination on the new born she


notes that the baby has cephalohematoma this baby is risk to develop
which of the following?
A. Sudden death
B. Pathological jaundice ✓
C. Infected umbilical cord
D. Increased intracranial pressure

156
737- A 3 months baby diagnosed with Hirsch sprung disease what is the
disease?
A. Symptoms occur after 6 months
B. Affect both small and large intestine
C. Absences of prestalisis movement in the distal part of large
intestine /colon✓
D. Telescoping of the intestine

738- A newborn has small, whitish, pinpoint spots over the nose, which
the nurse knows are caused by retained sebaceous secretions. When
charting this observation, the nurse identifies it as?
A. Milia ✓
B. Lanugo
C. Whiteheads
D. Mongolian spots

739- The nurse is aware that the age at which the anterior fontanelle
closes is months?
A. 20 to 24
B. 16 to 18 ✓
C. 6 to 10
D. 10 to 12

740- A 5-month-old boy has been vomiting green coloured vomit He has
intermittent abdominal pain during which he draws his chest, turns pale
and cries forcefully. On observation, the in the stool which has a jelly-like
consistency. Abdominal pal a long. tube-like mass. There is no fever, rash
nor diarrhea are hyperactive in all quadrants. Which is the most likely
form of initial treatment?
A. Manual manipulation
B. Surgical resection
C. Normal saline enema ✓

157
741- Mother has child with Spastic cerebral palsy. The mother was asking
the nurse when her child be adult he will have mental retardation or
impairment. What should the nurse reply?
A. Who has cerebral palsy get more than 70% in IQ .
B. Who has epilepsy and CP Limited for them to have mental problem
C. Who has CP the main cause genetic factors .
D. Who has CP most of them has cognitive impairement with some
mental retardation ✓

742- A 5 year old child was bought to the E room with a fractured right
forearm. He had several bruises on showed no signs of pain while
palpating them. He seemed scared and did not answer any questions
asked. Which of the areas should the nurse focus on to utilize her critical
thinking approach?
A. Physical abuse ✓
B. Child’s schooling
C. Developmental milestones
D. Mother child relationship

743- Child with VSD. What is blood flow characteristic?


A. mixes atrium blood
B .decreased pulmonary blood flow
C. Increase pulmonary blood flow ✓

744- What is the benefit of skin to skin after delivery?


A. control mother and baby temperature
B. initate and facilitate early breast feeding during 3rd stage of labor ✓
C. improve circulation of baby

745- A nurse in the newborn nursery is assessing a 2-hour old newborn.


She observed that the newborn has a caput succedaneum. Which
intervention has the highest nursing priority?
A. Activate code blue immediately
B. Turning on the radiant warmer
C. Do nothing, it will resolve in 3-4 days ✓
D. Administer oxygen by nasal cannula

158
746- A 6 months old infant mother decided to wean her child. Which of
the following is the best principle of weaning process?
A. Start the weaning process by 8 month of life
B. Gradually replace one breast session at a time ✓
C. Discontinues the nighttime feeding first
D. Allow the child to take a bottle of milk or juice bed

747- A new-born has a diagnosed Developmental Dysplasia of the Hip


(DDH) and is using a Pavlik Harness as treatment. Which of the following
mechanical factors is associated with DDH?
A. Intrauterine breech position ✓
B. Caesarean section
C. Small infant size
D. Single fetus

748- A nurse performing nursing care plan for a neonate after a birth,
which intervention has the highest nursing
priority neonate?
A. Obtained a dextrostix
B. Give the initial bath
C. Give the vitamin k injection
D. Cover the neonate head with a cap ✓

749- A healthy baby is born normally via vaginal delivery and when
transferred to newborn until the nurse administered vitamin K
intramuscularly. Which sits recommended for vitamin K injection?
A. Biceps
B. Deltoid
C. Vastus lateralis ✓
D. Gluteus maximus

750- A nurse prepares to administer a vitamin K injection to a full term


the mother wants to know the importance of the injection Which of the
following is the best nurse response to the mother?
A. Needed for blood clotting to prevent hemorrhage ✓
B. Accelerate the growth and development of infants
C. Help in maintain healthy gut and passage of meconium
D. Protect the infant from developing sever respiratory distress
159
751- A 2-year-old child is admitted to the pediatric unit with a diagnosed
pneumonia. Which of the following intervention would be a nursing
priority?
A. Encourage coughing
B. Encourage exercise
C. Perform postural drainage ✓
D. Avoid food high in carbohydrate

752- Child with Attention deficit hyperactivity disorder (ADHD). The


parent complains that child hyperactive and too much moving. What is
your action?
A. parent to provide private school for child.
B. Instruct parent to provide normal school for child
C. Instruct parent to allow child to share in activities that need more
movement ✓

753- What is the first assessment for neonate after cutting umbilical
cord?
A. Cover baby with towel and keep warming
B. Encourage early contact with mother
C. Check baby from cyanosis
D. Check score assessment ✓

754- The scale used by a nurse to assess the gestational age of a newborn
is?
A. Bishop score
B. Ballard score ✓
C. Bel ward score
D. Apgar score

755- Baby after delivery has hypoglycemia what the nurse should do?
A. Insulin
B. Ask mother to give breast feeding ✓
C. Iv dextrose 5%

160
756- A newborn is delivered by the midwife. The umbilical cord i cut
safely by following necessary aseptic techniques. Furth newborn is to be
taken. Which of the following intervention is the most desired?
A. Assess sucking response
B. Increase mother child bonding
C. Assess and record APGAR score
D. Keep dry and maintain thermoregulation ✓

757- The neonate is delivered from diabetic mother gestational. He is high


risk for hypoglycemia. What should the nurse do to avoid it?
A. Give insulin
B. Oral feeding ✓
C. Nothing to do

758- Which mother risk to have baby with cognitive impairment: Mother
with?
A. WBCs in urine
B. Albumin in urine
C. Folic acid deficit ✓

759- An infant was admitted with patent Ductus arteriosus. Indomethacin


0.2mg/kg with prostaglandin inhibitor via intravenous route was
prescribed for him. Which of the following is the function of
indomethacin?
A. Assist closure of ductus arteriosus ✓
B. Prevent infection
C. Improve tissue perfusion
D. Preserve hormone level

760- Infant borned with normal delivery. After 2 months during follow
up visit, the nurse has observed the infant has one thigh shorter than the
second one. The nurse performed Barlow test and the affected leg turn
outward. What should the nurse expect?
A. Fracture
B. Dislocation ✓

161
761- Child with type 1 diabetes mellitus. The nurse is instructing
mother. Which of the following signs should be alert?
A. Polyuria
B. Vomiting
C. Dizzines ✓
D. Thirsty

762- Child 2 years old has acute otitis media. What is the sign while the
nurse observing child, confirm the diagnosis?
A. Otorrhea
B. Roll head side to side ✓

763- Why is better for preventing use of powder for children after
bathing?
A. Respiratory problems ✓
B. Itching skin
C. Diarrhea
D. Skin dryness

764- PKU screen test for what?


A. Metabolic disorder ✓
B. Blood disorder
C. Glucose

765- New born stomach capacity?


A. 6ml ✓
B. 12ml
C. 28ml

766- Which of the Following Is the Most Commend Site to Obtain a


Capillary blood sugar sample from neonate?
A. Earl ape
B. fingertip
C. Heel ✓
D. abdomen

162
767- A newborn with hyperbilirubinemia was started on phototherapy
What will be the nurse's instruction regarding feeding?
A. Feed glucose drinks
B. Breastfeed two hourly ✓
C. Bottle feed till the bilirubin level reduce
D. Breastfeed alternatively with bottle feeds

768- Evidence based practice in maternity recommended by WHO?


A. Supine position
B. Episiotomy
C. Amniotomy
D. Active management during third stage ✓

769- how to maintain patient free of infection during labor?


A. Use clean technique
B. Change pad
C. Limit vaginal examination ✓

163
770- A baby born at 38 weeks of gestational with birth weight 1800gram.
Which of the following is the classification of this infant?
A. Low birth weight
B. Very low birth weight
C. Appropriate for gestational age
D. Small for gestational age ✓

771- A baby born at 29 weeks of gestation with birth weight 950 gram.
which of the following is the classification of this infant?
A. Low birth weight
B. Very low birth weight
C. Appropriate for gestational age
D. Small for gestational age ✓

164
Intrauterine growth chart ‫حساب‬
large for
1- Post term Intrauterine growth chart ‫إذا كانت نسبة‬ gestational age
‫ و اكبر‬%90 ‫من‬ (LGA)

Appropriate
2- Fall term Intrauterine growth chart ‫إذا كانت نسبة‬ for gestational
%90 ‫ إلى أقل من‬%10 ‫فوق‬ age ( AGA)

Small for
3- Pre term Intrauterine growth chart ‫إذا كانت نسبة‬ gestational Age
‫ و أقل‬%10 ‫من‬ (SGA)

772- A newborn born by elective caesarian section under general


anesthesia term 28 weeks of pregnancy. His weight is 850 gm, and he is in
(20) the percentile in intrauterine growth chart . He is admitted to
Neonatal tensive Care Unit. Which of the following is the classification of
this newborn according to stational age and birth weight using
intrauterine growth chart?
A. He is appropriate for gestational age ✓
B. He is extremely low birth weight
C. He is small for gestational age
D. He is very low birth weight.

773- A baby born at 24 weeks of gestation with birth weight 1000 gram
and at (90). Which of the following is the classification of this infant?
A. Low birth weight
B. Very low birth weight
C. Appropriate for gestational age
D. Large for gestational age ✓

774- A newborn born by elective caesarian section under general


anesthesia term 30 weeks of pregnancy. His weight is 1480 gm, and he is
in (30) the percentile in intrauterine growth chart. He is admitted to
Neonatal intensive Care Unit. Which of the following is the classification
of this newborn according to gestational age and birth weight using
intrauterine growth chart?
165
A. He is appropriate for gestational age ✓
B. He is extremely low birth weight
C. He is small for gestational age
D. He is very low birth weight.

775- Newborn born by elective caesarian section under general


anesthesia term 30 weeks of pregnancy. His weight is 1340 gm, and he is
in (20) the percentile in intrauterine growth chart. He is admitted to
Neonatal intensive Care Unit. Which of the following is the classification
of this newborn according to gestational age and birth weight using
intrauterine growth chart?
A. He is appropriate for gestational age ✓
B. He is extremely low birth weight
C. He is small for gestational age
D. He is very low birth weight.

776- A newborn born by elective caesarian section under general


anesthesia 30 weeks of pregnancy. His weight is 1000 gm . He is
admitted to Neonatal tensive Care Unit. Which of the following is the
percentile gestation of this newborn on the chart? ( see chart)
A. 20 % ✓
Note:
B. 10 % 776-777-778 ‫ تجي مع الثالث األسئلة‬Chart ‫صورة ال‬
C. 90 %
D. 95 %

166
777- A newborn born by elective caesarian section under general
anesthesia 32 weeks of pregnancy. His weight is 1000 gm . He is
admitted to Neonatal tensive Care Unit. Which of the following is the
percentile gestation of this newborn on the chart? ( see chart)
A. 10 % ✓
B. 20 %
C. 90 %
D. 95 %

778- A newborn born by elective caesarian section under general


anesthesia 22/24 weeks of pregnancy. His weight is 1000 gm . He is
admitted to Neonatal tensive Care Unit. Which of the following is the
percentile gestation of this newborn on the chart? ( see chart)
A. 20 %
B. 70 %
C. 90 % ✓
D. 25 %
Types Of Fever
TYPES OF FEVER
1- Intermittent fever ‫تكون الحرارة عاليه وتنزل بنفس اليوم طبيعية‬
Normal
2- Remittent fever ‫تكون الحرارة عاليه وتنزل شويه بس ما تكون في المعدل‬
( Abnormal )‫الطبيعي في نفس اليوم‬
3- Relapsing fever ‫ ساعه مستمرة عاليه و بعدين تنزل في المعدل الطبيعي‬٢٤ ‫تكون‬
Normal ‫وترجع ﺛاني يوم عاليه لمدة يوم أو يومين‬
4- Continuous fever ‫تكون الحرارة مستمرة تزيد ما تنخفﺺ‬

779- The nurse is caring for an adult patient who is admitted to the
hospital for fever and chills. The nurse repeatedly finds a temperature of
40° C (104° F) in the morning and 38.9°C (102°F) at night. What does the
nurse infer about the fever pattern?
A. Intermittent Fever
B. Remittent fever ✓
C. Relapsing fever
D. Continuous fever

167
780- The nurse notes that the patient has been experiencing febrile
episodes lasting more than 24 hours interrupted by periods of normal
body temperature that also last than 24 hours. What does the nurse infer
about the patient's fever pattern?
A. Intermittent Fever
B. Remittent fever
C. Relapsing fever ✓
D. Continuous fever

781- A 10-year-old child is brought to the hospital with high fever and
chills. The nurse records the vital signs and finds that her temperature is
104° F (40° C), blood pressure is 130/85 mm Hg, and pulse rate is 120/min.
The fever remains mostly high but is interspersed with periods of normal
body temperature. What pattern of fever does the child have?
A. Sustained
B. Intermittent ✓
C. Remittent D. Relapsing
4 Questions of breath sound lung
782- Which of the following related to Low pitched breath sound?
A. Crackles
B. Rhonchi ✓
C. Wheezing
D. Vescular

783- Which of the following related to High pitched breath sound?


A. Crackles
B. Rhonchi
C. Wheezing ✓
D. Bronchial

784- Which of the following related to harsh breath sound?


A. Crackles
B. Bronchial
C. Wheezing
D. Stridor ✓

168
785- Which of the following related to harsh breath sound?
A. Crackles
B. Tracheal ✓
C. Wheezing
D. Rhonchi

786- Which of the following lower postetior lung sound?


A. Bronchial
B. Bronchovesicular
C. Vesicular ✓
D. Both bronchial and vesicular

787- A Patient with yellowish sputum. What is expected lung sound?


A. Crackle ✓
B. Wheezing
C. Bronchial

788- Patient complain that he has difficulty in night sleeping, needs to put
3 pillows behind back lung sounds crackles & heart sounds confirm S
gallop which is the Most diagnosis?
A. Asthma.
B. Bronchitis.
C. Lt (left) side heart failure ✓
D. Rt side heart failure

169
Sickle Cell Anemia
‫األب و األم‬ ‫نسبة إصابة األطفال بالمرض‬
‫مصابين بالمرض‬ ‫مصاب‬%100
‫األب و األم حاملين للمرض‬ ‫ سليم‬%25 , ‫ مصاب‬%25 , ‫ حامل‬%50
‫أحد األبوين مصاب و األخر سليم‬ ‫ مصاب‬%50 , ‫ سليم‬%50
‫األب أو األم حامل المرض‬ ‫ سليم‬%50 , ‫ حامل‬%25 , ‫ مصاب‬%25

789- Both parent have sickle anemia trait the percentage for the child to
have the disease?
A. 100 %
B. 50 %
C. 75 %
D. 25% ✓

790-Both parent have sickle anemia trait the percentage for the child to
trait the disease?
A. 100 % Note:
‫ ( الكلمتين نفس المعنى تعني ناقل او حامل‬trait-carrier )
B. 50% ✓
C. 75 %
D. 25%

791- Both parent have sickle anemia disease?


A. 100% ✓
B. 50%
C. 75%

170
Psychiatric

792- Patient with paranoid schizophrenia most sign and symptoms?


A. Grandiose/granduer delusions ✓
B. psychomotor

793- According to Maslow hierarchy patient with schizophrenia has lack


of motivation to see herself. What are the needs for patient that lost?
A. Physiological need
B. Self-esteem ✓
C. Love an belonging
D. Safety

794- A child with deformity (broken) nose, the child went to the school
and his friends find this funny, the child was upset and went to the nurse
in the school and told him, he will stop coming to school, the nurse tokes a
paper and draw the child face and nose and tell him that he will look like
them after the procedure . Which of the following step in Maslow
hierarchy missed for child?
A. Self conception
B. Self deception
C. Self esteem ✓

795- A child with deformity (broken) nose, the child went to the school
and his friends find this funny, the child was upset and went to the nurse
in the school and told him, he will stop coming to school, the nurse tokes a
paper and draw the child face and nose and tell him that ‘he will look like
them after the procedure. The step the nurse perform is called?
A. Self conception
B. Self confidence ✓
C. Self esteem

796- Patient with schizophrenia seen with which of the following signs ?
A -Word salad
B. Associates lessness ✓

171
797- A parent asks the nurse how dissociative disorder can be treated?
A. Psychotherapy ✓
B. Psychology
C. Electro convulsive therapy (ECT)

798- A patient is diagnosed with agoraphobia. Which of the following


would the healthcare identify as a characteristic of this disorder?
A. Refuses to use a public restroom
B. Fear from crowded area ✓
C. Avoids interacting with strangers
D. Fear from dealing with animals

799- Patient is diagnosed with agoraphobia. Which of the following would


the healthcare identify as a characteristic of this disorder?
A. Refuses to use a public restroom
B. Avoids being in the presence of clowns
C. Avoids interacting with strangers
D. Fears the use of public transportation ✓

800- Patient with schizophrenia. He complains with anxiety episodes


which needs is highest priority in Maslow hierarchy?
A. Physiological needs
B. Self esteem
C. Safety ✓
D. actualization

801- A 70-year-old man presents to the clinic with difficulty sleeping at


night. He has not had a good night's rest for several months and feels
exhausted. He needs to place three pillows behind his back in order to
sleep. Examination of the lungs reveals crackles and wheel Auscultation
of the heart confirms an S gallop. Which of the following is the most
likely underlying health problem?
A. Apnea
B. Rest dyspnea
C. Orthopnea ✓
D. Dyspnea at night

172
802- What is the purpose of self help group for psychiatric patients?
A. Support
B. Psychotherapy ✓

803- Psychiatric patient repeats unknown words for nurse but the patient
can understand it. What is the best term describing situation?
A. Word salad
B. Neologism ✓
C. Circumstantiality
D. Thought problem

804- A nurse is interviewing a client with schizophrenia when the client


begins to say, "Kite, night, right, height, fright." The nurse documents
this as?
A. Verbigeration.
B. Stilted language.
C. Clang association. ✓
D. Neologisms.

805- A nurse was trying to establish a therapeutic with patient the


schizophrenia, During the conversation she noticed that he says; I go
shop, hop, bop, top. Which of the following symptom reflects the patient's
speech pattern?
A. Neologism
B. Flight of idea
C. Clang association ✓
D. Pressure of speech

806- When the psychiatric patient repeats his words boat, boat, boat.
What is the best term describes the patient action?
A. Echolalia
B. Palilalia ✓
C. Neologism.
D. Ward Salad

173
807- When the client imitates or repeats what the nurse is says this is an
example of?
A. Clang association
B. Echolalia ✓
C. Neologisms
D. Word salad

808- Which of the following psychiatric term best describes this symptom
"While talking, the psychiatric patient repeated the same words stated by
the nurse"?
A. Verbigeration.
B. Echolalia ✓
C. Echopraxia.
D. Preservation

809- Schizophrenic patient say (Sea, Airport, bus). What is the most term
for that?
A. Ward salad ✓
B. Loss of association
C. Neologism

810- Schizophrenic patient say ( I will eat fish, I will drink milk, I will go
to the sea). What is the most term for that?
A. Ward salad ✓
B. Loss of association
C. Neologism

811- 38 weeks-pregnant woman complains that she has been craving to


eat non food substance What is the term that describes this pregnant
mother’s condition?
A. Pica ✓
B. Bulimia
C. Anorexia
D. Binge eating

174
812- The doctor order an injection to the patient but not as a treatment .
It for the psychological concern to make the patient more concentrated ,
calm and follow orders?
A. Relief Pain
B. Drug Addiction
C. Level Of Tolerate
D. Placebo ✓

813- Doctor order medication for psychiatric patient , the drug is not for
therapeutic cause but to decrease the concern of the patient . The nurse
should expect which effect?
A. Placebo effect ✓
B. Tolerance level
C. Pain control
D. Addictive level

814- Nurse notes some students with anorexia nervosa what does she do?
A. Push them to eat.
B. One to one superior during eating
C. Let students eat with them the same meals ✓

815- A parent brings their teenage child the pediatrician’s office. The
parent reports that the patient frequently complains of abdominal
bloating and stomach pain after eating and also has a chronic sore throat.
The patient’s lab shows Hypokalemia. Which of the following diagnosis
should the nurse anticipate?
A. Anorexia nervosa
B. Bulimia ✓

816- A high school girl, who has fears of being obese and obsessed thin,
visited the primary healthcare center with her mother for The mother
reports that her daughter refuses to eat during with the family, and often
pretends being a sleep to skip meal. Which of the following disorders best
describe girl condition?
A. Bulimia
B. Obesity
C. Substance Abuse
D. Anorexia Nervosa ✓
175
817- Nurses meet another nurse college after failed in resuscitation of
child, she was crying and depressed. What is the suitable response in this
situation?
A. You need to take vacation some days until you feel better
B. Just return to your home and hugging your children
C. There was nothing in your hand to do.
D. Let her crying and talk about the matter ✓

818- Therapeutic communication is used by the nurse to create a


beneficial outcome in the delivery of quality nursing care. Which of the
following statement best describe the term therapeutic communication?
A. interactive process of socialization.
B. Assessment component of nursing process
C. Communication with patient through words and language
D. Process to encourage the expression of feelings and needs ✓

819- Woman came to ER with severe Vomiting. Vomiting not stop and
the patient was anxious and crying. She said I will die. What is the most
appropriate nursing diagnosis?
A. Fear to unknown disease ✓
B. Distribution body image

820- A patient was complaining of depression, muscle weakness and


continuous fatigue which of the following deficiencies is suspected?
A. Deficiency of vitamin B.
B. Deficiency of vitamin D. ✓
C. Deficiency of vitamin E
D. Deficiency of vitamin K

821- A 27 years old man is brought in Emergency Room after abdomen


in the road traffic accident. He has passing urine and complains of dull
pain and feeling of full back, abdomen and thighs. An emergency
laparoscopy is order. Which problem needs immediate attention?
A. Anxiety and restlessness ✓
B. Disturbed body function
C. Immobility and dysuria
D. Pain and discomfort

176
822- Extrapyramidal adverse effects and symptoms are most often
associated with which of the following drug classes?
A. Antidepressants
B. Antipsychotics ✓
C. Antihypertensives
D. Antidysrhythmic

823- To decrease the anxiety of a 10-year-old girl who is undergoing


surgery. Which of the following should the nurse do?
A. Use a heart model to show her how the surgery will go ✓
B. Provide her with verbal explanation of the upcoming surgery
C. Give her a book to read about the surgery 2 weeks prior
D. Let her parents talk to her about the importance of having surgery

824- While taking the history from a new patient, the nurse densified that
he had hypomanic episode which was alternating with a mirror
depressive\episode for the last two years. what is the most likely
diagnosis?
A. Bipolar I disorder
B. Bipolar II disorder
C. Dysthymic disorder
D. Cyclothymic disorder ✓

825- The psychiatric patient, She was speaking and laughing in a loud
voice, She was wearing a brightly-colored dress and an inconsistent make
up, and She had a history of depression before, what is her diagnosis?
A. Major depression
B. Bipolar disorder I ✓
C. Bipolar disorder ll
D. Cyclothymic disorder

826- The nurse was discussing with psychiatric patient. During meeting
the patient was getting close to her. What should the nurse do?
A. Shout and Push him
B. Change the topic of discussion
C. Keep silent and move steps back ✓

177
827- A 29-year-old woman patient was brought to the Outpatient for the
removal of stitches on her left cheek which was treated nine days back
after being involved in road traffic accident She covers her face
completely and requests to be seen by a female doctor. The site of the
wound was red, swollen and some pussy points were visible. She states
that she did not wash her face since her accident and kept her face
covered all the time as she did not want anyone to see it. What is the most
appropriate nursing diagnosis?
A. Hopelessness
B. Social isolation ✓
C. Anxiety
D. Powerlessness

828- Major depression activities?


A. Watch a movie with other patients.
B. Plan today's activities with the patient ✓

829- The patient came to receive his Lab results from the nurse. The
patient has diagnosed with cancer. The patient said to nurse I am not
believe that is my diagnosis, that is not mine. What is the stage
considering?
A. Denial ✓
B. Anger
C. Depression
D. Bargaining

830- Psychiatric patient doesn’t like her nose and mouth shape . She has
false belief of disturbed body image and counselled many doctors' for
doing operation. Doctor's not accepted her due to no need everything is
normal. What is type of disorder?
A. Pain
B. Conversion
C. Body dysmorphic disease ✓
D. Hypochondriasis

178
831- As identified by DR Elizabeth Kubler which stage of dying is
characterized by the transition from ‘NO’ not me to “yes, me but....”?
A. Anger
B. Depression
C. Acceptance
D. Bargaining ✓

832- Yemeni soldier caught in the war and he got many injuries. The
patient anxious and he is speaking about death for the nurse. What is the
most appropriate action for the nurse?
A -You say no or yes.
B. Why do you say you are dying
C. Why you say you will die.
D. Restating for patient wards ✓

833- The patient ask him self too much if he close the home door or no.
What is the most appropriate diagnosis?
A. Obsessive compulsive disorder. ✓
B. illusion
C. Hallucinations

834- The mother brought her child to hospital then the child admitted to
NICU. After leaving the mother for him. He started crying. what stage of
anxiety separation?
A. Protest, despair, denial ✓
B. Protest, despair and denial
C. Denial, protest, despair
D. Protest, denial, despair

835- Psychiatric patient was crying. When the nurse asked him, He said I
want to stay alone. I don’t want anyone with me leave me cry. What is the
best action for the nurse?
A. Maintain privacy and leave him alone
B. You appear sad. I am here to help you ✓
C. Stay with him and be silent
D. Say Okay and let him cry

179
836- What are the 3 stages of Separation Anxiety in order?
A. Protest, Despair, Detachment. ✓
B. Despair, Detachment, protst

837- The emotions that is happened during pregnancy and allow


pregnant to cry without any cause it is called which of the following?
A. Sadness
B. Mood swing ✓
C. Ambivalence

838- The nurse stated to Psychiatric patient " I will stay here with you".
Which of the following therapeutic communication technique's that the
nurse used?
A. Accepting
B. Exploring
C. Offering self ✓
D. Reflecting

839- Psychiatric patient use MAOI inhibitors. He should avoid tyramine


in food. What is the risk for tyramine intake?
A. GIT upset
B. Hypertension ✓
C. Kidney problems

840- Doctor write new order restrain (PRN) for Psychiatric patient that
he is overly aggressive which of the following indicate that the nurse
should do?
A. Call doctor while patient aggressive to do that by himself.
B. Apply the order one time only or if needed. ✓
C. Close patient room and do not apply restrain.
D. Express that is not acceptable and complains about your supervisor

841- Female patient saw the IV tubing as a snake the nurse consider this
reaction as ?
A. Illusion ✓
B. Delusion
C. Hallucination

180
842- Psychiatric patient is complaining from extrapyramidal side effect
with continuous spasms and muscle contractions), motor restlessness,
rigidity, slowness of movement, tremor, and irregular, jerky movements,
upward eye movement, smacking and sucking of lips and protruding
tongue. What is the most common drug cause that?
A. Clozapine
B. Haloperidol ✓

843- Psychiatric patient came to ER. The patient use razor markes on his
hand with letters of the word "I didn’t want to serve ever at this life".
The patient wrote on his arm bye tattoo. What is the most appropriate
diagnosis?
A. Borderline personality disorder ✓
B. Dissociative disease
C. Major manic episodes

844- The nurse is performing an admission assessment for a client who


has schizophrenia. The nurse notices that the client's appearance is
unkempt, and he appears to be actively hallucinating. Which of the
following should be the nurse's priority assessment?
A. Perception of reality
B. Ability to follow directions.
C. Physical needs ✓
D. Mental status

845- Which of the following is the most affected drug for The psychiatric
patient and cause more problem?
A. Atypical antipsychotic
B. Typical antipsychotic because it has side effects more than atypical ✓
C. Serotonin inhibitor
D. Dopamine inhibitor

181
846- A 15-year-old girl is admitted after a Motor Vehicle Accident child
has a fracture in her left leg and a wound in her face, when dealing with
this child the nurse should be aware that important task to achieve
during this age is which of the following?
A. Initiative versus guilt
B. Industry versus inferiority
C. Trust versus mistrust
D. Identity vs. Role confusion ✓

847- A 19-year-old boy has been hospitalized with fracture in upper and
lower extremities after accident, then provided with casts for upper and
lower limbs, Which of these Nursing diagnoses should the nurse consider
in the Nursing plan of care. According to his age?
A. Impaired social interaction
B. Alteration in body image ✓
C. Risk for infection
D. Anxiety

848- An 80-year-old man has Alzheimer's disease and increasingly has


frequent periods of dementia during which he doesn't remember family
members names, and is not oriented to person, place, or time. The health
care provider speaks with the man's son about becoming the medical
power of attorney for his father who is no longer able to do this for
himself. Which two elements of informed consent does the father lack?
A. Autonomy and fidelity ✓
B. Confidentiality and justice
C. Competence and comprehension
D. Voluntariness

849- Psychiatric patient with depression. The doctor order antidepressant


with low side effect. Which of the following is the most appropriate type
of antidepressant?
A. Anti depressant nor Adrenaline
B. Monoamine oxidase inhibitors (MAOIs)
C. Selective serotonin reuptake inhibitors (SSRIs) ✓

182
850- Women come to ER with her husband and the husband demonstrate
she not talk or voluntary eat since the son died in accident, She isolated
since she has seen his son body. What the long term care for her?
A. Physiological needs
B. psychological Care ✓
C. spiritual support
D. family support

851- Which of the following is the main reason the makes nurses
concerned about adolescents health status?
A. Take risky behaviors ✓
B. Consider themselves as adult
C. Have more health issues
D. Transitional period to adulthood

852- A patient with a spinal cord injury state, "I have no constitution; I
can’t do anything for myself." Which of the following best describes this
patient condition?
A. Powerlessness ✓
B. Delusions
C. Suicidal
D. Resignation

853- Maslow's hierarchy of needs is an idea in psychology. The priority


nursing care of Maslow hierarchy is?
A. Nursing diagnosis
B. Nursing care
C. Nursing planning ✓
D. Nursing care with identification

854- What is the Priority of patient with schizophrenia?


A. Self esteem
B. Medication compliance ✓
C. Family support
D. Impaired thought process

183
855- An 18-year-old girl was under weight and malnourished163 cm, her
body weight was 45 Kg and her admitted in the Female Medical Ward for
severe observed that on the second day she inducing Dinner. On asking,
she replied that inducing helps clear the stomach bacteria. Which aspect
of the nursing care area is altered?
A. Self-concept
B. Health perception. ✓
C. Value-belief system
D. Nutrition management

856- A 26 years old married woman is admitted in the plastic surgery for
the correction of burn strictures and skin g rafting on her neck and face
under general anesthesia. While discussing the treatment with her, the
plastic surgeon explained that she will have a series of surgeries but she
needs to be on family planning until the treatment is completed. the
patient aske d the nurse whether she will be normal again. What initial
assessment is required?
A. Detailed history and physical examination
B. Patient’s acceptance for the treatment plan
C. Need for psychological support to reduce anxiety ✓
D. Family’s involvement and consent for her treatment .

857- There is single parents. One of both has one child. They will married.
What is the type of family?
A. Nuclear
B. Extended
C. Blended ✓

858- Couple has a son and a daughter, and both moved out and got family
on their own Which type of family structure?
A. Extended ✓
B. Blended
C. Neclear

859- Patient complained from Loss of consciousness, The patient


diagnosed with meningitis. What's the first action for the patient?
A. Neurological assessment ✓
B. Observe for seizures
184
860- A schizophrenia paranoid has previous attempts to suicid which is
most appropriate diagnosis?
A. Risk to harm himself ✓
B. Risk to harm others

861- A schizophrenia patient started shouting loudly in the ward to


anyone who speaks to him. what should the nurse do?
A. Administer tranquilizer IM
B. Call the security
C. Isolate the patient
D. Speak quietly to the patient and be cautious ✓

Positions

862- Position post appendectomy?


A. High fowlers
B. Semi fowler ✓
C. Sitting
D. Dorsal recumbent

863- What is the Position after thyroidectomy?


A. Lateral flexed
B. Semi fowler with slight neck flexed ✓
C. Prone head extended
D. High Fowler with neck extended

864- What is the Position during thyroidectomy?


A. Supine with hyper flexion of neck
B. Supine with hyperextened neck ✓
C. Semi fowler
D. Lateral with slightly flexion neck

865- Position during liver biopsy?


A. Prone
B. Supine right hand below head ✓
C. Right side lying
D. Lateral
185
866- Position of flatulent?
A. Flat
B. Side lying
C. Knee chest ✓
D. Semi fowler

867- A 56- year old man with history of COPD is complaining of chest,
shortness of breath , lethargy, fever and productive cough. Upon
examination, crackles could be heard in the lower lobes BP 11./70 HR 111
RR 18 TEM 37.4 Oxygen Sat 90%. What is COPD position?
A. Prone
B. Supine
C. High fowler ✓
D. Trendelenburg

868- Position of patient with endoscopy on distal part of intestine?


A. Supine
B. Left lateral recumbent ✓
C. Right lateral sims
D. Prone

869- Child admitted to surgical ward for thyroidectomy. Which position


for child with thyroidectomy?
A. Semi fowler with slight flexion neck ✓
B. Supine with hyperextended neck

870- The nurse is preparing the patient for hospital for discharge
following above-the-knee amputation with rigid dressing over the
residual limb. Fourteen days following surgery, the patient is successfully
fitted with Prosthetic limb and begins physical therapy Which of the
following resting position is most Beneficial?
A. Sitting with legs crossed
B. Abduction of residual limb
C. Knee flexion when sitting in chair
D. Knee extensions when in bed ✓

186
871-The Foley Family is caring for their youngest child, Justin, who is
suffering from tetralogy of Fallot. Which of the following positions is used
for congenital heart condition?
A. Semi fowler's position
B. knee chest position ✓

872- The nurse is preparing a patient for an insertion of a central venous


pressure line into a neck vein. The nurse should place the patient into
which position?
A. Side-lying position
B. Fowler's position
C. Trendelenburg position ✓
D. Semi-Fowler's position

873- A 6 years old preschool returned from the operating room after
tonsillectomy. A nurse received the patient from recovery room. Which
position the nurse place the child post tonsillectomy?
A. Supine
B. Side lying ✓
C. Low fowler
D. High fowler

874- Child came to ER with broken noes, bleeding , mouth breathing ,


difficulty breathing .Which of the following Position?
A. Supine è elevation head and clean nose
B. Flexion head downward and clean nose ✓

875- Prior to administering an enema, the nurse will assist the patient to
assume what is the position?
A. Prone with pillow under knees
B. Left-side with right knee flexed ✓
C. Right-side with left knee flexed

876- What is the position during Chemotherapy session?


A. Supine
B. Semi fowler ✓

187
877- Position for lumbar puncture?
A. Lateral recumbent position ✓
B. Prone position
C. Supine position

878- After lumbar puncture position?


A. Supine ✓
B. Prone

879- What is the position for adenoidectomy?


A. Semi Fowler ✓
B. Sitting
C. prone

880- A client has just returned to a nursing unit after an above-knee


amputation of the right leg. The nurse should place the client i which
position?
A. Prone
B. Reverse Trendelenburg's
C. Supine, with the amputated limb flat on the bed
D. Supine, with the amputated limp supported with pillows ✓

881- The nurse who is planning to discharge education should instruct the
caregivers that during hyper cyanotic spell, which position is the most
likely to benefits the child?
A. Supine
B. Prone
C. Side lying
D. Knee chest position ✓

882- During the initial assessment, a registered nurse decided to apical


pulse of a patient with coronary artery disease. Which position is best of
the patient to be placed in to measure pulse?
A. Prone position
B. Supine position ✓
C. Position on his /her left side
D. Position on his /her right sid

188
883- A 43-year-old woman fell and hit her head. She was admitted to the
hospital and put on observation for a possible closed head injury. The
patient's orders include hourly checks for increasing intracranial
pressure and nursing interventions to reduce intracranial pressure
during the assessment, the nurse notes the intracranial pressure is 16
mmhg . Which is the most appropriate nursing Intervention?
A. Provide an intravenous fluid bolus
B. Position patient in semi-fowler's ✓
C. Prepare for hypothermia induction
D. Hyperventilate with positive pressure

884- Position after head trauma ?


A. Prone
B. Elevate head of bed 30degree to decrease intracranial pressure ✓

885- A 65-year-old man is undergoing pre-operative preparation fond


scope procedure in which the physician will visualize the large and distal
part of the small bowel with a camera attached to the flexible tube. Which
of the following positions is the most appropriate?
A. Left lateral Sim's ✓
B. Right lateral recumbent
C. Trendelenburg
D. Prone

886- A nurse is preparing to transfer a patient from the operating table


to the postoperative bed. The patient had a right total hip arthroplasty .
the nurse must maintain the patient hips of the following position?
A. Flexion
B. Abduction
C. Extension ✓
D. Adduction

887- A Pregnant woman with Leg cramps. What should the nurse
Instruct her?
A. Knee extended with dorsiflexion for foot ✓
B. Change position frequently

189
888- Which of the following Best position for breastfeeding at night?
A. Setting
B. Side-lying ✓
Diet & Food

889- A patient is attending a primary health clinic for regular checkup.


He complains of constipation. After assessment, the nurse instructed him
to consume bulk-forming foods. Which of the following is the best bulk-
forming foods?
A. Fruit juice
B. Raw meat
C. Whole grains ✓
D. Milk products

890- 5 year-old child was admitted with Nephrotic Syndrome. A nurse


noticed that the child has slight facial puffiness with mild pitting edema
on his hands and feet. There was no distended abdomen diet the nurse
should order for the child?
A. High protein, high salt diet
B. Low protein, low fiber diet
C. Low protein, normal salt diet
D. Normal protein, low salt diet ✓

891- An Indian patient, who is vegetarian, is being discharged from after


an elective surgical procedure. The unit nurse is teaching using visual
aids and pictures about food combinations complete protein. Which of
the following food items should the nurse recommended dietary list of the
patient?
A. Lentils ✓
B. Potatoes
C. Macaroni
D. Green salad

190
892- After thyroidectomy which of the following juice should be offered ?
A. Apple juice ✓
B. Tomato juice
C. Orange juice
D. Strawberry

893- A 40-year-old women patient with Parkinsonism Medical Ward. The


patient stated that she has the past two weeks The nurse was planning to
Which type of diet is most suitable for parkin?
A. Solid
B. Liquid
C. Semi solid ✓
D. Clear liquid

894- A 33 old woman presented to the ER with general weakness. The


laboratory investigation indicated VIT D deficiency. Which of the
following nutrient should be recommended as a good source of vitamin?
A. Rice
B. Green tea
C. Orange juice
D. Fish liver oils

895- While planning for discharge education for a mother or rickets, the
nurse knows to include the need for an adequate. Which food should the
mother choose for her child?
A. Potato and squash
B. Orange and tomatoes
C. Egg yolk and fish ✓
D. Milk and yogurt

896- A 9-year-old child is postoperative after tonsillectomy. The nurse


should ask the parents to give the child which of food after discharge
from the hospital?
A. Meat and rice
B. Hot dog and potato chips
C. Mashed potatoes and soup ✓
D. Cucumbers and tomato salad

191
897- Which food consider low potassium diet?
A. Bananas
B. Avocado
C. Berriers
D. Strawberry ✓

898- Which of the following food rich in potassium?


A. Orange ✓
B. Tofu
C. Butter
D. Milk

899- Oral iron supplements are prescribed for child with iron deficiency
anemia the nurse instruct the mother to administer the iron with which
food?
A. Orange juice ✓
B. Apple juice
C. Milk
D. Water

900- A 52 year old woman is scheduled to undergo an abdominal perineal


resection in the third day for removal of a cancer of the rectum. The
nurse review the care plan with the patient the patient will receive
prophylactic antibiotics and will be given a mechanical bowel preparation
the day before. Which additional preparation should the patient
undertake in this time?
A. Wear pressure stocking
B. Perform leg strengthening exercise.
C. Maintain high-protein low residual diet ✓
D. Take daily ferrous iron tablets

901- Which of the following is food high in purines?


A. Rice
B. Red meat ✓

192
902- 37 year-old woman with inflammatory bowel disease is scheduled to
undergo a procedure in which a stoma will be formed in the right lower
quadrant, five centimeters below the waistline. The nurse advises the
patient on how to avoid potential post-operative intestinal obstruction.
Which of the following types of food best recommended post-operatively?
A. Broccoli and fish
B. Meats and cauliflower
C. Yogurt and parsley ✓
D. Corn and seeds

903- Patient diagnosed with vitamin c deficiency. Which of the following


pups of foods would the nurse most likely instruct the patient to add to?
A. Broccoli, tomatoes, strawberries, potatoes ✓
B. Legumes, egg, oats, meat
C. Yeast, fish , bananas, nuts
D. Milk, liver, rice, beans

904- A nurse evaluate dietary practices of a patient who had been as


having acute glomerulonephritis. the patient verbalized following the
appropriate diet for acute glomerulonephritis. which of the following diet
verbalized by the patient is approved diagnosis?
A. Restricted fluid intake ✓
B. Restricted dietary protein
C. Increase intake of low fiber food
D. Increase intake of sodium-rich food

905- Which of the following food should the cancer patient avoid?
A. Fatty food ✓
B. High sodium food
C. Fiber food
D. Bran products containing food

906- A nurse has taught a patient with iron deficiency anemia about
eating also that are high in iron. Which of the following meal is low in
iron?
A. Dried beans and brown rice B. Eggs and whole wheat toast
C. Steak and a salad made with fresh spinach
D. Cheese pizza and pasta with tomato sauce ✓
193
907- Patient with high cortisol/ cholesterol. What should the nurse
instruct for Food restriction?
A. White bread, chicken ✓
B. Egg yolk ,liver
C. Fruits and vegetables

908- Which of the following is low fat diet?


A. Lemon Juice
B. Beans, fish ✓
C. Chicken, egg yolk

909- Patient with renal disease Low sodium , High potassium, Low
calcium. What should provide intake?
A. Low sodium ✓
B. High potassium
C. High phosphorus
D. Low carbohydrates

910- Patient with acute renal failure (ARF) Lab result : NA ( sodium )120
Potassium 6 Calcium normal result. What is the most appropriate diet
should nurse provide in food?
A. Low NA ✓
B. High potassium
C. High phosphate
D. Low carbohydrate

911- Which of the following diets would be most appropriate for the client
with Crohn's disease?
A. High-calorie, low-protein.
B. High-protein, low-residue. ✓
C. Low-fat, high-fiber.
D. Low-sodium, high-carbohydrate

912- A 6 year old male is diagnosed with Nephrotic syndrome. When


nursing care for the patient the plan of diet should be?
A. High salt, High fat
B. High salt, High cholesterol
C. Low salt, low fat ✓ D. Low protein, high
194
913- An adolescent with a juvenile diabetes mellitus develops chronic
renal failure. Which of the following diets is suitable?
A. Low fat
B. Low mineral
C. Low protein ✓
D. Low carbohydrate

914- The nurse is caring woman that has cancer, and she is under
Chemotherapy. She is complaining anorexia and the patient has low
weight. What should the nurse instruct her?
A. Eat small meals every day. ✓
B. Eat large meals every day.
C. Eat if you are hungry.
D. Eat your favourite food.

915- A home care nurse instructing a mother of child diagnosed with


cystic fibrosis (CF) about the appropriate dietary measures. Which diet
should the nurse tell the mother that the child needs to consume?
A. Low calorie – low fat diet
B. High calorie – restricted fat
C. Low calorie – low protein diet
D. High calorie – high protein diet ✓

916- The nurse is caring for a patient who has abdominal pain
constipation last three days. The nurse teaches the patient about the most
likely active-producing foods. What are the foods that are mostly useful
to relieve
constipation?
A. Cheese, pasta and eggs
B. Rice, eggs, and lean meat
C. Bran(Oats), figs, prune ✓
D. Cabbage, bananas and apple

195
ECG

917- What is this rhythm?


A. Sinus rhythm ✓
B. Tachycardia
C. Brady cardia
D. Supra ventricular tachycardia

918- What is this rhythm?


A. Sinus rhythm
B. Tachycardia
C. Bradycardia ✓
D. Supra ventricular tachycardia

919- What is this rhythm?


A. Supraventricular tachycardia
B. Ventricular Fibrillation
C. Ventricular Tachycardia
D. Tachycardia ✓

920- What is this rhythm?


A. Tachycardia.
B. Ventricular Tachycardia ✓
C. Ventricular Fibrillation
D. Supra ventricular tachycardia

196
921- What is this rhythm?
A. Tachycardia.
B. Ventricular Tachycardia
C. Ventricular Fibrillation ✓
D. Supra ventricular tachycardia

922- What is this rhythm?


A. Sinus rhythm
B. Tachycardia
C. Atrial flutter
D. Atrial fibrillation ✓

923- What is this rhythm?


A. Sinus rhythm. Note:
‫ غير موجودة بشكل‬ECG ‫بداية أسئلة‬
‫مباشر في قنوات الشرح‬

B. Tachycardia
C. Atrial flutter ✓
D. Atrial fibrillation

924- What is this rhythm?


A. Tachycardia.
B. Ventricular Tachycardia
C. Ventricular Fibrillation
D. Supra ventricular tachycardia ✓

925- Patient is experiencing Myocardial ischemia. The ECG is shown ST


depression. What is more changes in ECG the nurse expects?
A. T waves inversion ✓
B. Tall T wave

197
926- Patient came to ER with acute chest pain, restlessness. The patient
has cardiac surgery before. He has some injuries on his body Bl. P 110/70
mmhg, HR 140 b/m, RR 22. What is the most appropriate nursing
intervention?
A. Obtain 12 lead ECG ✓
B. Assess for bed sores
C. Administer Pain medication
D. Start I. V fluids

927- Adult Patient (big Scenario with ECG finding at last situation)
patient with HR : 55 b/m. What is the most appropriate Diagnosis?
A. Sinus Bradycardia ✓
B. Tachycardia
C. Normal rhythm

928- Infant 11 month On ECG heart rate 240 b/m. Embedded in the QRS
complexes and absent p wave. What is the expected diagnosis for the
infant?
A. Ventricular tachycardia (VT)
B. Bradycardia
C. Supraventricular tachycardia (SVT) ✓
D. Atrial fibrillation

929- Adult Patient (big Scenario with ECG finding at last situation)
patient with HR : 55 b/m. What is the most appropriate Diagnosis?
A. Sinus Bradycardia ✓
B. Tachycardia
C. Normal rhythm

930- Nurse is watching the cardiac monitor, and a client’s rhythm


suddenly changes. There are no P waves; instead, there are wavy lines.
The QRS complexes measure 0.08 second, but they are irregular, with a
rate of 120 beats a minute. The nurse interprets this rhythm as?
A. Sinus tachycardia
B. Atrial fibrillation ✓
C. Ventricular tachycardia
D. Ventricular fibrillation

198
931- The nurse reviews a client’s electrolyte laboratory report and notes
that the potassium level is 2.5 mEq/L (2.5 mmol/L). Which patterns
should the nurse watch for on the electrocardiogram (ECG) as a result of
the laboratory value ? Select all that apply.
A. U waves ✓
B. Absent P waves
C. Inverted T waves ✓
D. Depressed ST segment ✓

932- A 9 A cardiac monitor for a patient in a Coronary Care unit shows


abnormal ECG rhythm with heart rate of 159 beats, QRS complex (0.18
second), and absent P wave. What could be the type of possible
dysrhythmia?
A. Sinus tachycardia
B. Ventricular tachycardia ✓
C. Ventricular fibrillation
D. Supraventricular tachycardia

Different between SVT and VT

Supraventricular Tachycardia Ventricular tachycardia ( VT )


( SVT )
1 Heart Rate >200 1 Heart Rate 140/150/170
2 Absent P wave ,abnormal 2 Absent P wave
3 Rhythm regular 3 Rhythm irregular
4 QRS invited 4 QRS wided
5 QRS Complexes ( 0.12 5 QRS Complexes ( 0.18 second)
seconds)

199
933- A 57-year-old man is admitted to the Cardiac Unit with palpitation
headache, and chest tightness. On auscultation S3 gallop and murmur
can be heard (see image). Blood pressure 97/60 mmHg
Heart rate 170 /min Respiratory rate 25 /min Temperature 37.3 C . What
is the ECG rhythm of the patient ?
A. Atrial fibrillation
B. Ventricular tachycardia
C. Ventricular fibrillation
D. Supra-ventricular tachycardia ✓

934- A37 years-old man present man present to emergency department


with chest pain . An ECG shows significant elevation in the ST segm II ,
III and AVF ,indicating MI related to occlusion in the artery what is the
location of MI?
A. Posterior MI
B. Anterior MI
C. Inferior MI ✓
D. Lateral MI

935- Nurse is caring for a 58-year-old patient (See lab result). Test Result
Normal Value Magnesium 2.8 / 0.7-1.2mmoI/L. Which ECG change is
the nurse expected to note?
A. Prolonged QRS ✓
B. Multiple P waves
C. Prominent U waves
D. Depressed ST segment

936- ECG showed S-T elevation to confirm diagnosis of MI the nurse


expect to do?
A. Troponin level ✓
B. Cardiac catheterization
C. ECO

200
937- A physician ordered an ECG for a 35-year-old woman with chest
pain. What is the main finding that can be detected by the ECG?
A. Abnormality with the valves in the heart
B. Abnormality in the ejection function
C. Myocardial ischemia or infarction ✓
D. Cardiomegaly

938- The nurse reviews a clients ECG and notes P waves. With regard to
the cardiac cycle, what does the P
wave indicate?
A- Ventricular depolarization
B- Atrial depolarization ✓
C- Atrial repolarization
D- Ventricular repolarization

939- Patient diagnosed MI then


suffered from cardiogenic shock.
What should the nurse observe in
symptoms?
A. Hypertension and hyperventilation
B. Hypotension and shortness of breath ✓

940-The nurse is caring for a patient who is admitted with chest pain and
diagnosed with MI. Which test is assisting in diagnosis?
A. CK-MR
B. CK-MH
C. CK-MB ✓
D. CK-HM

941- The nurse is caring for an adult patient who is admitted with. Chest
pain that started four hours ago. Which test will be? most specific in
identifying acute heart damage?
A. CKP
B. Troponin level ✓
C. CK-MB
D. Cholesterol level

201
942- 19-year-old women presented to the Emergency Department
complaint of severe chest pain. The ECG showed that the patient
myocardial infarction. The doctor ordered the nurse to give the 800 mg of
aspirin. What is the primary indication of aspirin in this case?
A. Breaks down the thrombus
B. Decreases the formation of platelet plugs ✓
C. Inhibits the conversion of prothrombin to
D. Interferes with vitamin k to maintain

943- When ventricular fibrillation occurs in a CCU, the first person


reaching the client should?
A. Administer oxygen
B. Defibrillate the client ✓
C. Initiate CPR
D. Administer sodium bicarbonate intravenously

944- What occurs during cardiogenic shock and result I adequate tissue
perfusion?
A. Increase resistance of arterial
B. Decrease effectiveness of the heart as pump ✓
C. Increased shunting of critical blood flow to heart
D. Decrease capacity of the venous beds

945- A 50-year-old woman post myocardial infraction was admitted in the


medical ward . A nurse was assigned to care for the patient. Blood
pressure 140/80 Heart rate 120 Respiratory rate 22 Temperature 37.5 O2
sat 95% . What is most common complication?
A. Arrhythmias ✓
B. Endocarditis
C. Cardiac failure
D. Cardiogenic shock

202
CTG Categories

946- Which of the following category


of CTG ? See image
A. Normal
B. Category1.Early deceleration ✓
C. Category 2 Variable deceleration
D. Category 3

947- Which of the following category


of CTG ? See image
A. Normal
B. Category 1
C. Category 2 Variable deceleration ✓
D. Category 3

948- Which of the following


category of CTG ? See image
A. Normal
B. Category 1
C. Category 2
D. Category 3 Bradycadia ✓

203
949- Which of the following category of CTG ? (See image )

A. Normal ✓
B. Category 1
C. Category 2
D. Category 3

950- Which of the following category of


CTG? (See image )
A. Normal
B. Category 1
C. Category 2
D. Category 3 ✓

951- Which of the following category of


CTG? (See image)
A. Normal
B. Category 1 ✓
C. Category 2
D. Category 3

952- Which of the following category of CTG?? See image last question
ECG
A. Normal
B. Category 1
C. Category 2
D. Category 3 ✓

204
APGAR SCORE

953- A neonatal nurse performs Apgar assessment at 1 minute of birth to


evaluate the physical condition of the newborn and immediate need for
resuscitation. At 1 minute, Apgar score is 7. At 5 minutes, Apgar score is
to the progression of scores suggests?
A. A healthy newborn ✓
B. The need for supplement oxygen
C. A genetic defect
D. The infant is becoming stable

954- A newborn APGAR score at 1 and 5 minutes is 5 and 10, half an


hour later the baby became bluish in color with heart rate of 140/m, your
first action would be?
A. Estimate the score again
B. Shower the baby with warm water
C. Give oxygen immediately ✓
D. Ignore the finding because it is normal

205
955- A nurse is caring for a full term newborn who was delivered five
minutes ago. The infant's Apgar score was 8 at one minutes and 10 at five
minutes. Which of the following has the highest priority?
A. Assessing the infant's red reflex
B. Preventing heat loss from the infant ✓
C. Maintaining the infant in the supine position
D. Administering humidified oxygen to the infant

956- Neonate born with normal Apgar score. What should the nurse do?
A. Administer oxygen
B. Protect neonate from heat loss and keep warmed ✓

957- A newborn baby is being evaluated on the APGAR score


immediately after his birth. The score measures the quality of newborn's
pulse rate, reflexes, muscles tone and respiration. His APGAR score is
graded as 7 out 10.What is the significance of measuring the APGAR
score immediately after the birth?
A. It helps to plan the treatment for congenital diseases
B. It serves as a permanent record for the newborn babies
C. It provides the bases for the comparison, a few minutes later ✓
D. It helps identifying the abnormalities related to muscular tone

958- Five minutes post-birth, a neonate has a heart rate of 98, irregular
breathing, actively moves all extremities, but has bluish hands and feet,
all as a weak and timid cry. Which is the correct APGAR assessment
score?
A. 9
B. 8
C. 7
D. 6 ✓

959- Which of the following APGAR score is consider moderate depress /


risk at the first minutes and after 5 minutes of assessment?
A. 3-4
B. 3-5
C. 5-7 ✓
D. 8-10

206
960- In determining the one minute APGAR score of a male infant the
nurse assesses a heart rate of 120 beats per minute and respiratory rate of
44 per minute. He has flaccid muscle tone with slight flexion and
resistance to straightening. He has a loud cry with colour is acrocyanotic
What is the APGAR score for the infant?
A. 7
B. 8 ✓
C. 9
D.10

961- A nurse documented assessment on a newborn as listed. (See table)


Indicator Points Heart rate less than 98/min Respiratory rate 28/min
irregular Muscle tone minimal flexion of the extremities Reflex
irritability grimace Color body pink, extremities blue What is the total
Apgar score
A. 0
B. 3
C. 5 ✓
D. 7

962- The nurse Assess Neonate after delivery using Apgar score.
Spontaneous respiration Prompt respond Limited cry 98 pulse Pinkish
body color except hands. What is the expected score?
A. 8
B. 7 ✓
C. 6
D. 5

963- The neonate after delivery with normal heart rate and normal
respiration rate. The neonate with Pink skin, active reflexes and Lusty
cry. What is the most appropriate Apgar score?
A. 9
B. 10 ✓
C. 8
D. 7

207
GLASGOW COMA SCALE

964- A 55 years old male looks at you when you speak to him , when you
ask him about the date he says blue , and withdraws his legs to pain the
GCS score is?
A. 10 ✓
B. 11
C. 12
965- A nurse examines a client’s level responsiveness . She finds that the
patient opens his eyes to verbal commands, obeys verbal commands, and
is oriented to time, place, and person. What’s the client’s Glasgow Coma
Scale: Which of the above locations represents the best sites to inspect
chest retractions in a child with lower respiratory tract disorder?
A- 11/15
B-12/15
C-13/15
D-14/15 ✓

208
966- A 9 -year-old child is admitted to the Emergency Department injury.
The child is oriented to the place, person and time, spontaneously, obeys
commands. The nurse is doing a ped Coma Scale (PGCS). Which of the
following score the nurse should record?
A. 3
B. 8
C. 12
D. 15 ✓

967- A registered nurse in the Intensive care Unit performed Glasgow


scale assessment on a 50 - year - old man who sustained head 24 hours
ago. The recorded Glasgow Coma score was 10. Which one of the
following actions should be taken immediately nurse?
A. Inform the registered nurse in charge of the nursing
B. Protect the patient by raising the two side rails ✓
C. Inform operation theatre for emergency surgery
D. Perform neurologic assessment every 15 minutes

968- As per of a neurological assessment, which of the following is


associated with the higher score on the Glasgow coma scale?
A. Eye opening to pain, no verbalization.
B. Confused, obey command. ✓
C. Localized pain, abnormal extension.
D. Eye opening to speech, confused.

969- What is the time nurse should follow Glasgow's coma scale?
A. Every quarter of an hour
B. Every hour
C. Every 2 to 4 hours ✓
D. Every shift

209
Meningitis

2 Signs Of Meningitis :

970- Patient with meningitis after treatment with effect MCV, how to now
is effect from this sign?
A. CBC
B. Negative Brzezinski sign ✓
C. Relive headache

971- Patient complained from Loss of consciousness, the patient


diagnosed with meningitis. What's the first action for the patient?
A. Neurological assessment ✓
B. Observe for seizures

972- What is the main organism that cause meningitis for a child?
A. Meningococcal
B. Staphylococcus
C. H. Influenza
D. Streptococcal Pneumonia ✓

973- A nurse is putting together a presentation Meningitis. On which of


the following has not been linked to Meningitis in?
A. S. Pneumonia
B. H. Influenza
C. N. Meningitis
D. C. Difficile ✓

210
974- A child is treated for bacterial meningitis with an intravenous
antimicrobial agent. Which of the following BEST indicates effectiveness
of the treatment?
A. Increased appetite
B. Low Temperature ✓
C. Decrease pulse
D. Nodule of skin disappear

975- Mother has infant 6 months came to the clinic and afraid that her
baby May will have meningitis as his brother already have. What should
the nurse tell the mother?
A. Hib vaccine can decrease the meningitis ✓
B. There are now vaccinations for all meningitis type
C. Tell the mother the disease is will not come

976- A 6 month old infant is seen in the vaccination clinic to get his
regular vaccine. The mother inform the nurse about her previous child
that has meningitis and she is worries about his infant will get the same
disease. Which of the following the best nursing response to the mother?
A. Meningitis rarely occurs during infancy.
B. Genetic predisposition to meningitis is found
C. Vaccination is now available to prevent all types of meningitis
D. HIB vaccination has decreased the incidence of the meningitis ✓

977- During meningitis outbreak in one of the hospital, the non-immune


staff members were given immunoglobulin in order to prevent them from
the infection. What is the type of immunity will be developed by those
staff?
A. Active
B. Long term
C. Natural
D. Passive ✓

211
978- A 5 years old child admitted with bacterial meningitis and is having
seizure. Which of the following intervention the nurse should initiate?
A. restrain the child’s limbs to prevent injury
B. slowly put the child on his side on the floor ✓
C. clear the area of objects and administer oxygen
D. roll the child to prone position to protect the airway

979- The nurse observes the client CSF ,Which of the following of CSF
color indication for patient with bacterial meningitis?
A. Cloudy ✓
B. Clear
C. Red
D. Brown

980- 71-year-old male was diagnosed with subdural underwent burr hole
craniotomy for subdural hematoma days ago.
In order to detect the sign of meningitis as one of which of the following
indicates the patient has meninge?
A. Negative Kernig's signs
B. Positive Brudzinski's sign ✓
C. Absence of nuchal rigidity
D. Glasgow comma scale of 14 points

981- A 3-year-old child is admitted to the hospital with seizures. He


oriented and has a rash in his extremities and is diagnosed meningitis.
While doing physical examination of him, he starts to seizures. Blood
pressure 100/57 mmHg Heart rate 110 /min Respiratory rate 30 /min
Temperature 39.5 Which of the following is the priority of care during
the seizure?
A. Put the child on right side ✓
B. Protect the child from injury
C. Call the physician immediately
D. Administer oxygen 100%

212
982- A 3 years old child is admitted to the hospital with seizures. He was
alert, oriented and has a rash in his extremities and is diagnosed with
meningitis. While doing physical examination of him, he starts to develop
seizures . Blood pressure 100/57 mmHg Heart rate 110/min Respiratory
rate 30/min Temperature 39.5 C Which of the following vaccine is used
to prevent meningitis?
A. HIB vaccine ✓
B. Varicella vaccine
C. BCG vaccine
D. Rubella vaccine

983- Which of the following vaccines in highly recommended to be taken


by pilgrims before Haji season?
A. Meningococcal meningitis ✓
B. Tuberculosis
C. Hepatitis A
D. Polio myelitis

984- patient presented with high fever, headache, vomiting and neck
stiffness for the past 3 days, which of the following is the first diagnostic
intervention for this patient?
A. Urine and stool analysis
B. Lumber puncture with CSF aspiration . Rationale meningitis ✓
C. Complete blood count
D. Chest and abdomen x-ray

985- A 33-year-old man with meningitis present Department. Healthcare


providers plan to precautions. what is the best nursing practice for this
case?
A. Use N95 mask all the time
B. Monitor the negative air pressure
C. Use respiratory protective equipment all the ✓
D. Keep the patients with the same diagnosis

213
986- Patient admitted to hospital diagnosed with meningitis.. GCS 9 E2
V3 M4. The Patient is drowsy and loss of weight. Bp 163/80 -Temp 39 -
HR 106 Height 165 -Wight 56 , Most important finding?
A. vital signs instability
B. cognitive impairment
C. Mental state change ✓

987- A mother came to the clinic and afraid that her baby maybe will
have meningitis as his brother already have . What should the nurse tell
the mother?
A. it’s not common for any infant
B. The meningitis has a vaccine now
C. The vaccine is reducing the risk for this decease ✓

988- The nurse assists with a lumbar puncture on a child with suspect
bacterial meningitis. If the diagnosis is correct, the cerebrospinal
fluid, should have which of the following qualities?
A. High glucose level
B. Low protein level
C. Cloudy or turbid appearance ✓
D. Pink or blood-tinged appearance

214
Pyloric stenosis
Pyloric stenosis is an uncommon condition in infants that blocks food
from entering the small intestine.
Normally, a muscular valve (pylorus) between the stomach and small intestine
holds food in the stomach until it is ready for the next stage in the digestive
process. In pyloric stenosis, the pylorus muscles thicken and become
abnormally large, blocking food from reaching the small intestine.

989- Child with pyloric stenosis and de-hydration. What should the nurse
do?
A. Induces Vomiting for child
B. Give oral feeding
C. Start rehydration by nasogastric tube and breast-feeding ✓

990- A 5-week-old newborn admitted pediatric ward with pyloric stenosis


the newborn has weight loss, and projectile vomiting after feeing. Which
of the following abdominal organs are directly affected when stenosis
diagnosed?
A. Stomach and duodenum ✓
B. Stomach and esophagus
C. Liver and spleen

991- PYLORIC stenosis peristalsis movement?


A. From right to left.
B. From left to right ✓

215
992- A 5-week-old newborn was admitted to pediatric Ward with pyloric
stenosis, the newborn has weight loss, and projectile vomiting during
feeding. They scheduled surgical repair of pyloric stenosis which of the
following. Postoperative intervention for this?
A. IV fluid infant is retaining adequate amount by mouth ✓
B. Administration of proper analgesia until infant discomfort resolve
C. Start feeding immediately after postoperative.
D. Vomiting is uncommon in the first24-48 hrs.

993- 1 month-old infant is admitted to the surgical unit with hypertrophic


pyloric stenosis and scheduled for the surgery. Which of the following is
the findings of abdominal examination?
A. palpable olive-like mass in the left side
B. palpable olive-like mass in the right side ✓
C. Palpable olive-like mass moved from left to right.
D. Palpable olive-like mass moved from right to left.

994- The nurse is assessing a child (an infant) with pyloric stenosis. which
of the following is likely to note?
A. Diarrhea
B. Projectile vomiting ✓
C. Swallowing difficulties
D. Currant jelly like stool

995- Surgery for pyloric stenosis movement?


A. Pylorotomy
B. Pylorectomy
C. Pylorostomy
D. Pyloromyotomy ✓

996- Child with Pyloric stenosis. What is the expected signs and
symptoms postoperative?
A. Abdominal pain
B. Watery stool
C. Vomiting ✓
D. Urinary dysuria

216
997- Child came to ER with projectile vomiting and dehydration. The
child diagnosed with pyloric stenosis. What should the nurse expect
developing for?
A. Metabolic alkalosis
B. Metabolic acidosis ✓
C. Respiratory acidosis

998- A 3 week infant with Pyloric stenosis has projectile vomiting, weight
loss and decrease urine output. After operation of pyloric stenosis, which
sings, nurse can expect postoperative during 48 hours?
A. Electrolyte imbalance
B. Cough
C. Mild fever
D. Vomiting ✓

Intussusception
Intussusception (in-tuh-suh-SEP-shun) is a serious condition in which part of
the intestine slides into an adjacent part of the intestine. This telescoping
action often blocks food or fluid from passing
through. Intussusception also cuts off the blood
supply to the part of the intestine that is affected.

999- A 5year-old child was seen to the Emergency Department abdominal


pain, palpable sauge-shaped mass, and Intussusception is suspected
Which of the following is the best diagnostic evaluation to?
A. X-ray
B. endoscopy
C. Rectal biopsy
D. Ultrasonography ✓

217
1000- Nurse is preparing to care for a child with a diagnosis of
intussusception. The nurse reviews the child's record and expects to note
which sign of this disorder documented?
A. watery diarrhea
B. rib bone-like stools
C. profuse projectile vomiting
D. Red jelly stool ✓

1001- A nurse is taking a history from the infant parents who have a
suspected diagnosis of intussusception. Which of the following assessment
question would be most helpful?
A. Do you breast feeding your child?
B. What does your child's stool look like? ✓
C. How often your child urinating?
D. What is the colour of your child' urine?

1002- A 40year old client underwent and exploratory laparotomy with


anesthesia. An assessment of abdominal 36 hours postoperative showed
abdominal distension and an absent of bowel sound. Which complication
is most likely?
A. Paralytic ileus ✓
B. Hemorrhage
C. Rupture colon
D. Intussusception

1003- A 7month old infant seen in the Emergency Department suffering


from episodes of severe abdominal pain, and the infant's stool become
like red jelly. Abdominal examination revealed palpable sausage shaped
mass in the right upper quadrant. Which of the following is the best
diagnosis?
A. Hirsch sprung disease
B. Hypertrophic pyloric stenosis
C. Infant colic
D. Intussusception ✓

218
Spina bifida
Spina bifida is a condition that affects the spine and is usually apparent at
birth. It is a type of neural tube defect (NTD).

Types Of Spina Bifida


The mildest type of spina bifida. It
is sometimes called “hidden”
spina bifida. With it, there is a
small gap in the spine, but no
opening or sac on the back. The
1- Spina Bifida spinal cord and the nerves usually
Occulta are normal. This type usually does
not cause any disabilities.
‫وهو أخف نوع حيث تتكون فجوة صغيرة في‬
‫ ولكن ال يوجد فتحة أو كيس‬،‫العمود الفقري‬
‫في الظهر النخاع الشوكي واألعصاب‬
‫ وهذا النوع عادة ال يسبب أي‬.‫طبيعية‬
‫إعاقات‬
With meningocele, a sac of fluid
comes through an opening in the
baby’s back. But, the spinal cord
is not in this sac. There is usually
little or no nerve damage.
2-Meningocele ‫يتكون كيس من السائل من خالل فتحة في‬
‫ لكن النخاع الشوكي ليس في‬.‫ظهر الطفل‬
‫ عادة ما يكون هناك تلف بسيط أو ال‬.‫الكيس‬
‫يوجد تلف في األعصاب‬
the most serious type of spina
bifida. With this condition, a sac
of fluid comes through an opening
3-Myelomeningocele in the baby’s back. Part of the
spinal cord and nerves are in this
sac and are damaged. This type of
spina bifida causes moderate to
severe disabilities.
، ‫ في هذه الحالة‬.‫هذا النوع األكثر خطورة‬
‫يتكون كيس من السوائل من خالل فتحة في‬
‫ يوجد جزء من النخاع الشوكي‬.‫ظهر الطفل‬
‫ هذا‬.‫واألعصاب في هذا الكيس وهو متضرر‬
‫النوع يسبب إعاقات متوسطة إلى شديدة‬.

219
1004- Infant is born with spinal bifida which of the following
complication is always found in these Infants?
A. Hydrocephalus ✓
B. Craniosynostosis
C. Meningitis
D. Cerebral palsy

1005- Infant came to ER and during the examination, the doctor said he
had spinal bifida occult. What does the doctor know about this condition?
A. Infant has hair patch at back ✓
B. Infant has open wound on back

1006- Neonate in NICU with spina bifida. What is intervention regarding


feeding?
A. Give feeding at scheduled time ✓
B. Limit feeding for neonate
C. Stop feeding. When baby feel back pain during feeding
D. Burping the baby after feeding

1007- Which of the following vitamin supplements can decrease the


incidence of Neural tube defects such as anencephaly and spina bifida
newborn or congenital anomalies?
A. Vitamin A
B. Riboflavin
C. Folic Acid ✓

1008- While caring for a neonate with a meningococcal, the nurse should
avoid positioning the child on the?
A. Abdomen
B. Left side
C. Right side
D. Back ✓

1009- Position for the baby with spina bifida is?


A. Prone position ✓
B. Back
C. Supine

220
1010- Spina bifida risk for which of the following complication?
A. Infection ✓
B. Fever
C. Bleeding
D. Death

1011- All of the following are types of spina bifida EXCEPT?


A. Myelomeningocele
B. Hemophilia ✓
C. Meningocele
D. Spina Bifida Occulta

1012- Infant with spina bifida the nurse should monitor?


A. Head circumference ✓
B. Abdomen circumference
C. Respiratory rate
D. Temperature

1013- A midwife is discussing the birth spacing measures with a mother


whose first baby boy is a 1 year old and was born with spina bifida. The
midwife has explained the possible causes of condition of the baby and
the measures is necessary if the mother is planned for a next pregnancy?
A. Increase iron and calcium supplements
B. Multivitamin and folic acid intake ✓
C. Genetic screening
D. Immunization before and during pregnancy

2014- On the second day of hospitalization for ventriculoperitoneal shunt


revision, a child with spina bifida developed hives, itching and wheezing.
The nurse should determine if the patient has been exposed to?
A. Peanuts
B. Strawberries
C. Eggs
D. Latex ✓

221
1015- A newborn with a myelomeningocele was admitted to the neonatal
intensive care unit 4 hours ago. Which of the following would the nurse
do the when the presents see the child for the first time?
A. Ask the mother to hold, feed and cradle the infant ✓
B. Highlight the infant’s normal and positive features
C. Reassure the parents that the child will be normal after surgery
D. Give the parents verbal and written information about the defect

Tracheoesophageal fistula/ esophageal atresia

Normally, the esophagus (the tube that connects the throat to the stomach) and
the trachea (the tube connecting the throat to the windpipe and lungs) are
separate. A tracheoesophageal fistula (TEF) is an abnormal connection
between these two tubes. As a result, swallowed liquids or food can be
aspirated (inhaled) into your child's lungs.
Feeding into the stomach directly can also
lead to reflux and aspiration of stomach acid
and food. TEF usually occurs with a related
condition called esophageal atresia (EA).

is an abnormal connection (fistula) between


the esophagus and the trachea. TEF is a
common congenital abnormality, but when
occurring late in life is usually the sequela of surgical procedures such as a
laryngectomy?
Tracheoesophageal fistula is suggested in a newborn by copious salivation
associated with choking, coughing, vomiting, and cyanosis coincident with
the onset of feeding

1016- A full-term newborn admitted to the neonatal intensive care unit


with diagnosis of tracheoesophageal fistula. Which of the following
nursing measure to prevent pulmonary complication and improve good
prognosis of the newborn condition?
A. encourage exclusive breast feeding
B. insert nasogastric tube for gastric decompression
C. intermittent suction by double-lumen catheter ✓
D. prepare for the insertion of tracheostomy

222
1017- A newborn admitted to the NICU with tracheoesophageal fistula
(TOF). Which of the following nursing intervention should be included?
A. Elevate the head for feeding
B. Elevating the head but keep the child NPO ✓
C. Insert a nasogastric tube for feeding
D. Encourage the mother to breast feed

1018- Neonate after one day of delivery diagnosed with


tracheoesophageal fistula / atresia. What should the nurse observe for
neonate?
A. continuous crying ✓
B. Projectile Vomiting

1019- With tracheoesophageal fistula and has scheduled for surgery, how
to feed the baby before surgery?
A. gastrostomy
B. Breast feeding
C. NG Tube ✓
D-bottle feeding

1020- Infant with tracheoesophageal fistula TOF. What is the signs that
need suction for him?
A. Crying
B. Chocking and cyanosis ✓
C. Increase heart rate

1021- A child with tracheoesophageal fistula is scheduled for an


operation. what should the nurse do preoperatively?
A. Insert NGT
B. Suction periodically ✓
C. Prepare tracheostomy set if necessary

223
1022- During the home visit by the community health nurse, she observes
that the infant baby during breastfeeding stop sucking and the milk is
leaked from the nose. What would the nurse suspect?
A. Pyloric stenosis
B. esophageal atresia
C. Cleft lip and palate ✓
D. Intussusception

1023- A 2-day-old newborn is admitted to the nursery. While the nurse is


administrating oral feeding, the milk returns through the child's nose and
mouth and the infant becomes cyanotic. Which of the following
condition the newborn should have?
A. Anorectal malformation
B. Tracheoesophageal fistula ✓
C. Cleft lip and palate
D. Cardiac condition

1024- A term baby boy has diagnosed with Down syndrome. Physical
examination revealed flattened nose, low set ears, upward slanting eyes,
single palmer crease. Which of the following is the most common
congenital anomaly associated with the disease?
A. Developmental dysplasia of hip (DDH)
B. Congenital heart disease ✓
C. Hypospadias
D. Pyloric stenosis

224
Hirschsprung disease
Hirschsprung's (HIRSH-sproongz) disease is a
condition that affects the large intestine (colon) and
causes problems with passing stool. The condition is
present at birth (congenital) as a result of missing
nerve cells in the muscles of the baby's colon. Without
these nerve cells stimulating gut muscles to help move
contents through the colon, the contents can back up
and cause blockages in the bowel.

1025- 3-week-old newborn is diagnosed with Hirsch sprung disease which


of the following preoperative care should include it in the plan of care?
A. Restricting oral intake to clear fluids ✓
B. Administering a tap water enema
C. Inserting a gastrostomy tube
D. Using povidone-iodine to prepare the perineum

1026- An 8-month-old infant is admitted with Hirsch sprung disease.


Which of the following would be a significant finding in this infant?
A. Depressed anterior fontanel
B. Polyuria, hematuria
C. Weight gain, edema
D. Failure to thrive, constipation ✓

1027- A 9 month old child is diagnosed with Hirsch sprung disease he is


scheduled for surgical operation which of the following is the most
common complication expected during this age?
A. Mechanical obstruction
B. Entro colitis ✓
C. Pleural effusion
D. Esophageal Artesia

225
1028- A 24 hours after delivery, the nurses noted that the newborn failed
to pass meconium. This indicates which of the following condition?
A. GERD
B. Pyloric stenosis
C. Failure to thrive
D. Hirsch sprung disease ✓

1029- 3 days old newborn is diagnosed with Hirsch sprung disease. The
nurse is conducting a physical examination. Which of the following
findings will alert the nurse to suspect this disease in the newborn?
A. palpable sausage-shaped mass
B. cyanosis of fingers and toes
C. failure to pass meconium within 24-48 hours of life ✓
D. weight less than expected for height and age

1030- What is the Noninvasive test for Hirsch sprung disease?


A. Ultrasound
B. Anorectal manometry ✓
C. X- ray

1031- Which of the following Hirsch sprung disease causes?


A. Congenital ✓
B. Constipation
C. The child is infected at the age of 6 months

1032- A 9 month old child is diagnosed Hirschsprung disease scheduled


for surgical operation which of the following should the nurse understand
the purpose for surgical intervention?
A. To remove the aganglion portion of the bowel to relieve obstruction ✓
B. To maintain optimum nutritional status growth the intestinal
C. To stimulate intestinal adaptation with internal feeding
D. To minimize complication related to the disease

226
1033- Patient came to ER with cold clammy skin and loss of hair in leg.
He has pressure ulcer in both leg. What is the most appropriate nursing
diagnosis?
A. Risk for immobility
B. Risk for injury ✓
C. Risk for fall

1034- A 65 year-old woman presents to her care provider with complaints


of bright red blood in the stool, a loss of appetite, a feeling of fullness
and fatigue. She has lost five kilograms in the past three weeks without
dieting. A faecal occult blood test is positive and the patient scheduled for
an additional screening test. Which of the following screening tests is the
most appropriate?
NOTE: Bright red and blood in stool =
A. Barium enema Colon (B )
B. Colonoscopy ✓ Dark blood = Endoscopy (C )
C. Endoscopy
D. Computed tomography scan

1035- 3 months baby diagnosed with Hirschsprung disease


what is the disease?
A. Symptoms occur after 6 months
B. Affect both small and large intestine
C. Absences of prestalisis movement in the distal part of large
intestine /colon ✓
D. Telescoping of the intestine

1036- Child with recurrent UTI , What is the most risk factors?
A. Abdominal distention
B. Abdominal pain
C. Chronic constipation ✓

1037- What is the 4TS most common for postpartum hemorrhage?


A. Tone ✓
B. Tissue
C. Thrombin
D. Trauma

227
Float Nurse, LPN, RN
1038- Nurse manger float RN nurse from surgical ward to CCU which
patient should be assigned to this nurse?
A. Patient just transferred after coronary angiography
B. Patient postoperative after open heart surgery
C. Patient on I. V lasix have congestive heart failure ✓
D. Patient has ventricular tachycardia and connected to cardiac monitor

1039- Nurse manger pull out (float) nurse from medical ward to CCU
which of the following Patients should be assigned to the nurse?
A. Patient just transferred from cardiac catheterization ✓
B. Patient just received with unstable angina on heparin infusion
C. Patient need discharge education about coronary stenting
D. Patient on Lasix I. V have acute left ventricular failure

1040- Nurse manger pull out (float) nurse from medical ward to CCU
which of the following Patients should be assigned to the nurse?
A. Patient just transferred from coronary artery surgery
B. Patient just received with unstable angina on heparin infusion ✓
C. Patient need discharge education about coronary stenting
D. Patient on admission need assessment

1041- Float nurse from medicine floor. Came to telemetry unite Which
patient she receives?
A. MI patient in heparin infusion
B. Hypertensive patient on Lasix.
C. Atrial Fibrational with anticoagulant drug. ✓

1042- The float nurse is transferring from surgical word to neurological


ward. Which of the following cases should delegate for her?
A. Patient with myasthenia gravis
B. Patient stable with fracture hip and casted ✓
C. Patient with chronic disease and on oxygen

228
1043- You are the RN charge nurse on the medical surgical unit, and you
are in charge of delegating assignments for the shift . Which of the
following would be the most appropriate to delegate to the LPN/LVN?
A. A 25 year old patient who requires IV antibiotic therapy, and needs a PICC
line dressing change before administering the antibiotic
B. A 72 year old patient who requires Lasix 20mg IV push
C. A 50 year old patient who is 3 days post op and requires a simple
dressing change ✓
D. A 45 year old patient who is a newly diagnosed diabetic and requires
discharge teaching

1044- Nurse manger float nurse from surgical ward to CCU which
patient should be assigned to this nurse?
A. Patient just transferred after coronary angiography.
B. Patient postoperative after open heart surgery
C. Patient on I. V Lasix have congestive heart failure
D. Patient had ventricular tachycardia and connected to
cardiac monitor. ✓

Vaccinations
1045- Chicken box vaccine is?
A. Varicella vaccine ✓

1046- 9 month vaccine?


A. MCV4 ✓
B. Hepatitis A
C. Hib
D. MMR

1047- 1 year vaccine?


A. Opv ✓
B. Hib
C. DTap
D. Hepatitis A and varicella

229
1048- 18 month vaccine?
A. Rota
B. Hepatitis B
C. Hepatitis A ✓

1049- Which of the following routes is used to administered Diphtheria,


Tetanus and pertussis DTP vaccine?
A. Oral
B. Intramuscular ✓
C. Subcutaneous
D. Intraderma

1050- DTap vaccine administration route:


A. Intramuscular ✓
B. Subcutaneous
C. Intradermal
D. Intravenous

1051- Measles vaccine administration route:


A. Intramuscular
B. Subcutaneous ✓
C. Intradermal
D. Intravenous

1052- RSV vaccine route for premature baby is?


A. IM
B. IV
C. SC
D. Oral ✓

1053- What of the following disease are prevented by MMR vaccine?


A. Mumps-measles-Scarlet fever
B. Mumps-measles-rotavirus
C. Mumps-measles-rabies
D. Mumps- measles- germen- measles ✓

230
1054- What is the Contraindication of baby vaccine?
A. Antiemetic
B. Antibiotics
C. Steroid ✓

1055- A mother brought her 6-month-old healthy infant to the well-baby


clinic Which immunization should the nurse anticipate to administer as
per World Health Organization's recommendation?
A. Varicella (Chicken pox)
B. Rotavirus and hepatitis
C. Measles, Mumps, Rubella
D. Diphtheria, Tetanus and pertussis ✓

1056- A Rehabilitation nurse reviews post stroke patient immunization


history which immunization is a priority for a 72-year-old patient ?
A. Hepatitis A vaccine
B. Hepatitis B vaccine
C. Rotavirus vaccine
D. Pneumococcal vaccine ✓

1057- The nurse should understand that one dose of which of the
following vaccinations is recommended for this patient before the age of
65 years?
A. Varicella
B. Influenza
C. Hepatitis B
D. Pneumococcal ✓

1058- Side effects of BCG vaccination?


A. Cold and small scar ✓
B. Diarrhea
C. Rash for three days

1059- What are the symptoms of the BCG vaccination side effect?
A. Diarrhea
B. Skin ulcer or scar ✓
C. No symptoms
D. Seizure
231
1060- A pregnant woman informed a nurse that she was never vaccinated
against rubella. Which of the following is the best?
nursing advice?
A. No need for her to be distress, rubella is not harmful to the fetus.
B. The vaccine can be administered any time during her pregnancy.
C. She can get pregnancy any time after receiving the vaccine.
D. She should be vaccinated after delivery of the baby she get
discharge. ✓

9 Questions Mother Of Nine Children

1061- Mother of nine children, three of them with congenital anomales


down syndrome; she is a primary school graduate, with low first status.
She is not using any method of family planning. According primary
health care nurse has refered her counselling. Which of the following
phases of home visit accomplishes intervention?
A. Initial ✓
B. Closing
C. Action
D. Terminal

1062- Mother of nine children, three of them with congenital anomalies


Down syndrome, she is a primary school graduate, with low status. She is
not using any method of family planning. According to primary health
care nurse referred her for counseling which of the following types of
home visits that the community her nurse should conduct for this client?
A. Systematic routine
B. Selective
C. Follow up ✓
D. Field trip

232
1063- Mother of nine children, three of them with congenital anomalies
and one Down syndrome, she is a primary school graduate, with low
financial status, and she is not using any method of family planning.
According, the primary health are nurse has referred her for
counselin .Which of the following is a model concerned with disability as
from of social injustice due to stigma, or discrimination?
A. Health belief
B. Biomedical
C. Sociopolitical ✓
D. Economic

1064- Mother of nine children, three of them with congenital anomalies


and down syndrome she is a primary school graduate, with low financial
status. She is not using any. method of family planning Accordingly, the
primary health care nurse has referred her for counselling the following
terms describe any restriction or lack of ability to perform an activity?
A. Impairment
B. Abnormality
C. Handicapped
D. Disability ✓

1065- Mother of nine children, three of them with congenital anomalies


Down syndrome. she is a primary school graduate, with low status.
She is not using any method of family planning. So, the health care nurse
has referred her. for counselling Which of the following application the
counsellor can help? Mrs. regarding family planning
A. Prevention level
B. Gather model. ✓
C. Group teaching
D. Rejection of Mrs. M expression

233
1066- Mother of nine children, three of them with congenital anomalies
and one down syndrome; she is a primary school graduate, with low
financial status. She is not using any method of family planning.
Accordingly, the primary health care nurse has referred her for
counselling. Which of the following is a barrier facing the team
responsible for providing health services to handicapped individuals?
A. Sensory limitations
B. Rigid rules
C. Developmental disabilities ✓
D. Deafness & hearing limitations

1067- Mother of nine children, three of them with congenital anomalies


and one down syndrome: she is a primary school graduate, with low.
financial status. She is not using any method of family planning. So, the
primary health care nurse has referred her for counseling. Which of the
following indicates that the summarizes the important points during
counseling?
A. Provision broad ideas to the client
B. Restatement for better understanding ✓
C. Keeping silent while the client asking questions.
D. Interpreting feelings and resistance of the client

1068- Mother of nine children, three of them with congenital an down


syndrome; she is a primary school graduate, with status. She is not using
any method of family planning. So Health care nurse has referred her for
counselling which of the following must be focused on by the community
nurse to provide an effective health education?
A. Educate regardless realistic objectives.
B. Use clear and concise language. ✓
C. Use scientific terms during explanation.
D. Explain the negative consequences in the family.

234
1069- Mother of nine children, three of them with congenital anomalies
and one down syndrome; she is primary school graduate, with low
financial status. She is not using any method of family planning. So, the
primary health care nurse has referred her for counseling. Which of the
following is the best health education method that can be used?
A. community organization
B. individual counseling ✓
C. group discussion
D. health class

Types of heat loss from the baby

1070- When the nurse put the baby with high temperature in to cold
water the baby will loose temperature through?
A. Evaporation ✓
B. Conduction
C. Convection
D. Radiation

1071- When put baby with high temperature in cold water to lower his
temperature?
A. Convention
B. Evaporation ✓
C. Conduction
D. Radiation

1072- A nurse is caring for a newborn in Well Born Nursery she warps
the baby with blanket and ensures the nursey temperature is suitable for
the babies. What type of heat loss is the nurse preventing?
A. Radiation
B. Conduction
C. Convention ✓
D. Evaporation

235
1073- How to prevent heat loss via evaporation for neonate after
delivery?
A. Avoid exposure to air draft
B. Avoid contact to wall
C. Dry neonate and cover him, avoid any cold objects ✓
D. Warm any equipments before touching neonate

1074- When assess newborn wight in scale the nurse must avoid?
A. Radiation
B. Evaporation
C. Conduction ✓
D. Convection

1075- Neonate during bath what type of heat loss?


A. Radiation
B. Convection
C. Conduction
D. Evaporation ✓

1076- Neonate is near to cold window what is the type of heat loss?
A. Radiation ✓
B. Convection
C. Conduction
D. Evaporation

1077- How decrease heat loss for neonate by convection?


A. Dry baby
B. Air condition ✓

Research
1078- All the following are considered steps in the qualitative research
process, except ?
A. Literature review NOTE:
Qualitative research = research question Only
B. Data collection Quantitative research = research question +
C. Sample hypothesis
D. Hypothesis ✓
236
1079- Which type of study design provides the strongest evidence?
A. Qualitative study
B. Randomized control trial ✓
C. Systematic review of descriptive studies
D. Systematic review of correlational studies

1080- The nurse was doing research with two groups of smokers, the first
group took Nicotine Patch and the second group not acheived the target
goal, what is the research type?
A. Case Study
B. Cohort study ✓

1081- In a small pilot study, 12 women with endometrial cancer (cancer


of the uterus) and 12 women with no apparent disease were contacted and
asked whether they had ever used estrogen. Each woman with cancer was
matched by age, race, weight, and parity to a woman without disease.
What kind of study design is this?
A. Concurrent cohort
B. Retrospective cohort
C. Case-control ✓
D. Cross-sectional

1082- During the research process, when should a hypothesis be


developed by the researcher?
A. Before any statistical analysis
B. After a research design is determined.
C. Before development of the research question
D. After development of the research question ✓

1083- A handbook on nursing research emphasises on comprehensive


reading and summarising of previous publications related to the topic
Which of the following is the component of this research?
A. Literature review ✓
B. Systematic review
C. Discussion of results
D. Referencing

237
1084- Nurse was providing research study for 200 people, the research is
answered by 120 people only, calculate how many people that shared in
study in percentage?
A. 20 %
B. 40 % NOTE: 120/200 = 0.6 ×100 = 60
C. 80 %
D. 60 % ✓

1085-A nurse researcher asks charge nurse to obtain the patient’s consent.
Which of the following in the most appropriate action?
A. Invite nurse researcher to get patient’s consent directly
B. Make sure patient understands before signing ✓
C. Give consent form topatient for review
D. Read the consent form to patient

1086- A large group of people is followed over 10 years every 2 years its
determined who develops heart disease and who does not what
type of study is this?
A. Case study
B. Cohort study ✓
C. Case- control study
D. Randomized control trial

1087- A study evaluated the relationship between breast cancer and a


woman's history of breastfeeding. The investigator selects women
with breast cancer and an age-matched sample of women who live
in the same neighborhoods as the women with breast cancer. Study
subjects are interviewed to determine if they breastfed any of their
children. This is an example of?
A. Prospective cohort
B. Retrospective cohort
C. Case control ✓
D. Cross sectional

238
1088- A study was done to assess the effect of alcohol on 5000 individuals
was started in 1985, and then it assessed the incidence of liver cirrhosis
between 2005-2008, what is the type of study?
A. Case controlled
B. Retrospective study
C. Concurrent cohort ✓
D. Cross sectional study

1089- A study examined the relationship between exposure to chest


irradiation and subsequent risk of breast cancer that was conducted in
2005. In this study, women who received radiation therapy for
postpartum mastitis (an inflammation of the breast that occurs after
giving birth) in the 1940s were compared to women who received a no
radiation therapy for postpartum mastitis in the 1940s. The women were
followed for 50-60 years to determine the incidence rates of breast cancer
in each group. This is an example of?
A. Prospective Cohort
B. Retrospective Cohort ✓
C. Case Control

1090- In a study begun in 1965, a group of 3,000 adults in Baltimore were


asked about alcohol consumption. The occurrence of cases of cancer
between 1981 and 1995 was studied in this group. This is an example
of?
A. A Cross-Sectional Study
B. A Concurrent Cohort Study ✓
C. A Clinical Trial
D. A case-control study

1091- Residents of three villages with three different types of water


supply were asked to participate in a survey to identify cholera carriers.
Because several cholera deaths had occurred recently, virtually everyone
present at the time underwent examination. The proportion of residents
in each village who were carriers was computed and compared. What is
the proper classification for this study?
A. Cross-Sectional Study ✓
B. Case-Control Study
C. Concurrent Cohort Study D. Nonconcurrent Cohort Stud
239
Assessment

1092- Which of the following uses for abdominal examination?


A. Percussion Inspection. Auscultation. Palpation
B. Auscultation .Percussion .Palpation Inspection
C. Inspection. Auscultation. Percussion. Palpation ✓
D. Inspection. Palpation auscultation .percussion

1093- Which of the following is the most appropriate physical assessment


of a patient with blunt abdominal trauma?
A. Inspect, palpate, auscultate, percuss
B. Inspect, palpate, percuss, auscultate
C. Inspect, percuss, palpate, auscultate
D. Inspect, auscultate, palpate, percuss ✓

1094- A woman patient with , fever, weight loos, and watery diarrhea is
being admitted to health care facility. During assessment the nurse
notices the patient abdomen is concave. additional assessment should
proceed in which order?
A. Palpation, auscultation, and percussion
B. Percussion, palpation, and auscultation
C. Palpation, percussion, and, auscultation
D. Auscultation, percussion, and palpation ✓

1095- What is Newborn examination after delivery?


A. Vital Signs. Inspection .Auscultation. Percussion. Palpation. ✓
B. Inspection .Auscultation. Percussion. Palpation. Vital Signs
C. Palpation Inspection .Auscultation. Percussion. Vital Signs
D. Auscultation. Inspection. Palpation. Percussion. Vital Signs

1096- What is the physical assessment steps for the foot?


A. Inspection , palpitation ✓
B. Palpation , auscultation
C. Percussion, palpation

240
Different Between Cleft Palate and Cleft Lip

Cleft palate Cleft lip

Age of surgery 12 months to 18 months 2 Or 3 months to 6


months

Prone position on the


1- Supine position
stomach to prevent
Position after Or lateral
surgery repair aspiration
‫ألن عمره صغير ما انومه على‬
‫بطنه و ممكن يحصل له اختناق‬
‫و يكون‬

Risk for sudden death


syndrome

2- Cleft lip left side


position after
surgery should be in
the right lateral
‫العملية كانت جهة اليسار‬
‫الوضعية بعد العملية راح تكون‬
‫في الجهة اليمين‬

3- Cleft lip right side


position should be in
the left lateral
‫العملية كانت جهة اليمين‬
‫الوضعية بعد العملية تكون في‬
‫الجهة اليسار‬

Feeding before Special bottle feed


surgery Teaspoon
Feeding after Breast feeding and
surgery Open cup
Bottle feeding

241
1097- Cleft lip repair at which age?
A. 1-2 months
B. 2-3 months
C. 3-4 months
D. 4-6 months ✓

1098- A nurse is caring for a new-born with cleft lip. At which age would
the nurse expect the doctor to perform?
A. 3-6 months ✓
B. 6-10 months
C. 11-14 months
D. more than 14 months

1099- Cleft lip appropriate time for surgery?


A. 2-3 months ✓
B. 3-4 months
C. 4-5 months
D. 5-6 months

1100- The nurse have been teaching a new mother how to feed was born
with a cleft lip and palate before surgical repair of Which of the
following action from the mother indicate teaching has been successful
since?
A. Burping the baby frequently ✓
B. Prevent the infant from crying
C. Placing the baby flat during feeding
D. Keep the infant prone following feedings

1101- The nurse have been teaching a new mother how to feed was born
with a cleft lip and palate after surgical repair of Which of the following
action from the mother indicate teaching has been successful since?
A. Burping the baby frequently
B. Prevent the infant from crying ✓
C. Placing the baby flat during feeding
D. Keep the infant prone following feedings

242
1102- A 2-month-old infant with cleft lip is seen in the primary health
care to get the regular vaccine of 2 months. The mother asked proper
time for the corrective cleft lip surgery of her infant. Which of following
is the best nurse response?
A. No specific age for repair of cleft lip
B. It is too late, repair should be done immediate after delivery
C. The age of 2 months old is the time for repair ✓
D. The proper time for repair after the age of one year

1103-Which goal is priority after surgical repair of cleft lip?


A. Pain Management
B. Prevent infection ✓
C. Increase mobility
D. Develop parenting skills

1104- Which is the priority for a child who has returned to the unit after
surgery to repair a cleft palate?
A. Interacting with others
B. Managing pain ✓
C. preventing infection
D. Ambulating every hour

1105- Establishes baby's hydration A 11-month-old boy with cleft lip &
palate was discharged from the hospital after surgery. The nurse
provided the parents with teaching . Which of the following statement by
the parents indicate appropriate understanding of the teaching
instruction?
A. I should avoid that my infant's cry as much as possible ✓
B. I will use a spoon to feed my child after the surgery
C. I will put my child in his abdomen after feeding
D. I will use lotion to ease my child's lip irritation

1106- After repairing for child with cleft lip left side. Which of the
following position should the nurse put the baby to prevent Aspiration?
A. Prone
B. On stomach
C. Right lateral ✓
D. Left lateral
243
1107- A 4 month old infant returned immediately from OR room post
cleft lip repair which of the following nursing intervention should be
considered?
A. Apply elbow restrain ✓
B. Apply suction when needed
C. Measure temperature
D. Put infant in prone position

1108- During the home visit by the community health nurse she observes
that the infant baby during breastfeeding stop sucking and the milk is
leaked from the nose. What would the nurse suspect?
A. Pyloric stenosis
B. Tracheoesophageal fistula (TEF)
C. Cleft lip and palate ✓

1109- Cleft palate after repair position?


A. Prone ✓
B. Back ( supine)
C. Lateral

1110- A nurse is caring for 14 month - old immediately after a surgical


repair of cleft palate . In which of the following positions should the nurse
put the child?
A. Prone ✓
B. Lateral
C. Supine
D. Lithotomy

1111- How to prevent cleft lip and palate from occurring?


A. Eat green vegetables and citrus fruit ✓
B. Drink too much juice

1112- When should we do cleft palate and lip surgery?


A. After one year
B. After two years
C. After the child is able to hold a cup ✓
D. After improving his awareness

244
1113- Mother came to the Outpatient Department with an infant having
cleft and palate . The infant was underweight, so the nurse has to
consider Teaching the proper way of feeding the child in the treatment
plan . Which of the following is the proper way of feeding?
A. Use a non-squeezable bottle during feeding
B. Feed infant in an upright, sitting position ✓
C. Enlarge nipple holes of bottle to allow more milk to pass through
D. Feed infant longer than 45 minutes to allow more food to be small

1114-What is the action to prevent cleft lip & palate?


A. Give immunizations
B. Advice pregnant folic acid ✓

Period of isolation

A. Chickenpox :
Most common in the Child
Rash start from trunk
Rash fluid filled blisters ( ‫) هي أكثر حاجه تميزه‬
Red eye watery, malaise
Rash dry
Isolation period:
Total period 2 weeks
1- One week before rash appear
2- One week after rash appear ( 5 to 6 days after rash appear )

B. Smallpox
Most common in the Adult
High fever and hard pustules
Isolation period:
Total period 2 weeks (12 to 14 days)
1- One week before rash appear
2- One week after rash appear

245
C. Measles
Skin Rash start from face
Rash red and dry
Malaise
Conjunctivitis
Koplik spot inside the mouth and high fever ( ‫) أكثر حاجه تميزها‬
Isolation period:
Total period 10 days. Note :
1- 5 days before rash appear ‫لو سأل عن فترات العزل بدون ما يحدد قبل ظهورالطفح‬
2- 5 days after rash appear ‫والزم المصاب يعزل نفسه‬, ‫ أيام‬10 ‫الجلدي او بعد نختار‬
‫ ايام‬10 ‫ لكن أقل شي المتوسط‬, ‫ يوم‬15 ‫ الى‬10 ‫من‬

D. Rubella ( Germinal measles)


Skin rash , mild fever
Itching,
Isolation period:
Total period 3 weeks
1- One week before rash appear Note :
2- One week (7 days) after rash ‫ وكان في الخيارات‬after rash ‫لو في السؤال‬
appear or 5 to 7 days after rash ‫ اختار بعد سبعه ايام‬5 days and 7 days

1115- A toddler is seen in the Emergency Room with a history of high


temperature for 5 days. He had watery eyes and a cough for the last
3days, and then he developed a rash all over the body which started in his
face and spread down to his body. The mother did not remember the
vaccination schedule of her son. For how long the child should be
isolated?
A. 5 days ✓
B. 10 days
Note: Started from the face = Measles
C. 15 days
D. 20 days

1116- German measles isolation period after rash?


A. 5 days
B. 1 week ✓
C. 2 weeks
D. 3 weeks
246
1117- A 5 years-old child is seen in the primary care clinic with headache,
malaise for about 2 days and today he has a fluids. For how long the child
should be isolated after formation of?
A. 2 days
B. 6 days ✓
C. 10 days Note: Chickenpox: Malaise, fluids
D. 15 days
1118- What is the difference between measles and German measles?
A. Koplik spot and high fever ✓

1119- How can spread TB?


A. Air droplet ✓
B. Contact physical
C. Blood-
D. Hand to secretion

Questions of Epidemic,Pandemic,Endemic

1120- There is a flu that started in Hong Kong and spread to other areas?
A. Epidemic ✓
B. Endemic
C. Pandemic

1121- Obesity spread in Saudi Arabia is?


A. Epidemic ✓
C. Pandemic
C. Endemic

1122- Unexpected increase case in geographic area?


A. Endemic
B. Epidemic ✓
C. Sporadic
D. Pandemic

247
1123- Influenza in HONG Kong and killed millions of people consider
as?
A. Epidemic ✓
B. Pandemic
C. Endemic

1124- One question ask about Malaria transmission occurs in large areas
of Central and South America, Africa, Asia?
A. Epidemic
B. Endemic ✓
C. Pandemic

1125- Malaria Asia Africa Europe and USA consider as?


A. Endemic ✓
B. Pandemic
C. Epidemic

Pandemic Epidemic Endemic

‫ لو كان على مستوى العالم و‬A disease outbreak is ‫منتشر عند كل السكان في‬
‫يصيب الجميع و منتشر مثل‬ endemic when it is ‫ هو محصور عند فئة‬، ‫البلد‬
Covid-19 ‫مرض كورونا‬ consistently present ‫معينه من السكان و متواجد‬
‫ ألنه‬panademic ‫يعتبر‬ but limited to a ‫في هونج كونج و أنتشر في‬
‫يصيب جميع فئات المجتمع‬ particular region. . ‫اماكن أخرى‬
‫الصغار والكبار عند الجميع‬ This makes the The Centers for
‫و منتشر في كل دول العالم‬ disease spread and Disease Control and
rates predictable. Prevention
The World Health ,Malaria
(CDC) describes an
Organization epidemic as an
(WHO) declares a unexpected increase
pandemic when a in the number of
disease’s growth is disease cases in a
.exponential specific geographical
area. Yellow fever,
Example: Covid-19 smallpox: measles,
Corona virus and polio
248
Questions Bone of Healing
1126- What is timing for bone healing to child 3 years age?
A. 2-4weeks ✓
B. 4-6 weeks
C. 6-8 weeks

1127-What is timing for bone healing to child 5 years age?


A. 3-5 weeks
B. 4-6 week
C. 7-9 weeks ✓

1128- A 9-year-old girl is seen in the emergency department because of a


fracture in the right fibula. Which of the following is the expected
response to parent question about bone healing period of the girl?
A. 4-6 weeks
B. 2-4 weeks
C. 6-8 weeks ✓
D. 8-10 weeks

1129- A 10-year-old girl is seen in the emergency department because of a


fracture in the right fibula. Which of the following is the expected
response to parent question about bone healing period of the girl?
A. 4-6 weeks
Note: Timing healing bone after fracture
B. 2-4 weeks
C. 6-8 weeks Early Childhood: 2 to 4 weeks until 3 years

D. 8-10 weeks ✓ Late Childhood: 8 weeks for age 4,5,6,7,8,9


Adolescent: 8 to 12 weeks for age 10 and above

249
1130- A 13-year-old girl is seen in the emergency department because of a
fracture in the right fibula. Which of the following is the expected
response to parent question about bone healing period of the girl?
A. 4-6 weeks
B. 2-4 weeks
C. 6-8 weeks
D. 8-10 weeks ✓

Nursing Diagnosis / Maslow's Hierarchy

1131- According to Maslow's hierarchy of needs, which nursing diagnosis


has the lowest priority for a client admitted to the intensive care unit with
a diagnosis of congestive heart failure?
A. Impaired urinary elimination
B. Ineffective airway clearance
C. Ineffective coping
D. Risk for body image disturbance ✓

1132- Which of the following taking highest priority according to


Maslow's hierarchy of needs?
A. Enough breathe ✓
B. Safety
C. Love and belong

1133- Women come to ER with her husband and the husband


demonstrate she not talk or voluntary eat since the son died in accident,
what the women need according to Maslow?
A. Loving and belonging ✓
B. Psychological needs
C. Spiritual support
D. Family support

250
1134- Pregnant women came to ER. She diagnosed with preeclampsia .
She was crying and feeling anxious that may affect her baby. What is
describe woman situation according to Maslow hierarchy needs?
A. Self esteem
B. Safety
C. Physiological needs ✓
D. Love and belonging

1135- Using Maslow's hierarchy of needs, rank the following nursing


diagnoses in order of importance, beginning with the highest-priority
diagnosis?
1. Anxiety
2. Risk for infection
3. Disturbed body image
4. Sleep deprivation
Answer: Arrangement 4213 ✓

1136- According to Maslow hierarchy patient with schizophrenia has lack


of motivation to see herself. What are the needs for patient that lost?
A. Physiological need
B. Self esteem ✓
C. Love and belonging
D. Safety

1137- Maslow's hierarchy of needs is an idea in psychology. The priority


nursing care of Maslow hierarchy is?
A. Nursing diagnosis
B. Nursing care
C. Nursing planning ✓
D. Nursing care with scientification

1138- What is the function of Maslow hierarchy for nursing?


A. Set priority ✓
B. Helping establish nursing diagnosis NANDA

251
1139- Patient with schizophrenia. He complains with anxiety episodes
which needs is highest priority in Maslow hierarchy?
A. Physiological needs
C. Self esteem
B. Safety ✓
D. actualization

1140- Long Situation , teacher don’t want be tired front his student
Maslow's hierarchy of need?
A. Self esteem
B. Safety and Security ✓

1141- A child with deformity (broken) nose, the child went to the school
and his friends find this funny, the child was upset and went to the nurse
in the school and told him, he will stop coming to school, the nurse tokes a
paper and draw the child face and nose and tell him that ‘he will look like
them after the procedure’. Which of the following step in Maslow
hierarchy missed for child?
A. Self conception
B. Self deception
C. Self esteem ✓

1142- A child with deformity (broken) nose, the child went to the school
and his friends find this funny, the child was upset and went to the nurse
in the school and told him, he will stop coming to school, the nurse tokes a
paper and draw the child face and nose and tell him that ‘he will look like
them after the procedure’. The step the nurse perform is called?
A. Self conception
B. Self confidence ✓
C. Self esteem

1143- Patient with Dissociative disorder. Which of the following nursing


diagnosis should the nurse expect?
A. Impaired cognitive process
B. Self image deficit ✓

252
1144- Community health nurse visited Alzheimer's disease patient. The
patient complained from bruises and many injuries. The caregiver denied
any fallness caused for patient. What is the Nursing diagnosis for this
patient?
A. Impaired memory related to disease process
B. Risk of injury related to lack of judgment, impaired cognitive,
neglecting of caregiver ✓
C. Impaired physical mobility related to immobilization

1145- A patient with server diverticulitis had surgery for placement of a


colostomy. The patient is upset, crying, and will not look at the colostomy.
Which of the following would be the highest priority nursing diagnosis at
this time?
A. Knowledge deficit, colostomy care
B. Distorted body image ✓
C. Self-care deficit, toileting
D. Alteration in comfort

1146- The patient complained from stomachache continue after eating for
2 to 3 hours The patient has increased weight 3 kg during short time.
Which of the following is the most important nursing diagnosis?
A. Imbalance nutrition
B. Acute pain ✓
C. Fluid volume deficit

1147- Patient came to ER with cold clammy skin and loss of hair in leg.
He has pressure ulcer in both leg. What is the most appropriate nursing
diagnosis?
A. Risk for immobility
B. Risk for injury ✓
C. Risk for fall

253
1148- A 40-year-old man is admitted to a Coronary pain. The ECG has
normal sinus rhythm with leads V1-V4. Blood pressure 123/69 mmHg.
What is the most likely nursing diagnosis?
A. Acute chest pain ✓
B. Myocardial in fraction
C. Decreased cardiac output
D. Ineffective tissue perfusion

1149- The comatosed patient with NGT for feeding and positioned in low
fowler. The nurse enter the patient room. She found the patient in supine
position. She auscultated lung sound with diventure. What is the most
appropriate nursing diagnosis?
A. Risk for injury
B. Risk for aspiration due to NGT ✓

1150- You are preparing the nursing care plan for a middle-aged patient
admitted to the intensive care unit for an acute myocardial infarction
(heart attack). His symptoms include tachycardia, palpitations, anxiety,
jugular vein distention, and fatigue. Which of the following nursing
diagnoses is most appropriate?
A. Decreased Cardiac Output ✓
B. Impaired Tissue Perfusion
C. Impaired Cardiac Contractility
D. Impaired Activity Tolerance

1151- A male patient has an HIV and has a wife and son , his wife is
depressed due to his diagnosis What's the immediate goal?
A. Risk for infection related to low immunity ✓
B. Risk for infective psychosocial support

1152- What is the Nursing diagnosis of chest radiography tuberculosis?


A. Ineffective breathing pattern related to tuberculosis
B. Infection related disease
C. Difficult breathing related to secretion
D. Ineffective breathing pattern related to secretion ✓

254
1153- A client diagnosed with somatization disorder visits multiple
physicians because of var- ious, vague symptoms involving many body
systems. Which nursing diagnosis takes priority?
A. Risk for injury related to treatment from multiple physician
B. Anxiety related to unexplained multiple somatic symptoms
C. Ineffective coping related to psychosocial distress

1154- patient with a stroke is paralyzed on the left side of the body and
has developed a pressure ulcer on the left hip. The best nursing diagnoses
for this patient is ?
A. Impaired physical mobility related to left-sided paralysis
B. Risk for impaired tissue integrity related to left-sided weakness
C. Impaired skin integrity related to altered circulation and pressure ✓
D. Ineffective tissue perfusion related to inability to move independently
Note: High risk for pressure ulcer ‫لو سأل عن‬
‫اإلجابة تكون فقرة‬
D. Ineffective tissue perfusion related to
inability to move independently

1155- planning the care for a client who has pneumonia, the nurse collects
data and develops nursing diagnoses. Which of the following is an
example of a properly developed nursing diagnosis?
A. Ineffective health maintenance as evidenced by unhealthy habits
B. Ineffective airway clearance as evidenced by inability to clear
secretions ✓
C. Ineffective therapeutic regimen management due to smoking
D. Ineffective breathing pattern related to pneumonia

1156- An autistic child makes no eye contact, unresponsive to


continuously spins, twist and head bang. Which of the following is the
priority nursing diagnosis?
A. Risk of injury related to head banging ✓
B. Impaired verbal communication related to physical
C. Personal identity disorder related to poor ego develop
D. Social isolation related to unresponsiveness toward

255
1157- Intrauterine growth curves were used to classify a 32-week-old
preterm new-born. Birth weight and gestational age shows the infant's
growth rate falls below the 10th percentile. What is the priority nursing
diagnosis for a new-born with small for gestational age?
A. Risk for injury related to impaired gluconeogenesis
B. Risk for impaired gas exchange related to meconium aspiration
C. Risk for ineffective thermoregulation related to lack of subcutaneous
fat ✓
D. Risk for altered nutrition less than body requirement related to increased
metabolic needs

1158- A 22-year-old patient is admitted in the male diagnosis of tonsillar


abscess. He has high fever along with dysphagia, difficulty in talking and
patient is planned for needle aspiration of the intravenous antibiotic
including penciling. What is nursing problem need attention first?
A. An imbalance nutrition due to inadequate
B. Acute pain related to throat inflammation
C. impaired swallowing related to dysphagia ✓
D. hyperthermia related to acute infection

1159- Immediately following the birth of a full-term newborn, which of


the following is the priority nursing diagnosis for this newborn?
A. Airway clearance related to nasal & oral secretions ✓
B. Ineffective thermoregulation related to environmental factors
C. Risk for imbalance fluid volume related t weak sucking reflex
D. Risk for injury related to immature defense mechanisms

1160- The nurse cares for a client in the post anesthesia care unit (PACU)
after general anesthesia was given for an open small bowel surgery. The
nurse determines which nursing diagnosis is the most important when
caring for the client?
A. Impaired transfer ability related to sedation as evidenced by drowsiness
B. Nausea related to manipulation of intestines as evidenced by client
report ✓
C. Risk for infection related to recent open abdominal surgery
D. Acute pain related to surgery as evidenced by pain score of 4 out of 10

256
1161- A 72-year-old women is admitted in the Neurosurgery Ward
hairline fracture on her right cranial region due to a fall 24 had a brief
unconsciousness at the time of the fall and has bruises on her face and
right arm. She is complaining of pain at her right hip. Blood pressure
90/58 mmHg Heart rate 102 /min Respiratory rate 27 /min
Temperature 37.8C What nursing diagnosis should be focused on first?
A. Risk of fall and injury due to senility
B. Self-care deficit due to inability and weakness
C. Risk of altered consciousness due to head injury ✓

D. Hyperthermia due to injury response and infection

1162- Patient admitted to medical unit with paraplegia. The patient can
transfer himself by wheel chair but need some assistance. The nurse
inserted foley catheter and made urine analysis Ph of urine 6 normal
range 2-8 Urine color yellowish. What is the priority nursing diagnosis
for him?
A. High risk for fall ✓
B. Risk for UTI

1163- A nurse is reviewing the plan of care for a client with a breathing
problem. Which of the following would the nurse most likely expect to
identify as a relevant nursing diagnostic statement for this client?
A. Altered airway
B. Impaired respiration
C. Impaired breathing rate
D. Ineffective airway clearance ✓

1164- A 32 year old man is admitted to the isolation unit with acquired
immune deficiency syndrome. He is married and has one child. His wife is
a depressed because of his diagnosis. Which of the following nursing
problems requires immediate planning?
A. Risk of infection due to autoimmune cell destruction ✓
B. Nutrition deficit due to due body cell mass wastage
C. Risk of ineffective psychosocial and family support
D. Knowledge deficit about prognosis and treatment

257
1165- A 70-year-old woman is admitted to the Cardiac Care Unit with
new onset atrial fibrillation and is receiving intravenous diltiazem and
heparin. What is the most likely nursing diagnosis?
A. High risk for infection
B. High risk for impaired gas exchange
C. High risk for decreased cardiac output ✓
D. High risk for disturbed sensory perception

1166- A 25-year-old woman present to the Emergency Room with acute


gastritis and signs of moderate dehydration. Which is the most
appropriate nursing diagnosis?
A. Infection
B. Fluid volume deficit ✓
C. Activity intolerance
D. Fluid volume excess

1167- A 68-year- old woman is receiving parental nutrition at home. The


district nurse visits the woman and notes that has gained one kilogram of
weight since a health provider had visited one week ago. There is pitting
edema of 2+ of lower extremities. The patient is alert, active and oriented.
Which nursing diagnosis is most appropriate?
A. Non-compliance
B. Impaired gas exchange
C. Imbalanced nutrition
D. Fluid volume overload ✓

1168- A client with a blunt trauma underwent an exploratory lap


repair an intra abdominal injury closed. A nasogastric tube attached
suction and two wound Hemovac abdominal drains. He is receiving an
intravenous infusion of Ringer’s lactate epidural with continuous
morphine. Twenty oliguria and sever four hour post develops
hypotension, tachycardia, nausea. Which of the following is the
priority nursing diagnosis?
A. Nausea
B. Risk for infection
C. Deficient fluid volume ✓
D. Impaired urinary elimination

258
1169- A nurse evaluates dietary practices of a patient who had been as
having acute glomerulonephritis. The patient verbalizes following the
appropriate diet for acute glomerulonephritis. Which of the following diet
verbalized by the patient is approve diagnosis?
A. Restrict fluid intake ✓
B. Restrict dietary protein
C. Increase intake of low-fiber foods
D. Increase intake of sodium-rich foods

1170- Which of the following nursing diagnoses has the highest priority
when caring for an older adult client with Alzheimer's disease?
A. Impaired physical mobility
B. Impaired memory
C. Self-care deficit
D. Risk for injury ✓

1171- A nurse is assigned to care for a patient with physical immobility


due to right knee injury. the nurse is preparing to write nursing diagnosis.
Which of the following is a priority for nursing diagnosis?
A. Pain
B. Hygiene
C. Dehydration
D. Skin integrity ✓

1172- A patient recently underwent coronary artery graft (CABG)


surgery. Which of the following nursing diagnose PRIORITY?
A. Anxiety
B. Impaired gas exchange
C. Acute pain ✓
D. Sleep deprivation

1173- After a code blue, a doctor announced the death of a patient. A


signature other begins to show signs of decreased level of consciousness.
What is the priority intervention of the nurse at this time?
A. Offer a meal
B. Ensure safety ✓
C. Set up an IV line
D. Offer a glass of water
259
1174- Child has a third- degree burns of the hand, chest, face, which
nursing diagnosing takes priority?
A. Ineffective airway clearance ✓
B. disturbed body image
C. impaired urinary eliminations
D. risk for infection

1175- A women patient with a body mass index of 35 is admitted to ward.


Which of the following nursing diagnosis, related to Imbalance the most
appropriate?
A. Activity intolerance
B. Less than body requirement
C. More than body requirement ✓
D. Deficient knowledge on normal nutrition

1176- A nurse diagnosis a patient with readiness for the This diagnosis is
classified within which of the Nursing diagnoses?
A. Acute nursing diagnoses
B. Risk nursing diagnoses
C. Wellness nursing diagnoses ✓
D. Possible nursing diagnoses

1177- The nurse has been set priorities for health problem. She is starting
to management of time board. Which of the following step of nursing
process?
A. Planning ✓
B. Diagnosis
C. Intervention
D. Assessment

1178- Nursing diagnosis done and the nurse set priorities for all NANDA.
Which step in nursing process the nurse doing?
A. Planning ✓
B. Diagnosis
C. Evaluation
D. Implementation

260
1179- Which phase of nursing process, the nurse collect data about the
client:
A. Diagnosis
B. Assessment ✓
C. Implementation
D. Evaluation

1180- A 75-year-old bedridden patient is hospital Medical ward for the


treatment of her in Area. Her wound is infected with multiple Longer
than normal time to treat. She Crying spells and is talking about death
Which of the following nursing diagnosis must be?
A. Health and wellness
B. Coping mechanism ✓
C. Self-perception
D. Belief system

1181- A 45 year-old man who is hospitalized feels tge constant need to


keep things in order , particularly whilst eating the nurse observe him
arranging the food on his plate into symmetrical and equal bite sized
pieces , he constantly worries that the food served could be outdate and
potentially causes illness. Which nursing diagnosis is most important
A. Ineffective verbal communication
B. Self-esteem disturbance
C. Impaired social interaction
D. Anxiety ✓

1182- A pregnant woman had traffic accident and she came to the
emergency department during the nursing assessment Palpation from the
nurse the woman has acute pain and possible bleeding. Which of the
following nursing diagnosis should the nurse expect?
A. Abruption placenta ✓
B. Pain

261

You might also like